Avaruus.fi - keskustelualue

Tähtiharrastus ja tähtitiede => Kosmologia => Aiheen aloitti: Lorelei - 09.05.2015, 09:50:04

Otsikko: Musta aukko ja sen muoto, tapahtumahorisontti
Kirjoitti: Lorelei - 09.05.2015, 09:50:04
Minulla olisi - varmaankin alkeellinen - kysymys mustasta aukosta ja sen muodosta. Luin jostain, että se on (ehkä?) suppilomainen, tai torvimainen muodoltaan. Mitä siis on mustan aukon, hm, takana? Imeekö se itseensä kaiken myös toisesta päästä havaittuna?
Toinen kysymys; onko tapahtumahorisontti vain eräänlainen (ajateltu) rengas mustan aukon ympärillä vai kaareutuuko se myös ikään kuin kupolimaisena musta aukon ylitse? Eli jos valtava musta aukko imee itseensä materiaa ns suoraan edestäpäin (eikä sivusta, ohittaen tapahtumahorisontin aukon reunassa), kohtaako se myös silloin tapahtumahorisontin?


Otsikko: Vs: Musta aukko ja sen muoto, tapahtumahorisontti
Kirjoitti: Umbra - 09.05.2015, 11:12:28
Suosittelen lukemaan vaikkapa wikipedian artikkelin mustista aukoista, jos et ole jo lukenut, siinä näkyy ihan mukavasti olevan perustietoa. (http://fi.wikipedia.org/wiki/Musta_aukko)

Käsitykseni mukaan tapahtumahorisontti on pallomainen pinta singulariteetin ympärillä. Singulariteetti taas voidaan hahmottaa äärettömän pienenä pisteenä mustan aukon keskipisteessä. Musta aukko tosiaan joskus kuvataan torvimaisena. Tässä on käsittääkseni kyseessä avaruuden kaarevuuden kuvaaminen alempiulotteisen analogian tai vertauskuvan avulla, eli torvimaista muotoa ei pidä ymmärtää konkreettisesti.

En ehkä ihan ymmärrä kysymystäsi mitä on mustan aukon takana. Joissain tilanteissa (ainakin pyörivän mustan aukon tapauksessa) musta aukko voisi teoriassa toimia ikäänkuin porttina toiseen maailmankaikkeuteen. Tässä tapauksessa mustaan aukkoon pudonnut astronautti voisi putkahtaa esiin valkeasta aukosta toisessa maailmankaikkeudessa. (Tämä tilanne voitaisiin torvi-vertauskuvan avulla hahmottaa torven läpi kulkemisena.) Valkeassa aukossa on todennäköisesti kyseessä epäfysikaalinen suhteellisuusteorian ratkaisu, joka on suhteellisuusteorian matemaattisen rakenteen mukaan mahdollinen, mutta ei toteudu todellisessa maailmankaikkeudessa. Yhtään valkeaa aukkoa ei olla havaittu, ja jos sellasia olisi olemassa, ne ilmeisesti olisivat äärimmäisen epävakaita.

Jos kiinnostaa perehtyä asiaan englanniksi, vähän enemmän asiaa löytyy täältä: http://www.madore.org/~david/math/kerr.html#course
Otsikko: Vs: Musta aukko ja sen muoto, tapahtumahorisontti
Kirjoitti: Lorelei - 09.05.2015, 11:47:20
Kiitos vastauksesta!



Otsikko: Vs: Musta aukko ja sen muoto, tapahtumahorisontti
Kirjoitti: Celest1al Sphere - 06.09.2015, 19:03:18
En ole kosmologi enkä teoreetikko, mutta käsittääkseni on niin, että musta aukko voidaan ymmärtää jokseenkin pyöreänä pallona. Tapahtumahorisontti on vain sellainen pallonmuotoinen pinta, joka rajaa sen alueen, jonka sisäpuolelta ei edes valo voi päästä karkuun. Tapahtumahorisontti ei siis ole mikään konkreettinen pinta. Singulariteetti taas on sellainen matemaattinen kummajainen, joka nykyteorioiden mukaan lymyää mustan aukon ytimessä. Singulariteetti on myös käsittääkseni aika teorieettinen juttu, ja se oikeastaan tarkoittaa tilannetta, jossa matemaattiset mallit "menevät rikki", eivätkä enää pysty antamaan tarkkoja lukuarvoja vaan saadut lukuarvot äärettömiä. Lienee myös niin, että jotkin fyysikot ovat sitä mieltä, että jos teoria tuottaa singulariteetteja, on siinä jotain vikaa, koska singulariteetteja ei heidän mukaan voisi esiintyä todellisessa maailmankaikkeudessa.

Eli se, mitä mustan aukon sisällä todella on, on edelleen epäselvää!

Terveisin,

Juha
Otsikko: Vs: Musta aukko ja sen muoto, tapahtumahorisontti
Kirjoitti: mistral - 07.09.2015, 00:28:08
Lainaus käyttäjältä: Aldebaran - 06.09.2015, 19:03:18
Eli se, mitä mustan aukon sisällä todella on, on edelleen epäselvää!

Sikäli kuin ymmärrän, ei materia ole pakkautunut mustan aukon ytimeen vaan kuoreksi tapahtumahorisontin pintaan mutta tämä vain ajatuksena, en ole spesialisti. Tämä kuori johtuu siitä kun aika pysähtyy horisontissa, eli yksi sekunnin murto-osa siellä vastaa ikuisuutta täällä vapaassa avaruudessa.

edit
Yön yli nukuttuani muistui se että kun musta-aukko on historiansa aikana kasvanut, on sen horisonttikin kasvanut. Näin kaikki materia ei enää ole sen kuoressa vaan sitä mukaa kun horisontti kasvaa, tekee se uusia kuorikerroksia kuin sipuli. Mutta niin kuin Aldebaran sanoi, mitään varmuutta näistä ei ole.
Otsikko: Vs: Musta aukko ja sen muoto, tapahtumahorisontti
Kirjoitti: Umbra - 08.09.2015, 22:46:05
Mustan aukon sisuskalut taitavat edelleen olla erittäin hämäräperäinen ja huonosti ymmärretty asia. Homma menee suhteellisuusteorian mukaan käsittääkseni siten, että kaukana olevan havaitsijan näkökulmasta mustaan aukkoon putoava astronautti ei koskaan läpäise tapahtumahorisonttia, vaan jäätyy sen pintaan litteäksi "haamuksi". Mutta astronautin omasta näkökulmasta hän läpäisee horisontin huomaamatta mitään erikoista ja päätyy singulariteettiin äärellisen (ja varsin lyhyen) ajan kuluessa.

Joskus tulkitsin tämän siten, että singulariteettia ei ole tällä hetkellä olemassa, vaan se sijaitsee äärettömän kaukaisessa tulevaisuudessa. Mutta tämä tulkinta ilmeisesti on virheellinen. Physics.stackexchange.com -sivuston mukaan yleisessä suhteellisuusteoriassa ei voida määritellä yksikäsitteistä nykyhetkeä, eikä siis ole mielekästä kysyä onko astronautti "tällä hetkellä" läpäissyt horisontin. "Nykyhetki" voidaan määritellä siten, että astronautti on läpäissyt horisontin ja saavuttanut singulariteetin, tai vaihtoehtoisesti siten, että astronautti on jäätyneenä horisontin pintaan. Kumpikin määrittely on yhtä lailla oikeassa. Tämä tietysti lyö arkijärkeä raskaasti korville, koska arkijärjen näkökulmasta tekisi mieli uskoa yhteen perimmiltään oikeaan nykyhetkeen. (Lähde: http://physics.stackexchange.com/questions/5031/can-black-holes-form-in-a-finite-amount-of-time )

Eräs nykyään pohdittu ratkaisu mustien aukkojen informaatioparadoksiin on black hole complementarity, jonka mukaan musta aukko voidaan hahmottaa kahdesta vaihtoehtoisesta, komplementaarisesta näkökulmasta. Toisen näkökulman mukaan mustaan aukkoon pudotettu informaatio (ja luullakseni myös aine) varastoituu Planckin etäisyyden paksuiseen kalvoon horisontin yläpuolella. Tämä kalvo on kuuma ja säteilee Hawkingin säteilyä, jonka mukana informaatio voisi paeta mustasta aukosta (tai tarkemmin sanoen sen lähiympäristöstä). Toisen näkökulman mukaan informaatio putoaa horisontin läpi ja päätyy singulariteettiin. Nämä näkökulmat ovat komplementaarisia samoin kuin kvanttimekaniikan hiukkas- ja aaltonäkökulmat; yksikään havaitsija ei voi havaita molempia näkökulmia samaan aikaan. Ilmeisesti molemmat näkökulmat kuitenkin ovat "yhtä oikeita". Komplementaarisuus synnyttää helposti mielikuvan, että informaatiosta on olemassa kaksi kopiota, toinen horisontin yläpuolisessa kalvossa ja toinen horisontin sisällä putoamassa kohti singulariteettia. Tämän kuitenkin estää no cloning theorem, jonka mukaan tuntemattomasta kvanttitilasta ei ole mahdollista luoda kahta identtistä kopiota. Eli ilmeisesti mustan aukon sisäosat ja pinta ovat ikäänkuin kaksi näkökulmaa samaan todellisuuteen.  (Lähde: https://en.wikipedia.org/wiki/Black_hole_complementarity )
Otsikko: Vs: Musta aukko ja sen muoto, tapahtumahorisontti
Kirjoitti: mistral - 09.09.2015, 18:58:14
Lainaus käyttäjältä: Eusa - 09.09.2015, 10:52:43
ja silloin tapahtumahorisontit ovat vain asymptootteja, joita ei voi saavuttaa kaukaisen tarkkailijan mittaus, eikä fyysinen lähestyminen. Ratkaisuni on, että lähestyessään virtuaalista tapahtumahorisonttia kappaleet pienenevät / tila laajenee.

Jos avaruusalus lähestyy tapahtumahorisonttia, siirtyy horisontti sisäänpäin ihan sen mukaan missä valon pakonopeus ylittyy. Eli horisontti on vaan laskennallinen gravitaatiokaivon syvyys. Jos esim 3 alusta liikkuisi g.kaivossa eri syvyyksillä olisi kunkin asemasta laskettu horisontti eri. Tähän ajatusmalliin sisältyy sellainen kummallisuus että syvimmällä oleva alus voisi lähettää ylimpänä olevalle dataa ylimmän horisontin alapuolelta, mikä on tavallaan mahdotonta.
Otsikko: Vs: Musta aukko ja sen muoto, tapahtumahorisontti
Kirjoitti: mistral - 10.09.2015, 22:28:33
Lainaus käyttäjältä: Umbra - 08.09.2015, 22:46:05
Mustan aukon sisuskalut taitavat edelleen olla erittäin hämäräperäinen ja huonosti ymmärretty asia. Homma menee suhteellisuusteorian mukaan käsittääkseni siten, että kaukana olevan havaitsijan näkökulmasta mustaan aukkoon putoava astronautti ei koskaan läpäise tapahtumahorisonttia, vaan jäätyy sen pintaan litteäksi "haamuksi". Mutta astronautin omasta näkökulmasta hän läpäisee horisontin huomaamatta mitään erikoista ja päätyy singulariteettiin äärellisen (ja varsin lyhyen) ajan kuluessa.

"Nykyhetki" voidaan määritellä siten, että astronautti on läpäissyt horisontin ja saavuttanut singulariteetin, tai vaihtoehtoisesti siten, että astronautti on jäätyneenä horisontin pintaan. Kumpikin määrittely on yhtä lailla oikeassa.

Itse ymmärrän kyllä nyt-hetken olevan voimassa riippumatta ajankulun erilaisuudesta.

Siihen löytyi sellainen apukeino kuin a-nelosen paperi johon piirsin yläriville aallon vaikka kahden aallonpituuden mittainen, eli kaksi huippua ja laaksoa. (vapaa avaruus)

Sitten vaikka 5 senttiä alemmas aallon jossa olisi 3 huippua  (gravitaatiokaivo jossa ollaan 1,5x hitaammassa ajassa)

Ja sitten kolmas aalto jolla on neljä huippua. (nyt ollaan 2x hitaammassa ajassa)

Eli nämä aallot kuvaavat yhden ja saman aallon punasiirtymää kun se nousee vapaaseen avaruuteen.

Nyt tulee kysymys: Jos aikadilataatiota ei olisi olemassa, niin kuinka olisi mahdollista esim soittaa kännykällä sieltä 2x hitaammalta aikavyöhykkeeltä vapaaseen avaruuteen?

Kun alhaalta yritetään soittaa vaikka 5 min puhelu, muuttuisi 10 min puheluksi VAIKKA aikadilataatiota ei olisi (koska kännykän kaikki signaalit perustuu sm-aaltoihin) Edelleen, jos puhelua jatkettaisiin vaikka 5 tuntiin, ylhäällä se kestäisi 10 tuntia. Edelleen 5v, ylhäällä se kestäisi 10 vuotta. Kun nyt ero puhelun lopun välillä on 5 vuotta, joutuisi yhteinen nyt-hetki roskikseen.

Miten nyt-hetki saadaan takaisin voimaan?

Ottamalla aikadilataatio käyttöön, mikä yleisen suhteellisuusteorian kulmakivi.

Joku voi tietysti kysyä että miksi aika on pakko sitoa sm-aaltoliikkeeseen, eikö sitä voisi ajatella itsenäisenä suureena? Jos vaan päätettäis että aika on sama ylhäällä ja alhaalla, niin mitä siitä seuraisi.

Jos mennään taas puheluun ja päätetään että puhelun on oltava yhtä pitkä molemmissa päissä niin silloin jouduttaisiin pakkomuokkaamaan aaltoja vaikkapa pätkimällä aaltoja tietyn pituisiksi vaikka punasiirtymä protestoisi vastaan. Tässä vissiin kävisi niin että aallot menisivät muodottomiksi vänkyröiksi mikä on mahdottomuus. Siinä a-nelosen paperissa näkee ettei eri pituisia aaltoja voi yhdistää yhdeksi aalloksi, luonto ei siihen taivu.

Näin näyttäisi aikadilataatio olevan ainut ratkaisu, se lienee luonnon ominaisuus. Ihmisen intuitio ei vaan voi ymmärtää kuinka tapahtumahorisontissa näyttäisi liike pysähtyvän vaikka horisontin näkökulmasta nopeus on maksimissaan.
Otsikko: Vs: Musta aukko ja sen muoto, tapahtumahorisontti
Kirjoitti: mistral - 16.09.2015, 19:49:55
Lainaus käyttäjältä: Joksa - 16.09.2015, 11:17:07

Tapahtumahorisontissa ajan hidastuminen aina nollaan saakka on mielenkiintoinen. Se tarkoittaa että ulkopuolisten tarkkailijoiden aika nopeutuu äärettömiin tähän hidastujaan verrattuna, eli vaikka ko hidastajan mielestä hänen aikamittarinsa tikittää entiseen tahtiin niin muun kosmoksen aika kiittää hurjaa vauhtia ja saapuu kosmisen elinkaarensa päähän aivan mitättömän hetken kuluessa.

Ajan matemaattisessa kaavassa aika kääntyy tämän jälkeen eli tapahtumahorisontin sisällä negatiiviseksi. Sen fysikaalista vastinetta on vaikea mahduttaa tajuntaan. Kaava ei enää toimine koska merkitsisi singulaariteettiin matkaajan aikamatkaa takaisin bb:hen.

Aika ei vissiin paikallisesti koskaan muutu, tarkoitan jos astronautti jatkaa horisontin läpi, niin paikallinen aika toimii tavallisesti, ei mene miinukselle. Mutta miltä astronautista näyttäisi vapaa avaruus jos sen koko historia jo horisontissa näyttäytyy ikään kuin salamavalon välähdyksenä. Mitä sen välähdyksen jälkeen tulisi, kuinka filmi voisi lähteä pyörimään taaksepäin?
Otsikko: Vs: Musta aukko ja sen muoto, tapahtumahorisontti
Kirjoitti: Lorelei - 22.09.2015, 20:24:31
Tämä ei nyt liity ihan aiheeseen, mutta jotenkin on ristiriitaista, että esim fyysikot (luonnontieteilijät jne) sanovat selittävänsä "kaiken" oman tieteellisen systeeminsä puitteissa. Mitään mystistä ei ole, ja sitä rataa. Silti singulariteetti "selitetään" sillä, ettei se nyt (hupsis) mahdu kontekstiin, se on siis se Juttu aukottomassa tieteenteossa joka ei mahdu kuvioon... :azn: Se on siis järjen ulottumattomissa, mitä ei tosin kai saisi sanoa ääneen.



Otsikko: Vs: Musta aukko ja sen muoto, tapahtumahorisontti
Kirjoitti: mistral - 23.09.2015, 11:50:06
Myös M-teoria on kesken, enkä ole kuullut siitä mitään uutisia. Higgsin hiukkasesta en ole vielä kuullut että löydetty olisi 100% varmasti Higgs, varmaan sekin selviää. Myös monet luonnonvakiot on arvoituksia, koska ei tiedetä mistä ne saavat arvonsa.
Otsikko: Vs: Musta aukko ja sen muoto, tapahtumahorisontti
Kirjoitti: jussi_k_kojootti - 26.10.2015, 23:24:55
Lainaus käyttäjältä: Lorelei - 22.09.2015, 20:24:31
Tämä ei nyt liity ihan aiheeseen, mutta jotenkin on ristiriitaista, että esim fyysikot (luonnontieteilijät jne) sanovat selittävänsä "kaiken" oman tieteellisen systeeminsä puitteissa.

Juuri tuolla tavalla ilmaistuna -- eli kun selityksen reunaehdot ilmoitetaan -- asiassa vaan nimenomaan ei ole mitään ristiriitaista :-)

Lainaa
Mitään mystistä ei ole, ja sitä rataa. Silti singulariteetti "selitetään" sillä, ettei se nyt (hupsis) mahdu kontekstiin, se on siis se Juttu aukottomassa tieteenteossa joka ei mahdu kuvioon... :azn: Se on siis järjen ulottumattomissa, mitä ei tosin kai saisi sanoa ääneen.

Kyllä singulariteetti mahtuu kuvioon, ja esim. täältä maasta katsottuna Sgr A* joka sellaisen sisäänsä ehkä sulkee on periaatteessa aivan ymmärrettävä, käsiteltävissä oleva olio.  Tietenkin systeemin puitteissa! :-)  Singulariteetti on kuviossa "tuntematon muuttuja", mutta ei sen olemassaolo, tai edes olemassaolon mahdollisuus, ole ollenkaan niin todellisuuden rakenteita järisyttävä poikkeama, kuin annat ymmärtää.

Tiede ei yritä antaa keskeneräistä vastausta, ja vastaukset singulariteetista ovat vääjäämättä keskeneräisiä ennen kuin sellaisesta voidaan tehdä suoria havaintoja.  Spekuloida voi, saa, ja pitää.
Otsikko: Vs: Musta aukko ja sen muoto, tapahtumahorisontti
Kirjoitti: Oskari - 28.10.2015, 08:24:12
Eikös Interstellarin mustan aukon visuaalista toteutusta kehuttu ihan T+A:ta myöten? Elokuva taitaa muuten olla vielä Netflixissä katsottavissa.
Otsikko: Vs: Musta aukko ja sen muoto, tapahtumahorisontti
Kirjoitti: Umbra - 30.10.2015, 23:14:48
Lainaus käyttäjältä: Lorelei - 22.09.2015, 20:24:31
Tämä ei nyt liity ihan aiheeseen, mutta jotenkin on ristiriitaista, että esim fyysikot (luonnontieteilijät jne) sanovat selittävänsä "kaiken" oman tieteellisen systeeminsä puitteissa. Mitään mystistä ei ole, ja sitä rataa. Silti singulariteetti "selitetään" sillä, ettei se nyt (hupsis) mahdu kontekstiin, se on siis se Juttu aukottomassa tieteenteossa joka ei mahdu kuvioon... :azn: Se on siis järjen ulottumattomissa, mitä ei tosin kai saisi sanoa ääneen.

En tiedä kuinka moni fyysikko tai tiedemies ajattelee selittävänsä "kaiken" - veikkaukseni mukaan aika harva. Ainakin kosmologia ja teoreettinen fysiikka sisältävät monia todella suuria ratkaisemattomia kysymyksiä, esimerkiksi miksi avaruus on kolmiulotteinen tai miksi alkuräjähdyksen ensi hetkillä vallitsi matala entropia. En usko kovin monen vakavasti otettavan tutkijan olevan tietämätön näistä ja monista muista aukoista tietämyksessämme.

Jos jonain päivänä keksimme suuren yhtenäisteorian, se ehkä selittää "kaiken" siinä rajoitetussa merkityksessä, että se selittää kaikki luonnon neljä perusvoimaa ja yhdistää painovoiman ja kvanttimekaniikan. Tässäkään tapauksessa ei olisi varmaa, selittäisikö teoria esimerkiksi kaikkia luonnon perusvakioita, vai jäisikö siihen jotain vapaita parametreja, jotka vain "sattuvat" olemaan sellaisia kuin ovat. Toisekseen suuri yhtenäisteoria olisi käytännössä hyödytön monimutkaisten, monista hiukkasista koostuvien systeemien selittämisessä. Esimerkiksi säätiede tai biologia tuskin hyötyisivät suuresta yhtenäisteoriasta tuon taivaallista.

Mielestäni tiede ehkä voidaan nähdä parhaana tunnettuna keinona selitysten löytämiseen. Tästä kuitenkin on pitkä matka kaiken selittämiseen.

Singulariteetti ehkä voidaan nähdä "järjen ulottumattomissa" olevana siinä mielessä, että sen käyttäytymistä on suhteellisuusteorian mukaan mahdotonta ennakoida (http://infidels.org/library/modern/quentin_smith/simplicity.html). Singulariteetissa erilaiset suureet, kuten avaruuden kaarevuus, kasvavat äärettömiksi, mikä estää fysiikan lakien soveltamisen. Esimerkiksi alkuräjähdyksen singulariteetin ei olisi ollut  pakko synnyttää maailmankaikkeutta, vaan se olisi yhtä hyvin voinut olla itse kaikki, mitä on koskaan olemassa. (Tähän väitteeseen en valitettavasti muista lähdettä.)

Ei ole kuitenkaan selvää, onko singulariteetteja lopulta lainkaan olemassa. Nykyään suositun näkemyksen mukaan singulariteetti on matemaattinen illuusio, joka kertoo suhteellisuusteorian murtumisesta riittävän äärimmäisissä olosuhteissa. Esimerkiksi säieteoreettinen kuvaus mustasta aukosta ei sisällä singulariteetteja. https://en.wikipedia.org/wiki/Fuzzball_(string_theory) 

Säieteoria on valitettavasti erittäin keskeneräisessä kehitysvaiheessa, eikä liene varmaa, tuleeko siitä koskaan valmista – vaikka se lieneekin paras nykyään tunnettu ehdokas painovoiman kvanttiteoriaksi. Eli singulariteetti näyttää olevan jotain, mitä kukaan ei tällä hetkellä yksinkertaisesti ymmärrä.
Otsikko: Vs: Musta aukko ja sen muoto, tapahtumahorisontti
Kirjoitti: mistral - 26.12.2015, 20:18:07
Onko mahdollista että tyhjiö voi syöksyä valon nopeudella horisontin läpi?

Tämä rupesi askarruttamaan, eli jos tyhjiö sisältää virtuaalihiukkasia, ainakin hetkittäin, niin voiko ne olla valon nopeudella läpäisemässä horisontin?

Jos avaruusaluksesta viskattaisiin fortunakuula juuri oikeaan suuntaan, niin että se päätyisi Sagittariuksen mustaan aukkoon, ja se saisi vapaasti pudota törmäämättä mihinkään esteeseen, saavuttaisi se ilmeisesti valon nopeuden. Perustelen tätä sillä että jos aika käännettäisiin raksuttamaan taaksepäin, kuula osuisi pitkän ajan kuluttua astronautin käteen eikä mikään luonnonlaki olisi muuttunut. (kuulalla olisi horisontissa riittävä pakonopeus että pääsisi raketin luo)

Tämä on aika selvää mutta kuinka tyhjiö käyttäytyisi? Jos raketin vieressä olisi kuutiometri tyhjiötä joka matkustaisi kuulan rinnalla Sagittariukseen ja menisi horisontin läpi samalla hetkellä niin voisiko Hawkingin säteily toteutua siinä? Ideahan on se että tyhjiössä syntyy virtuaalihiukkaspari joka lyhyen ajan päästä kumoutuu. Jos kuitenkin toisen hiukkasen, joka on ylittänyt horisontin, nopeus "ylittää valon nopeuden"niin se ei voi palata takaisin toisen hiukkasen luo (joka on matkalla kohtisuoraan ylöspäin) ja ne jäisivät erilleen. Näin syntyisi Hawkingin säteily. (ehkä)

Tämä siis vain mietiskelyä, en ole saanut tähän päivään mennessä tolkkua asiaan. Sellainen malli jossa tyhjiö olisi paikallaan horisontin tuntumassa ei selitä ainakaan minulle säteilyä.
Otsikko: Vs: Musta aukko ja sen muoto, tapahtumahorisontti
Kirjoitti: jussi_k_kojootti - 04.01.2016, 23:04:19
Kaikissa Hawkingin säteilyn syntymekanismeissa toinen (virtuaalisesta) hiukkasparista syntyy valmiiksi tapahtumahorisontin sisäpuolelle.  Toinen on -- jo syntyessään, ja vaikka vain hetken -- tapahtumahorisontin ulkopuolella, mikä näyttää ulkopuolisen havaitsijan mielestä siltä että aukko säteili hiukkasen -- semminkin kun aukon massa vähenee, taaskin virtuaaliparin syntymekanismista riippumatta, juuri säteillyn hiukkasen massan verran.

Virtuaalipari voi syntyä tapahtumahorisontin reunalle useammankin järkeilyn perusteella, kaikille on kuitenkin yhteistä se että hiukkaspari kuvitellaan syntymään aivan tapahtumahorisontin reunalle, niin että jo alkutilanteessa toinen hiukkasista on aukossa.
Otsikko: Vs: Musta aukko ja sen muoto, tapahtumahorisontti
Kirjoitti: spuge - 05.01.2016, 11:17:39
Lainaus käyttäjältä: Umbra - 08.09.2015, 22:46:05
...kaukana olevan havaitsijan näkökulmasta mustaan aukkoon putoava astronautti ei koskaan läpäise tapahtumahorisonttia, vaan jäätyy sen pintaan litteäksi "haamuksi". ...

Jos näin on, niin eikö meidän silloin pitäisi pystyä havaitsemaan mustan aukon tapahtumahorisontin pinnalta kaikki sen aikojen saatossa imaisema aine tuollaisena haamuna?
Otsikko: Vs: Musta aukko ja sen muoto, tapahtumahorisontti
Kirjoitti: mistral - 05.01.2016, 11:42:30
Lainaus käyttäjältä: spuge - 05.01.2016, 11:17:39
Jos näin on, niin eikö meidän silloin pitäisi pystyä havaitsemaan mustan aukon tapahtumahorisontin pinnalta kaikki sen aikojen saatossa imaisema aine tuollaisena haamuna?

Kun materia putoaa horisonttiin, kasvaa aukon massa ja horisontti siirtyy ulemmas. Siksi vanhemmat kerrostumat jäisi uudempien alle eikä niitä voisi nähdä. Myös toinen syy estää näkemästä, horisontista nouseva säteily on niin pitkäaaltoista ettei siitä saisi valotettua kuvia. Tai en ainakaan tiedä, voiko esim meidän aurinkokunnan pituisen aallon valottaa, onko sellaisia laitteita olemassa.
Otsikko: Vs: Musta aukko ja sen muoto, tapahtumahorisontti
Kirjoitti: spuge - 05.01.2016, 15:27:22
No eipä sitten astronauttikaan jäisi siihen haamuksi. Eiköhän siihen päde ihan samat lainalaisuudet ;-)
Otsikko: Vs: Musta aukko ja sen muoto, tapahtumahorisontti
Kirjoitti: jussi_k_kojootti - 04.03.2016, 23:08:44
Lainaus käyttäjältä: mistral - 05.01.2016, 11:42:30
Kun materia putoaa horisonttiin, kasvaa aukon massa ja horisontti siirtyy ulemmas. Siksi vanhemmat kerrostumat jäisi uudempien alle eikä niitä voisi nähdä.

Tuollaiset "sipulit" ovat tavallisia tähtiä.  Mustissa aukoissa massa päätyy singulariteettiin.  Kerrostumia tai kuoria ei ole --itseasiassa mustia aukkoja tarkastellaan tyypillisesti 2-ulotteisina objekteina. Jos tämä tuntuu oudolta, niin pidä mielessä että tapahtumahorisontin sisäpuolella aika-avaruus on "äärimmäisen kaareutunut" -- tai ainakin niin kaareutunut, että se mikä astronautin mielestä hetkeä ennen tapahtumahorisontin ylitystä oli "sisäänpäin, kohti singulariteettia" on horisontin sisäpuolella "tulevaisuuteen, kohti singulariteettia".  Niin ikään kaikki maailmanviivat horisontin sisäpuolella johtavat singulariteettiin, ja kaikki yritykset käyttää (minkäänlaista) energiaa johtavat sinne vielä nopeammin.  Tämä on tarkoitus mieltää noiden kerrostumien vinkkelistä -- mikä niitä pitäisi paikallaan?

Edit:  vetäisin pöljäilyn yli -- visualisaatioissa avaruus pakataan 2:een, mutta itse matikka on 4-ulotteista (aika ja pallokoordinaatit).
Otsikko: Vs: Musta aukko ja sen muoto, tapahtumahorisontti
Kirjoitti: jussi_k_kojootti - 05.03.2016, 10:03:34
Lainaus käyttäjältä: Joksa - 05.03.2016, 09:51:34
Tuo ajatusrakennelma tuntuu kovin kummalliselta ja epäloogiselta, voisitko avata perusteluja hieman enemmän?

Mustaan aukkoon putoavan tulevaisuudessa (tai, kaikissa tulevaisuuksissa) odottaa singulariteetti.  Ei kai se kovin epäloogista ole?

Lainaa
Tapahtumahorisontin jälkeen putoajan aikakomponentti muuttuu negatiiviseksi,

?

Lainaa
Loogisesti ajatellen putoajan suunnan pitäisi kulkea kohti menneisyyttä ja singulariteettia.

?

Lainaa
Muulle ajattelulle pitäisi olla erinomaisen hyvät perustelut.

Ei siltä vaikuta :-)
Otsikko: Vs: Musta aukko ja sen muoto, tapahtumahorisontti
Kirjoitti: mistral - 05.03.2016, 12:19:56
Lainaus käyttäjältä: ketarax - 04.03.2016, 23:08:44
  Tämä on tarkoitus mieltää noiden kerrostumien vinkkelistä -- mikä niitä pitäisi paikallaan?

Ei mikään, siis jos aika ei ole pysähtynyt. Gravitaatiokaivon ulkopuolelta katsottuna horisontin aika on pysähtynyt ja meidän kannalta siellä olisi sipulin kerrostumia, tosin teoriat ei ehkä ennusta mitään jos ne menevät äärettömään jo horisontin pinnassa.
Otsikko: Vs: Musta aukko ja sen muoto, tapahtumahorisontti
Kirjoitti: mistral - 06.03.2016, 20:24:20
Joo, ulkopuolisen havaitsijan kannalta tilanne näyttää "jäätyneeltä", edes vapaata pudotusta ei ole koska nopeus on nolla. Mutta tosiaan g-vaikutus sieltä tulee täydellä teholla ja olettaisin ettei gravitaatioaallot edes punasiirry noustessaan syöveristä.

Otsikko: Vs: Musta aukko ja sen muoto, tapahtumahorisontti
Kirjoitti: jussi_k_kojootti - 07.03.2016, 11:08:14
Lainaus käyttäjältä: Joksa - 05.03.2016, 11:01:32
Voi se olla. Kuten mainitsin, tapahtuman aikataulu on oleellinen: jos asia ei tapahdu maailmankaikkeutemme elinkaaren aikana niin ainut käytännön tulkinta asialle on että sitä ei tapahdu. Ilmaisu "kaikissa tulevaisuuksissa" on virheellinen ilmaisu kun ei edes oman kosmoksemme tulevaisuudessa ennätä tapahtumaan.

Aikakomponen osalta ks. esim. Kari Enqvist, Johdatus suhteellisuusteoriaan, sivu 83: (metriikan aikakomponenttia dt^2 ilmaiseva funktio) F^2(r) = 1 - rs/r, jossa rs = 2GM/c^2 on tapahtumahorisontin säde, r = tarkasteltava etäisyys singulariteetista. Jos r < rs niin negatiiviseksihan komponentti menee.

Vaikuttaako jo...

Ei vaikuta.  Samassa kirjassa on varmasti jossain kohtaa selvitystä käsitteestä "samanaikaisuuden suhteellisuus".  "Kaikki tulevaisuudet" ei tarkoita mitään sen kummempaa kuin "kaikki maailmanviivat", ja tapahtumahorisontin sisällä kaikki maailmanviivat johtavat singulariteettiin.
Otsikko: Vs: Musta aukko ja sen muoto, tapahtumahorisontti
Kirjoitti: jussi_k_kojootti - 07.03.2016, 11:14:44
Lainaus käyttäjältä: mistral - 05.03.2016, 12:19:56
Ei mikään, siis jos aika ei ole pysähtynyt. Gravitaatiokaivon ulkopuolelta katsottuna horisontin aika on pysähtynyt ja meidän kannalta siellä olisi sipulin kerrostumia, tosin teoriat ei ehkä ennusta mitään jos ne menevät äärettömään jo horisontin pinnassa.

Teoriat ei tällä hetkellä ennusta mitään keinoa rakentaa sipuleita sen jälkeen kun gravitaatio peittoaa degeneroituneen neutronikaasun paineen.  Tämän vuoksi teoriat sanovat, että tiettyä massaa suuremmat neutronitähdet romahtaa mustiksi aukoiksi, joissa tähden aine päätyy singulariteettiin. 

Huom, en nyt puhu siitä mikä on mahdollista tai mahdotonta, vaan siitä miten asia tunnettujen teorioiden valoissa tulkitaan/käsitellään.

Teoriat ei myöskään "mene äärettömään" horisontin pinnalla.  Omituista, jos lähteissänne ei ole näitä asioita selvitetty.
Otsikko: Vs: Musta aukko ja sen muoto, tapahtumahorisontti
Kirjoitti: jussi_k_kojootti - 07.03.2016, 11:20:00
Lainaus käyttäjältä: Joksa - 06.03.2016, 17:40:30
ihan plankin epookkiin eli puhtaaksi singulariteetiksi saakka. Kvarkkitähden epäillään olevan olemassa,  tuntuisi että ainakin siihen saakka aine kompressaantuisi. Tapahtuisiko jotakin käänteistä inflaatiota (eli deflaatiota?) jossa häviäisi kvarkit ja gluonitkin jää selvitettäväksi, ehkä uusi gravitaatiotähtitiede joskus kertoo senkin. Mikäli taas aine jäisi kvarkki-gluonimassa tasolle niin silloinhan ainetta kerrostuisi aukkoon.

Kvarkkitähti on erittäin hypoteettinen objekti, en sanoisi että niitä ainakaan yleisesti "epäillään olevan olemassa".  Mutta kyllä, JOS kvarkeilla on degeneroitunut faasi, niin silloin tapahtumahorisontin sisällä voisi olla sipuli.
Otsikko: Vs: Musta aukko ja sen muoto, tapahtumahorisontti
Kirjoitti: jussi_k_kojootti - 07.03.2016, 11:29:44
Lainaus käyttäjältä: Joksa - 06.03.2016, 17:40:30
Ajan hidastuminen tapahtumahorisontiin putoavan aineen osalta koskenee ko ainetta itseään, aineen toiminnot hidastuvat ja se muuttuu tapahtumahorisontissa täysin passiiviseksi.

Puutun vielä tähänkin -- nimenomaan noin ei käy, putoavan aineen värähtelyt ei hidastu yhtään sen enempää kuin aukkoon pudotetun astronautin kello (tai sydän).  Joka siis ei astronautin siihen katsoessa hidastu ollenkaan.  _Ulkopuolisen_ havaitsijan mielestä hidastuisi kyllä, mutta se on sitä suhteellisuutta.
Otsikko: Vs: Musta aukko ja sen muoto, tapahtumahorisontti
Kirjoitti: jussi_k_kojootti - 07.03.2016, 13:04:34
Lainaus käyttäjältä: Joksa - 07.03.2016, 12:18:51
Juuri niin, ulkopuolisen eli muun kosmoksen kannalta (ajassa) putoava aine hidastuu, ja se on asiassa oleellista.

Eikö oleellisempaa ole se, mitä aineelle oikeasti tapahtuu kuin se mitä sille näyttää tapahtuvan?  Vertaa relativistiseen avaruusmatkaan.  Maahan jääneiden sukulaisten (arkijärjen) mielestä astronautti ehtii kuolla taittaessaan 1000 valovuoden matkan, vaikka astronautti itse tietää päässeensä perille.  Sukulaisetkin tietävät, jos hylkäävät tarkastelussa arkijärjen ja hyväksyvät suhteellisuuden.
Otsikko: Vs: Musta aukko ja sen muoto, tapahtumahorisontti
Kirjoitti: mistral - 07.03.2016, 13:24:07
Lainaus käyttäjältä: ketarax - 07.03.2016, 11:14:44
Teoriat ei tällä hetkellä ennusta mitään keinoa rakentaa sipuleita sen jälkeen kun gravitaatio peittoaa degeneroituneen neutronikaasun paineen.  Tämän vuoksi teoriat sanovat, että tiettyä massaa suuremmat neutronitähdet romahtaa mustiksi aukoiksi, joissa tähden aine päätyy singulariteettiin. 

Siis näkökulma on nyt vapaasta avaruudesta käsin. Näin materia "jäätyy" horisontin pintaan, tämä lienee peruskauraa. Mutta nyt tulee sipulirakenne ja se syntyy kun miljoonien vuosien kuluessa tavaraa kertyy vanhan horisontin päälle jolloin jäätynyt kerros paksunee ja Schwartzildin säde kasvaa. Näin voidaan olettaa että pinnan alla olisi kerrosrakenne, ikäänkuin Loimaan savipatja joka on syntynyt hitaasti.

Mutta paikan päällä havaittuna ei tietenkään mitään kerrostumia näy, päinvastoin kaikki tavara syöksyy n. c nopeudella singulariteettia kohti.
Lainaa
Teoriat ei myöskään "mene äärettömään" horisontin pinnalla.  Omituista, jos lähteissänne ei ole näitä asioita selvitetty.

Tämä on vaan maalaisjärjellä tehty intuitiivinen johtopäätös, eli viivan alle tulee ääretön mikä pysäyttää ajan nopeuden nollaan.
Otsikko: Vs: Musta aukko ja sen muoto, tapahtumahorisontti
Kirjoitti: jussi_k_kojootti - 07.03.2016, 14:11:34
Lainaus käyttäjältä: Joksa - 07.03.2016, 13:30:16
Menemättä avaruusmatkailun puolelle voisitko kertoa mitä ajattelet aineelle tapahtuvan, aineen itsensä näkökulmasta, kun sen aika on hidastunut niin että se = 0?

Ei aineen aika hidastu.  Kaikki kellot käyvät omassa koordinaatistossaan (siis "aineen itsensä näkökulmasta") ihan tasaisesti.
Otsikko: Vs: Musta aukko ja sen muoto, tapahtumahorisontti
Kirjoitti: jussi_k_kojootti - 07.03.2016, 14:16:57
Lainaus käyttäjältä: mistral - 07.03.2016, 13:24:07
Näin voidaan olettaa että pinnan alla olisi kerrosrakenne, ikäänkuin Loimaan savipatja joka on syntynyt hitaasti.

Mutta kun näin ei voida olettaa, jos pitäydytään suhteellisuusteoriassa -- joka puolestaan _selittää_ miksi näin ei voida olettaa.

Lainaa
Tämä on vaan maalaisjärjellä tehty intuitiivinen johtopäätös, eli viivan alle tulee ääretön mikä pysäyttää ajan nopeuden nollaan.

Maalaisjärjen ja suhteellisuusteorian yhdistäminen on turhaa, tarpeetonta, ja toivotonta.  Maalaisjärjen vinkkelistä relativistiset ilmiöt on aivan aidosti paradoksaalisia.  Suhteellisuusteoria selittää miksi näin on.

Ongelma tuntuu olevan se, että hyväksytte suhteellisuusteorian ja sen ennustukset ajattelunne _lähtökohdaksi_ (eli esim. sanotte "tarkastellaanpa mustaa aukkoa" -- joka on suhteellisuusteorian ennustus), mutta hylkäätte sen heti seuraavassa askeleessa.


Otsikko: Vs: Musta aukko ja sen muoto, tapahtumahorisontti
Kirjoitti: jussi_k_kojootti - 07.03.2016, 15:49:13
Lainaus käyttäjältä: mistral - 06.03.2016, 20:24:20
Mutta tosiaan g-vaikutus sieltä tulee täydellä teholla ja olettaisin ettei gravitaatioaallot edes punasiirry noustessaan syöveristä.

Tämä kuulostaa gravitoneilta, jotka voivat olla osa tulevaa kvanttigravitaation teoriaa, tai sitten olla olematta.  Niillä on konseptuaalinen sijansa fysiikassa jo nyt, kun tarkastellaan aika-avaruuden kaarevuuden *muutoksia*, siis painovoima-aaltoja, mutta esimerkiksi Maan ja satelliittien välisessä vuorovaikutuksessa ei tarvita mitään painovoimasäteilyä, Maan aikaansaama aika-avaruuden kaareutuminen riittää.

En siis -- ainakaan toistaiseksi -- sanoisi, että mustasta aukosta "tulee" jotain "g-vaikutusta".  "G-vaikutus", avaruuden kaareutuminen, "vain on".
Otsikko: Vs: Musta aukko ja sen muoto, tapahtumahorisontti
Kirjoitti: jussi_k_kojootti - 07.03.2016, 16:44:51
Lainaus käyttäjältä: Joksa - 07.03.2016, 15:20:58
Milestäni hylkäämisestä ei ole kyse.

No siis, esimerkiksi:

Teoria sanoo karkeasti ottaen:  jos neutronitähden massa kasvaa riittävän suureksi (raskain havaittu n 2 x Msol, teoreettinen maksimi ehkä 3 x Msol), aika-avaruuden kaareutuminen kasvaa niin suureksi ettei mikään tunnettu ilmiö riittää estämään materian romahtamista kasaan, yhteen pisteeseen (tai mahdollisesti infinitesimaalisen pieneen tilavuuteen), singulariteettiin.

Te sanotte, että sipuli.

Jotain on hylätty, tai ainakin lisätty (se sipuli!).

Enkä muuten tarkoita, ettenkö jopa suuresti arvostaisi pyrkimyksiänne ymmärtää suhteellisusteoriaa "maalaisjärjellä".  Kannan vain korteni kekoon, kun luulen näkeväni epäjohdonmukaisuuksia.

Otsikko: Vs: Musta aukko ja sen muoto, tapahtumahorisontti
Kirjoitti: Eusa - 07.03.2016, 16:49:39
Lainaus käyttäjältä: mistral - 06.03.2016, 20:24:20
Joo, ulkopuolisen havaitsijan kannalta tilanne näyttää "jäätyneeltä", edes vapaata pudotusta ei ole koska nopeus on nolla. Mutta tosiaan g-vaikutus sieltä tulee täydellä teholla ja olettaisin ettei gravitaatioaallot edes punasiirry noustessaan syöveristä.
Kvanttimekaanisesti tapahtumahorisontin kuuluisi kai olla aivan sileä niin, ettei minkään lomittuneen vuorovaikutuksen tarvitse hajottaa amplitudeja horisontin eri puolille, kyseessä jonkinlainen palomuuriongelma.

Horisontin periaatteen mukaan mikään signaali ei tule ulos, vaan gravitaation kenttäkaarevuus myös tulisi siletä tasaiseen arvoon ennen tapahtumahorisonttia. Sileysvaatimuksia näyttäisi siten esiintyvän. Niiden ohittamiseksi on esitetty ratkaisuja.

Joka tapauksessa gravitoneja aukosta ei voi tulla ulos horisontin takaa. Kuten tunnettua, gravitonit tai gravitaatioaallot välittävät gravitaatiokentän muutosvaikutuksia, ei itse gravitaatiota. Jos tapahtumahorisontti on sileä, eikä rakenteita kulje sen läpi, musta aukko toimisi ongelma-asetteluitta.

Korjannee kellä on parempaa tietoa.
Otsikko: Vs: Musta aukko ja sen muoto, tapahtumahorisontti
Kirjoitti: mistral - 07.03.2016, 17:31:37
Lainaus käyttäjältä: ketarax - 07.03.2016, 16:44:51
Teoria sanoo karkeasti ottaen:  jos neutronitähden massa kasvaa riittävän suureksi (raskain havaittu n 2 x Msol, teoreettinen maksimi ehkä 3 x Msol), aika-avaruuden kaareutuminen kasvaa niin suureksi ettei mikään tunnettu ilmiö riittää estämään materian romahtamista kasaan, yhteen pisteeseen (tai mahdollisesti infinitesimaalisen pieneen tilavuuteen), singulariteettiin.

Te sanotte, että sipuli.

Otan havainnollistustaulukon jossa kappale lähestyy horisonttia koko ajan vaikka 99%(tämä ei vastaa todellista tilannetta mutta yksinkertaistaa selitystä) valon nopeudella:

a) 0,9x hitaammassa ajassa (aikavyöhykkeessä) kuin me--------->nopeus meidän mielestä 270 000km/s
b) 0,5x ------------->150 000km/s
c) 0,1x-------------->30 000km/s
d) 0,0001x---------->30km/s
e) 0,0x-------------->0km/s

Kohdassa e) näyttäisi kappale pysähtyneen mikä aikavyöhyke on horisontin pinnassa.


Tässä ei myöskään olla huomioitu pituuskontraktiota mikä myös vaikuttaa relatevistisilla nopeuksilla kappaleen etenemissuunnassa. Jos punasiirtymä ei estäisi näkemästä nopeasti kohti mustaa aukkoa menevää kappaletta, me näkisimme sen hyvin lähellä olevana, tämäkin vielä sekoittaa asioita.
Otsikko: Vs: Musta aukko ja sen muoto, tapahtumahorisontti
Kirjoitti: mistral - 07.03.2016, 17:56:54
Lainaus käyttäjältä: Eusa - 07.03.2016, 16:49:39
Horisontin periaatteen mukaan mikään signaali ei tule ulos, vaan gravitaation kenttäkaarevuus myös tulisi siletä tasaiseen arvoon ennen tapahtumahorisonttia. Sileysvaatimuksia näyttäisi siten esiintyvän. Niiden ohittamiseksi on esitetty ratkaisuja.

Jos horisontissa siis olisi kaikki massa, sanotaan vaikka 1cm paksuisessa (tai 1mikrometrin) kerroksessa, kuinka se silloin voisi olla vaikka Linnunradan keskusaukkona? Eihän massa riittäisi.
Otsikko: Vs: Musta aukko ja sen muoto, tapahtumahorisontti
Kirjoitti: mistral - 07.03.2016, 18:22:45
Lainaus käyttäjältä: Joksa - 07.03.2016, 17:15:19
Jos singulariteetteja on vain yhtä laatua (hypoteesi) niin se tarkoittaisi että mustan aukon kama ei pressaantuisi suoraan sellaiseksi vaan aine jäisi joksikin aikaa kvarkkitasolle. Aukon kasvaessa ja gravitaatiokuopan syventyessä ainetta alkaisi puristua singularitettitasolle asti, lopun massan säilyttäessä g-vuorovaikutuksen ja esiintyen bb-vaiheiston eri epookkien muodoissa, sipulina, kuten aine massiviisissa tähdissä fuusion edetessä. Tästä seuraisi että mustan aukon gravitaatiovuorovaikutus ei voisi kasvaa äärettömän kokoiseksi kuten toisessa vaihtoehdossa.

Olisihan se hurjaa jos gravitaatio raukeaisi tyhjiin kun materia puristetaan tarpeeksi pieneksi paketiksi. Toisaalta jos gravitaatio häviäisi, silloin ytimen painekin putoaisi ja sinne palautuisi gravitaatio. Tästä seuraisi tasapainotila jossa olisi joku yläraja.
Otsikko: Vs: Musta aukko ja sen muoto, tapahtumahorisontti
Kirjoitti: Eusa - 07.03.2016, 18:27:18
Lainaus käyttäjältä: mistral - 07.03.2016, 17:56:54
Jos horisontissa siis olisi kaikki massa, sanotaan vaikka 1cm paksuisessa (tai 1mikrometrin) kerroksessa, kuinka se silloin voisi olla vaikka Linnunradan keskusaukkona? Eihän massa riittäisi.
Looginen johtopäätös onkin, ettei mustaa aukkoa muostukaan, vaan avaruus miniarisoituu, mittakaava "tardistuu". Hypoteesille pitäisi vielä leipoa suhteellisuusteorioiden mukainen energiaympäristön metrinen (perturbatiivinen) ratkaisu...
Otsikko: Vs: Musta aukko ja sen muoto, tapahtumahorisontti
Kirjoitti: Eusa - 08.03.2016, 05:53:51
Ursan blogisti Syksy Räsänen viittaa kommenttiosiossaan muodostumattoman mustan aukon mahdollisuuteen:

"On kyllä esimerkiksi ehdotettu, että kvanttiefektien takia mustia aukkoja ei koskaan synny, aine vain lähestyy romahtamista yhä hitaammin ja hitaammin. Kaukaa katsottuna kappale näyttäisi tällöin aivan samalta kuin musta aukko, mutta törmäykset ja niiden jälkeinen paikoilleen asettuminen olisivat erilaisia."

https://www.ursa.fi/blogi/kosmokseen-kirjoitettua/arvoituksellista-piristysta/#comment-4430

Tapahtumahorisontti olisi tuossa tapauksessa näennäinen, virtuaalinen ja/tai asymptoottinen.
Otsikko: Vs: Musta aukko ja sen muoto, tapahtumahorisontti
Kirjoitti: mistral - 08.03.2016, 21:35:59
Lainaus käyttäjältä: Joksa - 08.03.2016, 20:41:20
Ajattelin kyllä että maksimin ylittävä massa painuisi 'pakettiin' sitä mukaa kun uutta massaa kertyisi, välitilassa oleva massa olisi vakio ja prosessi siten rauhallinen. Aukon maksimikoko on arvioitu kertymäkiekon stabiliisuuteen perustuen mutta se ei rajanne törmäyskasvua.

http://www.avaruus.fi/uutiset/tahdet-sumut-ja-galaksit/mustien-aukkojen-koolle-loytyi-ylaraja.html

Tämä voi mahdollisesti toimia jos aika ei pysähdykään aukossa. Jos pysähtyy, maailmankaikkeudessa ei taida olla yhtäkään singulariteettia.

Täytyy myöntää että aivot tahtoo nyrjähtää tässä pähkäilyssä.
Otsikko: Vs: Musta aukko ja sen muoto, tapahtumahorisontti
Kirjoitti: jussi_k_kojootti - 10.03.2016, 18:22:30
Lainaus käyttäjältä: Joksa - 09.03.2016, 09:38:20
Kumpi on oikea?

Vaihtoehto yksi.

Lainaus käyttäjältä: Joksa - 10.03.2016, 15:50:38
Tämä kai tukee selkeästi keskustelussamme esiin nousseita sipulimalleja?

Internetistä löytyy "tukea" ihan mille vain.

Pointti(ni) ei nyt kuitenkaan ole niissä sipuleissa sinällään, vaan siinä miten ne perusteltiin.   Ja perusteluhan meni niin, että koska horisonttiin vajoava astronautti (gravitaatio-)aikadilatoituu kaukaisen havaitsijan mielestä, hänestä jää "haamu" horisontin pinnalle (tässä kohtaa kuulen myös kaikuja säieteorian holografisesta periaatteesta, liekö vaikuttanut Mistralin alkuperäiseen näkemykseen?);  ja aukon massan kasvaessa horisontin säde kasvaa, jolloin aiemman horisontin "haamut" jäisivät säteen sisäpuolelle sipulinkuoreksi.

En löytänyt pikaisesti silmäillen, lähinnä lainatusta kohdasta ja sen ympäriltä,  tukea _tuolle_ (tai tuonkaltaiselle) näkemykselle.  En myöskään jaa näkemystäsi:

Lainaus käyttäjältä: Joksa - 10.03.2016, 15:50:38
Eli singulariteettiin pakkautuvan aineen gravitaatio todellakin katoaa sen vuoksi että niiden energiat ovat erimerkkisiä!

En tiedä mitä negatiivinen (gravitaatio-)energia tarkoittaa yleisessä suhteellisuusteoriassa, mutta newtonilaisittain kyse on kokonaisenergian säilymisestä.  Lyhyesti:  kaksi toistaan lähestyvää kappaletta vetävät toisiaan puoleensa <=> lähestymisvauhti kasvaa <=> kineettinen energia kasvaa.  Toisaalta samalla gravitaatiokentän voimakkuus, ja siten gravitaatioenergia, kasvaa, koska kappaleiden välinen etäisyys pienenee.  Molemmat ei voi kasvaa, koska systeemin kokonaisenergia ei voi kasvaa <=> gravitaatioenergian on oltava miinusmerkkistä. 

Jordanin sinällään suuri oivallus on se, että jos maailmankaikkeuden kokonaisenergia on 0, niin 0-energia voi olla maailmankaikkeus :-)

No mutta siis:  on olemassa näitä hiukkasia, bosoneita, joita ei Paulin kieltosääntö pitele.  Niitä voi olla vaikka kuinka paljon ihan yhdessä ainokaisessa pisteessä.  Ja tämmöinen bosonipiste kaareuttaa massaenergiallaan aika-avaruutta ihan tavallisesti <=> ei gravitaatio minnekään katoa.

Sivumennen sanoen yksi ehdotettu ratkaisu mustien aukkojen singulariteetin dilemmaan on se, että jokin tuntematon prosessi muuttaa aineen (eli fermionit) bosoneiksi ennen singulariteettia.
Otsikko: Vs: Musta aukko ja sen muoto, tapahtumahorisontti
Kirjoitti: mistral - 10.03.2016, 23:45:41
Lainaus käyttäjältä: Joksa - 09.03.2016, 09:38:20

1) Lähetin-vastaanotin parin kelloja (aikaa) hidastetaan (aukoon putoaja). Vastaaonotin ei havaitse lähettimen hidastumista, koska sen kello on hidastunut myös, mutta hidastamaton ulkopuolinen vastaanotin havaitsee. Vrt. http://www.tiede.fi/artikkeli/jutut/artikkelit/syoksytaan_mustaan_aukkoon "Kun ajan tajuamiseen kuluu aikaa, hidas aika vaikuttaa normaalilta".

Tulkinta: aikamuutos vaikuttaa putoajaan, eli aineen värähtelyt hidastuvat oikeasti, mutta niin totaalisesti että putoaja itse ei sitä voi sitä itse mitenkään todeta aistiensa tms vastaanottimiensa kellojen myös hidastuttua. Tästä seuraa mm että kun/jos putoaja ja ulkopuolinen joskus kohtaisivat, heillä on aito aika/ikäero.   

Voiskohan ajan nopeutta testata mittaamalla valon nopeus? Eli hitaammalla aikavyöhykkeellä jos valo menisikin reippaasti yli 300 000km/s. Tai jos edelleen menisi samaa nopeutta kuin maassa, olisi se hidastunut. Toisaalta jos atomit suurenee ajan hidastuessa niin metrimittakin suurenee ja sillä mitattuna valon nopeus olisi vakio. Mutta atomien suureneminen kasvattaisi massaa ja sehän ei taas voi olla totta.

Sori muokkasin päinvastaiseksi.
Otsikko: Vs: Musta aukko ja sen muoto, tapahtumahorisontti
Kirjoitti: jussi_k_kojootti - 10.03.2016, 23:47:24
Lainaus käyttäjältä: Joksa - 10.03.2016, 19:40:59
Piste "maalaisjärjelle":?)

Jos niin sanot, vaikka mun maalaisjärkeen ei mahdu se että kaksoset voi olla saman ajan kuluttua eri ikäisiä :-)
Otsikko: Vs: Musta aukko ja sen muoto, tapahtumahorisontti
Kirjoitti: Eusa - 11.03.2016, 11:26:36
Lainaus käyttäjältä: mistral - 10.03.2016, 23:45:41
Voiskohan ajan nopeutta testata mittaamalla valon nopeus? Eli hitaammalla aikavyöhykkeellä jos valo menisikin reippaasti yli 300 000km/s. Tai jos edelleen menisi samaa nopeutta kuin maassa, olisi se hidastunut. Toisaalta jos atomit suurenee ajan hidastuessa niin metrimittakin suurenee ja sillä mitattuna valon nopeus olisi vakio. Mutta atomien suureneminen kasvattaisi massaa ja sehän ei taas voi olla totta.
Ajatellaanpa vaikka valon nopeuden mittaamista lasisen vesialtaan sisältä, valonlähde ja varjostin altaan ulkopuolella. Jos mittari on hieman lähempänä varjostinta, näkyy valo varjostimella ennemmin kuin valonlähteessä johtuen väliaineesta. Tuosta saa "maalaisjärkeä" hiukkastason tunneloitumishavaintoihin. Gravitaatiokaivot vain ovat siitä "kivoja", että yleensä pallosymmetrisinä vähentävät tuollaisen astronomisen efektin mahdollisuutta/hämäävyyttä.

Puhut mitä ilmeisimmin tuossa ns. itseisnopeudesta - se on kyllä valolle aina ääretön. Hitaamman aikavyöhykkeen ulkopuolelta mitatut valon nopeudet ovat siis aina vähemmän kuin c. Samoin ulkopuolelta tarkastellut kappaleiden mitat päinvastoin pienenevät gravitaatiokaivossa. Valolla mitaten esim. kiertorata "mustan aukon" lähellä on pidempi ja kaukaa havaittava orbitaalinopeus hitaampi kuin pelkän Newtonin opin perusteella olisi pääteltävissä. Noin olen ymmärtänyt.
Otsikko: Vs: Musta aukko ja sen muoto, tapahtumahorisontti
Kirjoitti: jussi_k_kojootti - 11.03.2016, 12:47:46
Lainaus käyttäjältä: Joksa - 11.03.2016, 07:59:02
Joo-o, eri aikahan siinä kulua pitää... toinenkin piste maalaisjärjelle;?)

Me tullaan selvästikin eri landeilta.
Otsikko: Vs: Musta aukko ja sen muoto, tapahtumahorisontti
Kirjoitti: jussi_k_kojootti - 11.03.2016, 15:43:25
Laskeudutaan singulariteettiin (http://jila.colorado.edu/~ajsh/insidebh/schw.html)
Otsikko: Vs: Musta aukko ja sen muoto, tapahtumahorisontti
Kirjoitti: mistral - 11.03.2016, 16:33:35
Lainaus käyttäjältä: Eusa - 11.03.2016, 11:26:36
Ajatellaanpa vaikka valon nopeuden mittaamista lasisen vesialtaan sisältä, valonlähde ja varjostin altaan ulkopuolella. Jos mittari on hieman lähempänä varjostinta, näkyy valo varjostimella ennemmin kuin valonlähteessä johtuen väliaineesta. Tuosta saa "maalaisjärkeä" hiukkastason tunneloitumishavaintoihin.

Tarkoitatko että akvaarion toisella laidalla on valonlähde ja toisella varjostin ja säde menee veden läpi? Tässä tapauksessa valo ehtii suunnilleen yhtäaikaa sekä mittarille että varjostimelle mutta viiveellä varjostimelta heijastuksena mittarille, tämä siis arkijärjellisesti. Mitä vesi tuossa haittaa?

Lainaa
Puhut mitä ilmeisimmin tuossa ns. itseisnopeudesta - se on kyllä valolle aina ääretön.

Olen ymmärtänyt ettei valon itseisnopeus ylitä c:tä, juju on siinä että pituuskontraktio typistää avaruuden pituuden nollaan vauhdin suunnassa. Näin itseisnopeus saa olla rajallinen, siihen ei tarvitse koskea.
[/quote]
Otsikko: Vs: Musta aukko ja sen muoto, tapahtumahorisontti
Kirjoitti: Eusa - 11.03.2016, 16:54:20
Tarkoitin, että valonsäde kulkee lähteestä varjostimelle akvaarion ulkopuolella ja lähteestä sekä varjostimesta akvaariossa olevaan ilmaisimeen. Jos ilmaisin on riittävästi lähempänä varjostinta kuin lähdettä, näyttää siltä, että valo siirtyy varjostimelta valonlähteeseen.

Itseisnopeus on valolla mitattu etäisyys jaettuna kohteen kokemalla ajalla siirryyttyään ko etäisyyden. Valolla tulee aina nollalla jako. Ks. Wikistä "itseisnopeus".
Otsikko: Vs: Musta aukko ja sen muoto, tapahtumahorisontti
Kirjoitti: mistral - 11.03.2016, 19:54:26
Lainaus käyttäjältä: Eusa - 11.03.2016, 16:54:20

Itseisnopeus on valolla mitattu etäisyys jaettuna kohteen kokemalla ajalla siirryyttyään ko etäisyyden. Valolla tulee aina nollalla jako. Ks. Wikistä "itseisnopeus".

Täällä S. Räsänen kommentoi itseisnopeudesta: https://www.ursa.fi/blogi/kosmokseen-kirjoitettua/kaksi-tarinaa-ajasta/

   Eusa kirjoitti:   
4.9.2014 19:10   

Aikaan liittyy nopeus. Olisin kiinnostunut viimeisimmästä käsityksestä, onko suurinta mahdollista itseisnopeutta olemassa. Itseisnopeudella tarkoitan kappaleen kokemaa nopeutta, jolla yleisen suhteellisuusteorian mukaan ei ole ylärajaa. Voisiko jokin tekninen raja kuitenkin tulla vastaan?

Millaisia ovat suurimmat havaitut (epäsuorasti lasketut) itseisnopeudet? Onko havaittu jotain taustaeetteriä, jonka suhteen tuo nopeus voisi eksaktisti määrittyä? Ohitettavien kohteiden keskimääräinen gravitaatiokenttä? Vrt. itseiskiihtyvyys, joka vertautuu inertiaalikoordinaatistoon l. vapaan putamisen liiketilaan.

Mihin perustuu kappaleen aineessa tikittävä kello? Jos mukaan otetaan kvanttimekaaninen tulkinta, asettaako se omia rajojaan itseisajalle, -nopeudelle ja -kiihtyvyydelle?



Syksy Räsänen kirjoitti:   
4.9.2014 19:19   

Eusa,

Itseisnopeus, eli havaitsijan nopeus itsensä suhteen, on aina nolla.

Kahden havaitsijan välinen nopeus ei voi ylittää valonnopeutta.



Eusa kirjoitti:   
4.9.2014 20:49   

Kyse on suhteellisuusteoreettisista käsitteistä:

http://en.wikipedia.org/wiki/Proper_velocity

https://fi.wikipedia.org/wiki/Itseisnopeus

Toivon, että voisit kuvailla lyhyesti aiheesta tehtyä tutkimusta ja nykyistä tietoa suhteellisuusteorian ja kvanttimekaniikan rajankäynnistä aiheeseen liittyen.



Syksy Räsänen kirjoitti:   
4.9.2014 21:35   

Eusa,

Kuten sanoin, kahden kappaleen välinen nopeus ei voi ylittää valonnopeutta. Asialla ei ole kvanttimekaniikan kanssa mitään tekemistä. Tämä riittäköön tästä.
Otsikko: Vs: Musta aukko ja sen muoto, tapahtumahorisontti
Kirjoitti: Eusa - 11.03.2016, 20:36:28
Joka tapauksessa yleisesti käytettävä määritelmä itseisnopeudelle on tuo wikiartikkelissa Proper velocity esitetty:
"In flat spacetime, proper velocity is the ratio between distance traveled relative to a reference map frame (used to define simultaneity) and proper time τ elapsed on the clocks of the traveling object."

Syksy halusi jostain syystä määrittää nopeuden itsensä suhteen, kun tarkoitetaan itsen nopeus kartan suhteen. Itsensä suhteen nopeudessa ei ole mitään järkeä, olisi turha käsite.
Otsikko: Vs: Musta aukko ja sen muoto, tapahtumahorisontti
Kirjoitti: mistral - 11.03.2016, 20:52:40
Ok, itse en ymmärrä mitä itseisnopeus sitten tarkoittaa enkä noista kaavoista myöskään ottanut selvää.
Otsikko: Vs: Musta aukko ja sen muoto, tapahtumahorisontti
Kirjoitti: jussi_k_kojootti - 12.03.2016, 11:27:42
Lainaus käyttäjältä: Joksa - 12.03.2016, 03:09:46
Satuja nuo horisontin sisällä tapahtuvat ilmiöt. 0-aikavyöhykkeen läpäisy ohitettu ikäänkuin ohimenevällä migreenioireella.

Silkkaa satua joka on keksaistu paremman selityksen puutteessa, ihmettelen vaan miksi siihen uskotaan eikä etsitä jotain paremmin järkeenkäypää ja toivon mukaan myös todenettavaa selitystä.

Tulihan se sieltä.

Mutta piste johdonmukaisuudesta!
Otsikko: Vs: Musta aukko ja sen muoto, tapahtumahorisontti
Kirjoitti: mistral - 12.03.2016, 12:34:03
Lainaus käyttäjältä: Joksa - 12.03.2016, 03:09:46
Nollavyöhykeen viereltä fotonin lähtiessä muuhun kosmokseen se punavenyy koko kosmoksen mittaan niin ettei ole enää havaittavissa. Vastakkaiseen suuntaan tuleva fotoni pakkautuu 0-tilavuuteen, alle plankin mitan, niin ettei senkään allopituus enää ole mitattavissa. Eli menee sinne singulariteettiin jo siinä.

Lihavoitu pätee vain jos fotoni tulee vapaasta avaruudesta. Jos se tulee kertymäkiekon sisäosista, se ei ehdi Planckin mittaan koska horisontilta pakonopeus kertymäkiekolle ei ole täyttä 300 000km/s.

Otsikko: Vs: Musta aukko ja sen muoto, tapahtumahorisontti
Kirjoitti: jussi_k_kojootti - 12.03.2016, 21:05:42
Lainaus käyttäjältä: Joksa - 12.03.2016, 16:13:16
Kiitos vaan... tarkoitukseni ei ole mitenkään vähätellä s-teorian ja sen työstäneiden upeita saavutuksia mutta teoriahan toimii aika-avaruuden puitteissa, ja aika-avaruus loppuu kun joko aika tai avaruus loppuu. Avaruuden loppumisen eli singulariteetin osalta tilanne on yleisesti myönnetty mutta ajan loppumisen osalta ei vaan teoriaa on sovellettu surutta mustan aukon sisälläkin.

Se että aika loppuisi horisontissa on yksinkertaisesti väärä luulo.  Aika-avaruus loppuu vasta singulariteetissa (horisontin ja singulariteetin välissä se käyttäytyy erikoisesti). Horisontista nousevien fotonien ääretön punasiirtymä kaukaiselle havaitsijalle ei tarkoita "ajan pysähtymistä", "putoajan ominaisajan loppumista", "aineen rakenteen värähtelyiden pysähtymistä", "0-aikavyöyhkettä" tai muitakaan vastaavia vääriä käsityksiä, vaan on suhteellisuusteoreettinen ilmiö.  Yhden suhteen ei ole suhteellisuutta <=> suhteellisuusteoreettisilla ilmiöillä on aina toinen osapuoli, ja punasiirtyneen fotonin sekä kaukaisen havaitsijan tapauksessa toinen osapuoli putosi mustaan aukkoon.

Musta aukko kuvataan Einsteinin kenttäyhtälöiden ratkaisulla, metriikalla, metrisellä tensorilla.  Schwarzschild, Reisner-Nordström, Kerr, ja vielä variaatioita teemasta.  Yksikään ei divergoi vielä horisontissa, vaan kuvaa aukon sisäpuolista aika-avaruutta hyvän matkaa kohti singulariteettia.

Schwarzschildin ratkaisu on validi ja käyttökelpoinen myös Auringolle, tai Maalle.  Taitaapa olla, että kun Eddington mittaili Auringon aiheuttamia poikkeamia taustatähtien sijainnissa, niitä verrattiin nimenomaan Schwarzschildin ratkaisuun.  Tietääkö joku, mitä metriikkaa GPS:ssä käytetään?   Kiistätkö GR:n käyttökelpoisuuden näissä tai muissa kokeellisesti vahvistetuissa tilanteissa?  Vapaa pudotus tapahtumahorisonttiin on pohjimmiltaan samaa matikkaa, ja samaa fysiikkaa, kuin vapaa pudotus Maan pinnalle; ja yleisen suhteellisuusteorian kuvaus aika-avaruuden kaarevuudesta mustan aukon sisällä (mutta singulariteetin ulkopuolella) aivan yhtä validi kuin sama tarkastelu Maan -- tai tähtien -- sisäosien gravitaatiokentästä.  Minusta tämä ei ole mikään sininen-ja-punainen-pilleri -tilanne -- vaan jos nielee yhden, joutuu nielemään toisenkin.

Lainaa
Bb:n alkuhetkien osalta tilanne lienee toisin kun siellä ei ole ollut vastaavaa gravitaatiokynnystä vaan aika on alkanut inflaatiosta. '

Kysyitkin aiemmin -- alkuräjähdyksen singulariteetti ei ole aivan sama kuin mustan aukon singulariteetti (toinen sijaitsee menneisyydessä, toinen tulevaisuudessa -- jms).  Esitit tulkinnan mustan aukon sisuksista "käänteisen alkuräjähdyksen termein".  Minusta se on kiehtova, suorastaan kutkuttava, ajatus.
Otsikko: Vs: Musta aukko ja sen muoto, tapahtumahorisontti
Kirjoitti: mistral - 12.03.2016, 23:36:34
Lainaus käyttäjältä: ketarax - 12.03.2016, 21:05:42
Se että aika loppuisi horisontissa on yksinkertaisesti väärä luulo.

Varmaan teorian mukaan voidaan edetä horisontin sisäpuolelle, siis kun ollaan paikan päällä, asiat etenee niinkuin teoria niitä kuvailee. Se ei ole ongelma, vaan se että horisontissa vapaan avaruuden historia on ohitse. Eli vapaan avaruuden aika on mennyt äärettömyyteen, eikös tämä ole divergoimista?

edit
Ja toinen divergoiminen olisi vaikka esimerkkinä tiiliskivi, jos se on pudonnut vapaasta avaruudesta suoraan ilman jarruttavia törmäyksiä, niin se saavuttaisi valon nopeuden horisontissa jolloin sen liikemassa kasvaisi äärettömäksi. Kuinka ääretön liikemassa vielä kasvaisi entisestään horisontin sisäpuolella, sehän kiihtyy vielä sisäpuolellakin.
Otsikko: Vs: Musta aukko ja sen muoto, tapahtumahorisontti
Kirjoitti: jussi_k_kojootti - 14.03.2016, 14:06:57
Lainaus käyttäjältä: Joksa - 14.03.2016, 08:54:39
Otetta wikistä:

Samassa/seuraavassa luvussa (Black holes and singularities) kerrotaan kyseessä olevan koordinaatti-singulariteetti, sekä mitä se tarkoittaa (näennäinen singulariteetti).

Lainaa
Noita Schwarzschildia uudempia metriikoita näyttäisi olevan useita, ja henki näyttäisi olevan että jos se toimii jossakin niin se toimii, eli eri metriikat olisivat eri koordinaatistoja. Tulkinnanvaraisessa tilanteessa olisi ihan perusteltua syytä edetä molempien vaihtoehtojen suhteen ja hyväksyä sipulimalli(t) muiden musta-aukkomallien joukkoon kunnes asia varmistuu.

Syytä ottaa vaihtoehto vakavasti on vasta sitten kun sille löytyy kenttäyhtälöiden ratkaisu.  Eli ei muuta kuin kehittelemään Joksan metriikkaa.  Muista, että _pelkkä_ sipuli ei riitä, saman metriikan pitää toimia vielä aukon ulkopuolisessakin avaruudessa.
Otsikko: Vs: Musta aukko ja sen muoto, tapahtumahorisontti
Kirjoitti: jussi_k_kojootti - 14.03.2016, 22:34:32
Lainaus käyttäjältä: Joksa - 14.03.2016, 16:59:12
"Näennäinen singulariteetti" on tulkinta jonka avulla saadaan aasinsilta paikansuuntaisille tapahtumille eli koordinnaatistolle jossa aika ja paikkakoordinaatit ovat vaihdettu keskenään.   Tälle operaatiolle perään parempia perusteluja ja käytännön havaintoja sen paikkansapitävyydestä tai sen asemasta  ainoana kysymykseen tulevana vaihtoehtona on luovuttava. Jos koordinaattien vaihto-operaatiosta pidättäydytään niin siitä seuraa että horisontin singulariteetin tulkinta muuttuu aidoksi.

Peräämäsi perustelu on yleisen suhteellisuusteorian postulaatti (https://fi.wikipedia.org/wiki/Yleinen_suhteellisuusteoria#Perusperiaatteet) ... :rolleyes:

Lainaa
Tarkoittaako tämä kenttäyhtälöratkaisun vaade sitä että niillä pitäisi ulottaa s-teorian aika-avaruus mustan aukon sisään? Sehän ei oikein ole ollut ajatuksena.

Asia on aivan toisin päin -- kenttäyhtälöiden ratkaisu kuvaa koko aika-avaruuden geometriaa, poislukien singulariteetit (*).  Ratkaisua ei tarvitse erikseen "ulottaa" mustan aukon (tai tähden) sisään, vaan se ulottuu sinne ihan itsestään.  Ja paljastaa, tai ainakin ennustaa, mustan aukon tapauksessa sisällä olevan singulariteetin.

Minun vaateeni kenttäyhtälöistä tarkoittaa sitä, että KUN yleinen suhteellisuusteoria on parhaiten kaikki gravitaatiosta tekemämme havainnot selittävä teoria, kannattaa aloittaa sipulihaaveiden todentaminen yrittämällä löytää mallillesi suhteellisuusteoreettinen ratkaisu.  Voit myös yrittää kehittää kokonaan oman painovoimateorian.

(*) ja tietenkin idealisoidussa tilanteessa -- schwarzschildin ratkaisu kuvaa varauksetonta, pyörimätöntä ja pallonmuotoista massaa tyhjiössä. 

Pari linkkiä jotka tuli vastaan vastauksia laatiessa:

- "Yleistajuinen fysikaalinen esitys" (http://www.mantta.fi/~hamlet/math/albert/kallio.html#luku8) vuodelta 1922.
- Sami Raatikaisen tuore kandintutkielma (http://www.helsinki.fi/~syrasane/SR-Kerr.pdf) Kerrin ratkaisun horisontti- ja singulariteettirakenteesta.
Otsikko: Vs: Musta aukko ja sen muoto, tapahtumahorisontti
Kirjoitti: jussi_k_kojootti - 15.03.2016, 12:15:26
Lainaus käyttäjältä: Joksa - 15.03.2016, 11:44:25
Tarkoittanet erityisen eli suppean suhteellisuuterosin postulaatteja, yleisessähän sellaisia ei ollut.

En tarkoita; onpas, kaikki erikoisen suhteellisuusteorian postulaatit ynnä ekvivalenssiperiaate, ynnä vielä muutama:  Fysiikan lakien muoto on sama kaikille havaitsijoille, ja kaikissa koordinaatistoissa.  Kaikki liike on geodeettistä.

Lainaa
Argumentti että "niin tapahtuu koska  s-teorian olettaa niin" ei juuri edistä asian ruotimista. Onko tästä ilmiön todentavaa havaintoa olemassa vai toimiiko s-teoria tältä osin edelleen pelkän oletuksen varassa? Jos niin muunkin vaihtoehtoisen oletuksen voi tältä osin ottaa keskusteluhypoteesiksi.

Voitaisiinhan me keskustella vaikka minun mahtavasta pyörivien aukkojen teoriasta.  Jäniksiä siellä sisäpuolella on, vahvoilla takajaloillaan potkivat aukolle impulssimomenttia.

Lainaa
Eka oli yleisesittelyä, peruskauraa,

Jep.  Ei pidä juosta ennen kuin osaa kävellä.
Otsikko: Vs: Musta aukko ja sen muoto, tapahtumahorisontti
Kirjoitti: jperala - 15.03.2016, 12:19:30
Taitaa olla sellainen aihe että ei kovin monella ole varmaa tietoa mitä siellä tapahtuu, voisi varmaan tarkentaa että ei kenelläkään ole. Tässä alkaa olemaan sellainen suohon laulamisen meininki :) Ihan mukava varmaan väitellä mutta tuskin ne faktat on hallussa yhdelläkään ihmisellä.
Otsikko: Vs: Musta aukko ja sen muoto, tapahtumahorisontti
Kirjoitti: jussi_k_kojootti - 15.03.2016, 14:05:30
Lainaus käyttäjältä: Joksa - 15.03.2016, 13:26:03
"Annettuna otettava väite täi oletus" ei ole peräämäni perustelu tai todeksi osoitettu asia.

Lähtöoletukset tulevat "toteen osoitetuksi" samalla kuin teoria.    Eli (esimerkiksi) GPS on se peräämäsi perustelu.

Lainaus käyttäjältä: jperala - 15.03.2016, 12:19:30
Taitaa olla sellainen aihe että ei kovin monella ole varmaa tietoa mitä siellä tapahtuu, voisi varmaan tarkentaa että ei kenelläkään ole. Tässä alkaa olemaan sellainen suohon laulamisen meininki :) Ihan mukava varmaan väitellä mutta tuskin ne faktat on hallussa yhdelläkään ihmisellä.

Väärinkäsitykset on tehty korjattaviksi! (https://xkcd.com/386/)
Otsikko: Vs: Musta aukko ja sen muoto, tapahtumahorisontti
Kirjoitti: mistral - 15.03.2016, 20:51:11
Ketarax, sinulla on nyt todistustaakka todistaa että mustissa aukoissa aika ei ole pysähtynyt meidän näkökulmasta. Jos onnistut, silloin materia ei olekaan "jäässä" siellä ja tapahtumia voidaan pitää meille todellisina, jos et, jää teoriat aukon sisäisistä tapahtumista mielenkiintoisiksi teorioiksi  :wink:
Otsikko: Vs: Musta aukko ja sen muoto, tapahtumahorisontti
Kirjoitti: jussi_k_kojootti - 16.03.2016, 08:25:57
Lainaus käyttäjältä: mistral - 15.03.2016, 20:51:11
Ketarax, sinulla on nyt todistustaakka todistaa että mustissa aukoissa aika ei ole pysähtynyt meidän näkökulmasta.

Enhän minä ole kertaakaan sanonut ettei aika meidän, siis kaukaisen havaitsijan, näkökulmasta näytä pysähtyvän.  Kaukaiselle havaitsijalle fotonit punasiirtyy ja fotonivuo heikkenee, putoaja ei koskaan ylitä horisonttia.  Tästä ei ole mitään kiistanalaisuutta sen enempää meidän välillä kuin suhteellisuusteoriassakaan.

Olen sanonut: putoajan aika (ominaisaika (https://en.wikipedia.org/wiki/Proper_time)) ei pysähdy horisontissa, eikä aine jäädy siihen;  ja putoaja putoaa horisontin toiselle puolelle, jossa tapahtuu asioita.  Olen sanonut tämän, koska sinä ja Joksa tunnutte kuvittelevan että kaukaisen havaitsijan näkökulma on jotenkin absoluuttinen, oikea.  Näin ei ole.  Totta puhuen molemmat näkökulmat ovat aivan oikeita, ja molemmat versiot "todella tapahtuvat" (eli havaitsija todella mittaisi punasiirtynyttä valoa, ja putoaja todella ylittäisi horisontin).  Ja tämä on tietenkin maalaisjärjen paradoksi.

Haluatko todistuksen tälle väittämälle ("ominaisaika ei lopu horisontissa"), ja missä muodossa ("fyysikoiden lausuntoja", vai matematiikkaa)?  Voit toki tutkia asiaa itsekin, googlaa "does time stop at horizon".
Otsikko: Vs: Musta aukko ja sen muoto, tapahtumahorisontti
Kirjoitti: jussi_k_kojootti - 16.03.2016, 08:32:53
(voi poistaa)
Otsikko: Vs: Musta aukko ja sen muoto, tapahtumahorisontti
Kirjoitti: jussi_k_kojootti - 16.03.2016, 10:49:50
Lainaus käyttäjältä: Joksa - 16.03.2016, 09:17:35
Tuolla googlauksella löytyi ainakin tällainen uutinen
http://www.extremetech.com/extreme/175414-stephen-hawking-research-there-are-no-black-holes

AE itsekin taisi olla sitä mieltä alunperin. Ehkä asiassa on sittenkin vielä - jopa joidenkin 'lujaa juoksevien mielestä' - tutkimisen varaa vaikka taas joidenkin toisten mielestä GR tietää ja selittää kaikken;)

Älä pistä sanoja suuhuni, tai siis sormenpäihin.  En ole missään vaiheessa sanonut että GR "tietää ja selittää kaiken" (*);  vaan olen sanonut, että teillä on vääriä luuloja ja väittämiä mustien aukkojen, tapahtumahorisontin ja YS:n ominaisuuksista.  Eli, kun puhuitte sipuleista, puutuin niihin kohtiin (oikeastaan kai vain yhteen kohtaan) jotka _nykytietämyksen valossa ovat varmasti väärin_ -- kuten "0-aikavyöhyke jossa aine jäätyy sipuleiksi", tmv. variaatiot ajatuksestanne.  Sinällään sipuli-haaveilussa ei ole minusta mitään moitittavaa, mutta fysiikan vääristely ... no, huvinsa kullakin, kai.

Voi kai olla että kvanttigravitaation jälkeen näkemyksenne osoittautuvatkin oikeiksi.  Vielä sittenkään väittämät kuten "yleisessä suhteellisuusteoriassa mustan aukon tapahtumahorisontissa aine jäätyy sipuleiksi" -- tmv -- ei muutu oikeaksi, vaan on edelleen YS:n (nykyversion) virhetulkinta / väärä käsitys.

(*) Vaikka niinhän se toistaiseksi tekee -- mitä aika-avaruuden havaittuun geometriaan tulee.

Täällä on vaikka kuinka paljon matematiikkaa, ja myös esimerkkilaskuja, mm. tapahtumahorisontin ylityksestä:  Physics Pages (http://www.physicspages.com/tag/event-horizon/).
Erityisesti:  Aukkoon putoajan ominaisajasta (http://www.physicspages.com/2013/09/09/falling-into-a-black-hole/)

Lopetan samojen asioiden toistelun tähän.  Kun keksitte uusia väärinkäsityksiä tai muuten mielenkiintoista pohdittavaa, niin jatketaan toki!
Otsikko: Vs: Musta aukko ja sen muoto, tapahtumahorisontti
Kirjoitti: jussi_k_kojootti - 16.03.2016, 17:30:57
Lainaus käyttäjältä: Joksa - 16.03.2016, 12:00:58
PS3: ristiriitaitasuus ei ilmeisesti - pienen uudelleenharkinnan jälkeen - ole teoriassa vaan edelleenkin sen tulkinnassa. Eli jos putoajan aika menee niiin hitaaksi että horisontin saavuttaminen ei tapahdu kosmoksen elinkaaren aikana, vaikka putoaja menesi sinne omassa äärellisessä ajassaa niin ulkoisen kosmoksen kannalta sitä ei tapahdu, eli oltaisiin asymptoottisessa tulkinnassa. Asia on tosiaan tullut esiin ketjussa jo aemmin eli menee tosiaan toiston puolelle.

Siinä se on!  Pidä toi! 

Tarkistus:  olemmeko nyt yhtä mieltä siitä, että kun ominaisaikakellon mukaan on kulunut yksi puoliintumisaika, radioaktiivista ainetta on jäljellä tasan puolet, missä ikinä maailmankaikkeuden kolkassa -- lähellä horisonttia tai kaukana siitä -- mittaus tehdäänkin.  ?

(En ole aivan varma ymmärrämmekö "asymptoottisen tulkinnan" samalla tavalla -- minä ymmärrän nämä tulkinta-asiat niin, että sekä kaukaisen havaitsijan että putoajan havainnot ovat tosia -- eli putoaja ylittää horisontin, ja havaitsija todella mittaa punasiirtyneitä fotoneita (eikä koskaan mittaa sitä näennäiseltä "0-aikavyöyhkkeeltä" lähtevää fotonia).  Toisin sanoen yhden sijaan totuuksia on kaksi -- ja tämä on edelleen mielestäni "paradoksaalista" (mutta pakko se on niellä, koska GPS jne.)

Lainaus käyttäjältä: Joksa - 16.03.2016, 16:20:13
Summa summaarum kohtuullisen pontevan ja asiantuntevan opponoinnin jälkeenkään ei ole tullut mitään pitävää argumenttia sitä vastaan etteikö kehittelemämme asymptoottinen eli sipulimalli 1 olisi täysin tiedossa olevien tosiasioiden mukainen, ja nimenomaan suhteellisuusteorian.

Tästä olen hieman eri mieltä ... muistaakseni vielä ei ole tuotu selvästi esiin sitä seikkaa, että tapahtumahorisontin sisäpuolella avaruus putoaa tulevaisuuteen eli singulariteettiin ylivalonnopeudella.  Vaikuttaako tämä sipulimalliin? :-)

Lainaa
Kritiikki kohdistuu korkeintaan siihen seikkaan että horisontin ylittämisen laskennassa on painotettu pelkästään putoavan aineen aikaskaalaaa

Niin kuin putoajalle kuuluukin.  Absoluuttista, siis yhtä ainoaa ja oikeaa, aikaa ei suhteellisuusteorian mukaan ole olemassa.

Lainaa
Näin voi putoajan ajan osalta ollakin mutta muun kosmoksen kannalta kuuluva aika on ääretön jolloin ylitys ei tapahdu kosmoksen elinkaaren kuluesssa.

Juuri näin -- ilman suhteellisuusteorian tarjoamaa "lisätietoa" ulkomaailma ei koskaan tietäisi putoajan päätyneen horisontin tuolle puolen.

Lainaa
Itse ajattelin jatkossa syventyä Hawkingin ajatuksiin, hän tuntuisi profiloituneen siten että uudet näkökulmat ja painotukset tulevat kyseeseen.

Luulisin että ennestään tuntematon fysiikka kiinnostaa jokaista fyysikkoa.  "Ongelma" on siinä, että YS ja kvanttiteoriat 'kiinnittää todellisuutta ja todellisuudentajuamme' niin vahvasti, että ns. aukkopaikkoihin on _erittäin_ vaikea mahduttaa sellaista uutta, mikä ei pilaisi vanhaa.  Tämä on yksi (luulisin että merkittävä, tietenkin tarvittavien havaintojen puutteen lisäksi) tekijä siinä, että kvanttigravitaation teoria antaa odottaa itseään.

Kuten sanoin, "käänteisen mini-big-bangin sipuli" on mielestäni fantastinen ajatus.  Omasta puolestani voin kuvitella em. ylivalonnopeudella tippumisen käänteiseksi inflaatioksi.  Joksa-Ketaraxin metriikkaa odotellessa jne :-)

Pohdittavaa:  kuinka kauan "ajan pysähtyminen" kestää?  Yhden ainoan hetken.
Otsikko: Vs: Musta aukko ja sen muoto, tapahtumahorisontti
Kirjoitti: mistral - 16.03.2016, 20:24:10
Lainaus käyttäjältä: ketarax - 16.03.2016, 17:30:57
(En ole aivan varma ymmärrämmekö "asymptoottisen tulkinnan" samalla tavalla -- minä ymmärrän nämä tulkinta-asiat niin, että sekä kaukaisen havaitsijan että putoajan havainnot ovat tosia -- eli putoaja ylittää horisontin, ja havaitsija todella mittaa punasiirtyneitä fotoneita (eikä koskaan mittaa sitä näennäiseltä "0-aikavyöyhkkeeltä" lähtevää fotonia).  Toisin sanoen yhden sijaan totuuksia on kaksi -- ja tämä on edelleen mielestäni "paradoksaalista"

Tai oikeastaan totuuksia on yksi mutta se yksi totuus on kaksiosainen (tai useampiosainen). Ensimmäinen osa kattaa vapaan avaruuden aikakauden, toinen osa poissulkee ensimmäisen ja kattaa horisontin sisäpuolta. Mutta se että se kattaa sisäpuolta, ei tarkoita että kattaisi koko sisäpuolen, sillä kun astronautti syöksyy horisontin läpi, on hänen näkökulmasta seuraava horisontti syvemmällä syövereissä. Se on yksinkertaisesti niin syvällä, mistä ei edes valo pääse pakenemaan ykkös horisontille. Ja edelleen kun hän on kakkoshorisontilla, odottaa kolmoshorisontti seuraavana. Eli tavallaan voidaan niinkin ajatella että horisontti olisi liukuva taso.

Kummallista tässä on jos ykköshorisontin aika on pysähtynyt, niin ykköseltä katsottuna taas kakkosen aika on pysähtynyt jne. Eli kuinka pysähtyneen ajan suhteen voi jonkun toisen paikan aika pysähtyä? Tämä tietysti vaatii sen että suhteellisuusteoria on voimassa gravitaatiokaivon pohjalle saakka.
Otsikko: Vs: Musta aukko ja sen muoto, tapahtumahorisontti
Kirjoitti: jussi_k_kojootti - 17.03.2016, 00:00:27
Lainaus käyttäjältä: mistral - 16.03.2016, 20:24:10
Tai oikeastaan totuuksia on yksi mutta se yksi totuus on kaksiosainen (tai useampiosainen).

Tämän nielen mukisematta!

Lainaa
Ensimmäinen osa kattaa vapaan avaruuden aikakauden, toinen osa poissulkee ensimmäisen ja kattaa horisontin sisäpuolta.

Eikös se ole juuri näin!  Toki molemmat osat tulevat yhdestä ja samasta ratkaisusta -- mutta tämä on mitätön tekninen yksityiskohta.


Lainaa
Mutta se että se kattaa sisäpuolta, ei tarkoita että kattaisi koko sisäpuolen, sillä kun astronautti syöksyy horisontin läpi, on hänen näkökulmasta seuraava horisontti syvemmällä syövereissä. Se on yksinkertaisesti niin syvällä, mistä ei edes valo pääse pakenemaan ykkös horisontille. Ja edelleen kun hän on kakkoshorisontilla, odottaa kolmoshorisontti seuraavana. Eli tavallaan voidaan niinkin ajatella että horisontti olisi liukuva taso.

Tämän voi tulkita kahdella tavalla, joista toinen on (kai) OK, ja toinen ei. 

OK:
Enpä keksi syytä miksei aukon sisäpuolta voisi sanallisessa tarkastelussa jakaa "virtuaalisiksi kuoriksi", tai "pseudokuoriksi".  Jätetään nyt ajan ja paikan käsitteiden vaihtuminen käytännössä kokonaan huomiotta -- eli sanon "sisäpuoli" ikäänkuin musta aukko olisi jalkapallo. En ole varma onko tämä edes väärin, luulisin että on, tavallaan, ainakin, kai.

Kaikille etäisyyksille singulariteetista, r < r_s, missä r_s schwarszchildin säde, pätee, että vapaaksi päästetyn fotonin etäisyys singulariteetista pienenee.  Siis sellaisellekin, joka on lähetetty suoraan kohti ulointa horisonttia.  Se ei siis liiku inasen pätkää ulospäin, ei pysyttele hetkeäkään paikallaan, vaan heti emittoiduttuaan putoaa vain ja ainoastaan kohti singulariteettia.  Fotoni EI hidastu nopeuteen nolla ja sitten kiihdy takaisin c:hen pudotakseen singulariteettiin; fotonit kulkevat aina geodeesejä pitkin, ja horisontin sisäpuolella kaikki geodeesit osoittavat singulariteettiin.

Täten mikä tahansa etäisyys r singulariteetista (edelleen r < r_s) käyttäytyy ulkopuolisen (r_u > r < r_s) tarkkailijan vinkkelistä kuten uloin horisontti.  Tällaisten kuorien lukumäärä on ~ääretön, eikä niillä ole nähdäkseni sen kummempaa merkitystä (toisin kuin uloimmalla horisontilla, joka tehokkaasti jakaa aika-avaruuden kahteen osaan).  Siksi niitä ei myöskään ole erikseen syytä mainita, tai nimetä.  Aivan erityisesti tämmöisissä kuorissa ei ole ainetta tai energiaa "jäätyneenä", "pysähtyneenä", tms.

Ei-OK:
Jäätyneitä, ajassa pysähtyneitä (mitä se edes tarkoittaa), materiaa pidätteleviä kuoria, tms.  Uloin horisontti ei ole tällainen.  Myöskään varattujen ja/tai pyörivien aukkojen sisempi horisontti ei ole tällainen (sisempi horisontti on ilmeisesti kirkas, mutta vain aukon ulkopuolisesta valosta (jos sitä ei tule, sisempi horisontti pimenee)).

Lainaus käyttäjältä: Joksa - 16.03.2016, 20:41:15
Putoajan havaitsema kosmoksen vanhenemisvauhti sen sijaan ei pysähdy mihinkään äärelliseen arvoon vaan hän näkee kosmoksen saavuttavan loppunsa ennen tuloaan tapahtumahorisonttiin.

Loppuun asti ei näy, valonnopeus on kaikille sama.  Jos putoajalla on horisontin jälkeen ominaisaikaa jäljellä 10s, niin nähtävillä on ne fotonit jotka oli horisontin ylityksen hetkellä 10 valosekunnin etäisyydellä putoajasta.

Jäätymättömyydestä englanniksi (http://math.ucr.edu/home/baez/physics/Relativity/BlackHoles/fall_in.html) (John Baez).

Kuten aiemmin, kirjoitettu koskee Schwarzschildin aukkoa (ellei muuta mainittu) niin kuin minä sen "puhtaan" YS:n puitteissa ymmärrän.
Otsikko: Vs: Musta aukko ja sen muoto, tapahtumahorisontti
Kirjoitti: Eusa - 17.03.2016, 11:33:41
Osasitpa Joksa kirjoitella selvästi saman, jonka olen itsekin opiskellut ja josta saa oivan perustan kvanttimekaniikan ja yleisen suhteellisuuden yhdistelmälle ääriolosuhteissa.

Seuraava johtopäätökseni oli, että alkeishiukkanen täyttää tapahtumahorisontin ja singulariteetin tunnusmerkit 1/2spin-vaiheiden välissä, mutta se on toinen tarina...

Jos ajatellaan teoreettista mustaa aukkoa massiivikohteen romahtaessa, olen jäänyt joskus miettimään kuinka suureksi romahtavan aineen mittakaavassa keskiön epätarkkuus eli asymptoottinen tapahtumahorisontti kasvaa ja varaako se erikseen runsaasti tilaa ja kenen koordinaatistossa?
Otsikko: Vs: Musta aukko ja sen muoto, tapahtumahorisontti
Kirjoitti: jussi_k_kojootti - 17.03.2016, 15:05:39
Lainaus käyttäjältä: Joksa - 17.03.2016, 10:08:52
10s riittää hyvin. Kosmoksen loppua on kuvattu siten että ensivaiheessa pimeä energia repii galaksit, tähdet ja muut planeetat niin kauas havaitsijasta että taivas pimenee etäältä alkaen niiden valon punasiirtyessä havaitsemattomiin. Eli avaruudessa ei olisi enää mitään nähtävääkään 10 valosek. etäämpänä.

Vaadin lisärahoitusta SETI-hankkeille!  Jos tuolla yksikin minunkaltaiseni humoristi löytää mustan aukon, niin päivämme ovat luetut.

Edelleen sama kuin aiemmin:  absoluuttiset käsitykset ajasta on vääriä.  Sinun, tai protonin, putoaminen mustaan aukkoon ei kosmosta hetkauta (anteeksi vain).

Kosmosta voi odottaa Big Rip (https://en.wikipedia.org/wiki/Big_Rip), tai sitten ei, tällä hetkellä mittaukset antavat ymmärtää että ehkä ei (artikkelin loppupäässä, parametri w).

Lainaa
"Viimeiset värinät maailmakaikkeudessa syntyvät protonien mahdollisesta hajoamisesta, jonka arvioidaan tapahtuvan 10^46 – 10^200 vuoden kuluttua." http://www.tieteessatapahtuu.fi/987/markkanen.htm

Protonien hajoaminen (https://en.wikipedia.org/wiki/Proton_decay) on aivan eri tarina kuin mahdollinen Big Rip.
Otsikko: Vs: Musta aukko ja sen muoto, tapahtumahorisontti
Kirjoitti: jussi_k_kojootti - 17.03.2016, 20:36:54
Lainaus käyttäjältä: Joksa - 17.03.2016, 19:42:18
Tarvitaanko tähän välttämättä sitä absoluuttista yleistä kosmista aikaa,

Minulla on suuria vaikeuksia ymmärtää visiotasi muutenkaan.  Alunperin valitsit kaukaisen havaitsijan ajan universaaliksi, nyt putoajan -- tai siis siltä minusta näyttää.

Lainaa
w=-1 tasan ja sitä big rippiä ei tulisikaan niin meidän aikaskaalassamme r=0 singulariteetti ei edelleenkään muodostuisi missään äärellisessä ajassa, horisonttia lähestyminen vain jatkuisi hamaan ikuisuuteen.

Mustan aukon muodostuessa on horisontin muodostava tiheys olemassa ennen horisonttia.  OK? 
Otsikko: Vs: Musta aukko ja sen muoto, tapahtumahorisontti
Kirjoitti: mistral - 18.03.2016, 00:03:27
Lainaus käyttäjältä: ketarax - 17.03.2016, 00:00:27
Kaikille etäisyyksille singulariteetista, r < r_s, missä r_s schwarszchildin säde, pätee, että vapaaksi päästetyn fotonin etäisyys singulariteetista pienenee.  Siis sellaisellekin, joka on lähetetty suoraan kohti ulointa horisonttia.  Se ei siis liiku inasen pätkää ulospäin, ei pysyttele hetkeäkään paikallaan, vaan heti emittoiduttuaan putoaa vain ja ainoastaan kohti singulariteettia.  Fotoni EI hidastu nopeuteen nolla ja sitten kiihdy takaisin c:hen pudotakseen singulariteettiin; fotonit kulkevat aina geodeesejä pitkin, ja horisontin sisäpuolella kaikki geodeesit osoittavat singulariteettiin.


Tuo tarkoittaisi epäjatkuvuuskohtaa horisontissa. Tarkoitan, sen yläpuolella, vaikka vain 1mm, fotoni lähtisi ylöspäin, mutta alempana se törmäisi "lasikattoon". Eikö olisi siistimpää antaa sen fotonin lähteä lasikaton läpi ja kaartaa kuin tykinkuula takaisin mustaan aukkoon?

Tosin tässä tullaan mielenkiintoiseen asiaan, jos fotoni ei osaa päättää mihin suuntaan lähtee kaartamaan, se häviäisi niin kuin virtuaalihiukkanen vakuumissa. Eikös fotonin negatiivinen ja positiivinen energia, siis jos ne on yhtä suuret, voisi hävittää sen olemasta? Negatiivinen on sen syvyys kaivossa, ja positiivinen on sen "pakonopeus". Jos pakonopeus ei riitä, voi yhteenlaskulla hävittää koko fotonin. Tämä siis vain niiden fotonien kohdalla jotka ei osaa ajoissa kaartaa takaisin.

Joka tapauksessa, en purematta niele lasikatto teoriaa koska siinä 1mm sisällä ikään kuin lakkautetaan liikemäärän säilymislaki.
Otsikko: Vs: Musta aukko ja sen muoto, tapahtumahorisontti
Kirjoitti: Eusa - 18.03.2016, 16:32:45
Avainasia mielestäni on lisätä ymmärrystä siitä kuinka usean kappaleen ryhmä ja vuorovaikutukset määrittävät metriikan yhteisesti. Yleisen suhteellisuusteorian lineaariapproksimaatiot perustuvat rauhalliseen symmetriaan yhden havaitsijan näkökulmasta. Kun mittaajia ja vuorovaikutuksia on useita, voi yleisen suhteellisuusteorian koordinaattisynteesi poiketa approksimaatiosta huomattavastikin. Häiriöteoreettisen syklisyyden soveltaminen globaalina mekanismina voisi avata jotain..
Otsikko: Vs: Musta aukko ja sen muoto, tapahtumahorisontti
Kirjoitti: mistral - 19.03.2016, 18:46:35
Lainaus käyttäjältä: mistral - 18.03.2016, 00:03:27

Joka tapauksessa, en purematta niele lasikatto teoriaa koska siinä 1mm sisällä ikään kuin lakkautetaan liikemäärän säilymislaki.

Tuli sellainen ajatus, jospa horisontin sisäpuolella oletetaan kaiken menevän valon nopeudella alaspäin. Jos on näin, silloin esim elektronin emittoiman fotonin liikemäärä olisi nolla suhteessa aukkoon ja voisi tipahtaa saman tien alaspäin. Asiaa voi lähestyä siltäkin suunnalta että jos siis elektroni menee c nopeudella alaspäin, niin sen emittoima fotoni kohti zeniittiä olisi äärimmäisen punasiirtynyt ja siksi se häviäisi olemattomiin tai sitten putoaisi saman tien alaspäin. Mutta tämä vain sillä ehdolla että elektroni menisi c nopeudella alaspäin. Tilanne muuttuu jos materiaa putoaa kiekon alaosista  jossa se on menettänyt nopeutta, silloinhan c:tä ei saavuteta vielä horisontissa.

Mielenkiinnolla odotan Alma teleskooppien tuloksia Sagittarius a:sta.
Otsikko: Vs: Musta aukko ja sen muoto, tapahtumahorisontti
Kirjoitti: mistral - 25.03.2016, 15:30:09
Lainaus käyttäjältä: ketarax - 17.03.2016, 00:00:27


Jäätymättömyydestä englanniksi (http://math.ucr.edu/home/baez/physics/Relativity/BlackHoles/fall_in.html) (John Baez).
Lainaus linkistä:

Will you see the universe end?

If an external observer sees me slow down asymptotically as I fall, it might seem reasonable that I'd see the universe speed up asymptotically—that I'd see the universe end in a spectacular flash as I went through the horizon.  This isn't the case, though.  What an external observer sees depends on what light does after I emit it.  What I see, however, depends on what light does before it gets to me.  And there's no way that light from future events far away can get to me.  Faraway events in the arbitrarily distant future never end up on my "past light-cone," the surface made of light rays that get to me at a given time.


Tuossa Matt McIrvin sanoo ettei horisontista katsottuna "mielivaltaisen kaukainen tulevaisuus koskaan lopu menneen ajan valokartiossa". Kuitenkin ajattelen että jos pinnassa aika pysähtyy, kuluu avaruuden koko ikuisuus läpi ja jotta valo ehtii tulla jostain 20 mrd vv:n päästä (hatusta vedetty matka), on se kova pähkinä, eikös siinä rikota sitä postulaattia että valon nopeus on kaikille havaitsijoille sama?

Pitäisikö tästä tehdä se johtopäätös että myös gravitaatiokaivo aiheuttaa pituuskontraktion? Tämä auttaisi ymmärtämään sen että pinnassa tulee kertarysäyksenä koko historian fotonit, siis jos kontraktio toisi "metrin" päähän koko universumin, niin kaikki fotonit ehtisivät samaan rysäykseen, eikä postulaattia tarvitsisi purkaa.
Otsikko: Vs: Musta aukko ja sen muoto, tapahtumahorisontti
Kirjoitti: Eusa - 25.03.2016, 17:50:12
Hyvähän on pohtia. Schwarzschildin geometrian mukaan mustan aukon säde ilmoitetaan kuin avaruus olisi laakea eli se on kaukaisen havaitsijan mittakaavaa. Aukkoon putoavan kannalta mittakaava on venynyt säteen suunnassa, ei lyhentynyt. Asia on tosin hieman moniselitteinen, koska avaruusaika on kaareutunut kappaleiden kannalta yhä enemmän ajanluonteiseksi ja kiertoradallakin kappale ikään kuin liikkuu enemmän kuin mitä kiertoratamitta antaisi liikemääräksi.

Paradoksin saisi purettua sillä, että tila venyisi myös kiertoradan suunnassa. Tällä olisi sekin fiksu merkitys, että aukon ympäri pyörivät kappaleet säilyttäisivätkin mittakaavansa ulkoisen havaitsijan tarkastelussa. Geometria tosin muuttuisi niin paljon, että vain alle 45 asteen valokartiokallistumat olisivat mahdollisia ja tapahtumahorisonttia ei todellisesti muodostuisikaan, aine ja massa mahtuisivat miniatyyristyneinä laajentuneeseen alueeseen.

Visio kaikkeuden kutistumisesta metrin mittaan perustuu ainakin osin absoluuttiseen aikaideaan, eikä toimi sellaisenaan. Suhteessa valonnopeuteen c verraten tilanne toki on samanluonteinen kuin relativistisen nopeuden kyydissä menosuunnan pituuskontraktio, mutta aukko on pallomainen ja signaalisuunta kääntyy, pallokuorella on pituutta runsaastikin. Paikallisesti avaruus on vuorovesi-/sidosvoimarajalle liki laakea. Itseisnopeudeltaan muuhun kaikkeuteen verrattuna kiertoratanopeus on joka tapauksessa hurja, vaikka kiertoratamittaa tulisikin kaarevuuden myötä lisää.

Siis pohdiskelua.
Otsikko: Vs: Musta aukko ja sen muoto, tapahtumahorisontti
Kirjoitti: mistral - 25.03.2016, 18:01:20
Lainaus käyttäjältä: Eusa - 25.03.2016, 17:50:12
Hyvähän on pohtia. Schwarzschildin geometrian mukaan mustan aukon säde ilmoitetaan kuin avaruus olisi laakea eli se on kaukaisen havaitsijan mittakaavaa. Aukkoon putoavan kannalta mittakaava on venynyt säteen suunnassa, ei lyhentynyt.

Tarkoittaako Schwarzschildin geometria sitä että säteen pituus kasvaa samassa suhteessa kuin aika hidastuu? Eli jos aika pysähtyy, säde kasvaa äärettömyyteen?
Otsikko: Vs: Musta aukko ja sen muoto, tapahtumahorisontti
Kirjoitti: Eusa - 25.03.2016, 21:06:04
Lainaus käyttäjältä: mistral - 25.03.2016, 18:01:20
Tarkoittaako Schwarzschildin geometria sitä että säteen pituus kasvaa samassa suhteessa kuin aika hidastuu? Eli jos aika pysähtyy, säde kasvaa äärettömyyteen?
Schwarzschildin yleisen suhteellisuusteorian ratkaisu on lineaariapproksimaatio tensoriteoriasta. Siinä säde kuvaa mustan aukon sädettä laakean avaruusajan projektiona. Paikallisessa tilanteessa tapahtumahorisontin lähellä ei voi mitata etäisyyttä edes tapahtumahorisonttiin, koska mittakeppinä on valo. Kiertoradalta mitattuna etäisyys lähestyy ääretöntä kuta lähemmäs tapahtumahorisonttia mitataan. Tosin kiertoratoja voi olla vain yli 1,5-kertaisen Schwarzschildin säteen etäisyydellä aukon keskustasta.

Tuo ratkaisu on kovin ilmeisesti hyvin väärä, kun huomioidaan, että kvanttimekaniikankin tulee toteutua noissa oloissa...
Otsikko: Vs: Musta aukko ja sen muoto, tapahtumahorisontti
Kirjoitti: mistral - 26.03.2016, 19:48:52
Lainaus viestistä 106:

If an external observer sees me slow down asymptotically as I fall, it might seem reasonable that I'd see the universe speed up asymptotically—that I'd see the universe end in a spectacular flash as I went through the horizon.  This isn't the case, though.

Tässä Matt McIrvin kuvaa ylivalonnopeudella tapahtuvaa loppuleimahdusta (jonka ilmeisesti kiistää). Mutta mikä estää olettamasta ylivalonnopeutta? Siis samalla tavalla kuin valo jäätyy horisontissa, eli menee alivalonnopeutta, voisiko samaa ajattelua myös soveltaa ylivalonnopeuteen? Kuitenkaan siinä ei rikota postulaattia omassa koordinaatistossa. Postulaatti siis rikkoutuu sekä ali- että ylinopeuden suhteen toisessa koordinaatistossa. Jos ylinopeus sallitaan, voidaan mielestäni sen avulla ymmärtää loppuleimahdus horisontissa, silloinhan avaruuden äärilaidalta tulevat fotonit ehtii samaan ruuhkaan, eli sekunnin murto-osassa horisontti ruuhkautuu materiasta ja fotoneista.

Ja jos ylinopeutta vielä miettii, ei se ole ylinopeutta fotonin tai kiven omassa koordinaatistossa, niissä liikutaan tutuissa ES:n puitteissa.
Otsikko: Vs: Musta aukko ja sen muoto, tapahtumahorisontti
Kirjoitti: mistral - 27.03.2016, 19:11:04
Lainaus käyttäjältä: Joksa - 27.03.2016, 10:37:53
a) loppu tulee kosmoksen pimentymisenä mikäli etäisyys jolta valon ennättää tulla putojan näkyviin lyhenee tapahtumahorisonttia lähestyttäessä. Samankaltainen asia kuin kosmoksen horisontin läheneminen kosmoksen loppunäytöksessä, eli olisi kosmoksen loppunäytös putoajan koordinaatistossa. Ei leimahdusta tms ruuhkaa horisontin tienoilla.

Totta, jos astronautti menee vauhdilla alaspäin, vaikuttaa doplerin ilmiö eli punasiirtymä sen että kosmos pimenee, ainakin liikesuunnassa. Eikä ruuhkaakaan välttämättä ole edessä, sensijaan takana sitä on koska koko universumin historia pakkautuu lähiavaruuteen, siis se osa historiasta joka sattuu aikojen saatossa juuri siihen avaruuden kolkkaan.
Otsikko: Vs: Musta aukko ja sen muoto, tapahtumahorisontti
Kirjoitti: mistral - 14.04.2016, 16:44:49
Lainaus käyttäjältä: Joksa - 14.04.2016, 14:48:06
Hologrammimassa ei reagoisi enää gravitaatioon (se kun on vaan 'informaatiota') eikä siten painuisi kvanttifysiikan kammoamaksi informaation kadottavaksi singulariteetiksi, ja koska hologrammimassa edelleen sisältäisi informaation hiukkastensa gravitaatiovaikutuksista niin hologrammistuneen aineen gravitaatiovaikutus ulkoiseen aika-avaruuteen ei katoasi.

Aina silloin tällöin puhutaan informaatiosta, itselle se on kuitenkin vain ilmassa leijuva asia, en tiedä mikä sen tarkoitus on. Siis mitä väliä on informaatiolla? Jos yhteen pölyhiukkaseen sisältyy enemmän informaatiota kuin teran kovalevyyn, ketä se hyödyttää?

Eli onko ideana säilyttää katkeamaton syy/seuraus ketju?
Otsikko: Vs: Musta aukko ja sen muoto, tapahtumahorisontti
Kirjoitti: mistral - 16.04.2016, 19:44:08
Tosiaan mustan aukon sisällä jopa kvarkitkin olisi pysähtyneet. Näin myös gravitonit olisi pysähtyneet eikä ne voisi siirtää vuorovaikutusta. Voisiko tämän ongelman kiertää heittämällä gravitonit roskikseen  :grin: ja käyttää vanhaa mallia avaruuden kaareutumisesta jossa massaenergia tunnistaa kaareutumisen mikä johtaa samaan lopputulokseen kuin gravitonit.
Otsikko: Vs: Musta aukko ja sen muoto, tapahtumahorisontti
Kirjoitti: Eusa - 16.04.2016, 20:35:07
Lainaus käyttäjältä: mistral - 16.04.2016, 19:44:08
Tosiaan mustan aukon sisällä jopa kvarkitkin olisi pysähtyneet. Näin myös gravitonit olisi pysähtyneet eikä ne voisi siirtää vuorovaikutusta. Voisiko tämän ongelman kiertää heittämällä gravitonit roskikseen  :grin: ja käyttää vanhaa mallia avaruuden kaareutumisesta jossa massaenergia tunnistaa kaareutumisen mikä johtaa samaan lopputulokseen kuin gravitonit.
Gravitonit, jotka signaloivat gravitoivasta kohteesta ulospäin, on tuhoontuomittu viritys.

Yksinkertainen toimiva malli gravitoneista on esimerkiksi sellainen, jossa gravitonit ovat tilakvantteja ja liikkuvat tilasta gravitoivaan kohteeseen (valonnopeudella c ilman liikemäärää). Gravitaatioaaltojen seisova (pallokuorityyppinen) pitkittäisaallokko voi kulkea gravitoivan kohteen mukana ohjaamassa kvantteja kohti kohdetta. Jos gravitoiva kohde muuttuu, seisovassa aallokossa siirtyy muutoshäiriö nopeudella c korjaten seisovan aallokon muuttuneen gravitoivan muodon mukaiseksi.

Tuo esimerkki on oma johdokseni yleisen suhteellisuusteorian mekanismiksi.
Otsikko: Vs: Musta aukko ja sen muoto, tapahtumahorisontti
Kirjoitti: mistral - 20.04.2016, 19:08:37
Katsoin vielä uudestaan Kumpulan videon gravitaatioaalloista. Nyt tuli tenkkapoo ajan hidastumisesta. Kun kaksi noin 30 auringonmassaista kappaletta kiertää toisiaan sekunnin osissa, tässä ei ole ongelmaa. Ongelma on ajan hidastumisessa eli kun kappaleet menee lähemmäs toisiaan, on niiden yhteinen gravitaatiokaivo syvenevä ja jokainen kierros tapahtuu hitaammin aikadilataation takia. Toki kappaleen "omasta mielestä" kierrokset vaan nousee, tämä selvä, mutta jos aikadilataatio menee äärettömyyteen, voittaa se kierrosten nousun. Tämä  tarkoittaisi sitä että Ligon signaalin pitäisi olla taajuudeltaan laskeva. Kuvista päätellen taajuus ei ainakaan laske.
Otsikko: Vs: Musta aukko ja sen muoto, tapahtumahorisontti
Kirjoitti: mistral - 22.04.2016, 00:00:53
Lainaus käyttäjältä: Joksa - 20.04.2016, 22:18:24
Aukkojen osalta mieltäisin tilanteen niin että 'ainetoimintojen' (esim radioaktiivinen puolittuminen ym) aika on sisäinen, hidastuva aikaa kun kiertoliike käyttääkin ulkoista aikaa.

Kun ajatellaan pyörimätöntä mustaa aukkoa, on vielä helppoa tajuta kuinka se kasvaa miljardien vuosien aikana kerrostumalla. Mutta kun mennään Ligon havaintoon jossa kaksi aukkoa pyörivät toistensa ympäri monta kierrosta sekunnissa, ollaan villissä tilanteessa. Aukkojen ajan hidastumisen vuoksi niiden pitäisi kiertää toistensa ympäri ääretöntä vauhtia. Tai jos ajatellaan "avaruusalus" joka pysyisi horisontin pinnassa niin sen näkökulmasta ympäröivä avaruus kiertäisi aukkoparia äärettömällä kierrosnopeudella äärettömän lyhyen hetken. Eikö tässä olla mahdottomassa todellisuudessa? Toisaalta fotoninkin todellisuus on mahdoton kun sillä ei ole aikaa, siis fotonikin värähtelee esim 10 potenssiin 30 värähdystä nollassa sekunnissa, sehän on mahdotonta.
Otsikko: Vs: Musta aukko ja sen muoto, tapahtumahorisontti
Kirjoitti: mistral - 24.04.2016, 15:08:36
Lainaus käyttäjältä: Joksa - 22.04.2016, 08:50:21
Schwarzildin aukon osalta on ollut helpompaa hahmotella pelkän gravitaation vaikutuksia, oikeiden fyysisten pyörivien aukkojen osalta on lisäksi huomioitava ne erilaiset pyörimisliikkeen aiheuttamat vaikutukset. Tutkijoille riiitää töitä ennenkuin kokonaisuus on hallinnassa.

Kun kaksi mustaa aukkoa kieppuu toistensa ympäri, jossain vaiheessa niiden tapahtumahorisontit yhtyy, ensin pienellä sillalla mutta kuitenkin ovat pysähtyneen ajan mielessä "yhtä puuta". Eli samoin kuin horisontit on jähmettyneet, on myös kieppuminen jähmettynyt. Tässä tullaan kummalliseen tilanteeseen, jos kieppuminen pysähtyy niin miksi yksinäisen mustan aukon liike avaruudessa ei pysähdy? Kyse on kuitenkin molemmissa tapauksissa liikkeestä, siis miksi pyörivää liikettä ei sallita mutta suoraviivainen sallitaan. Esimerkki suoraviivaisesta liikkeestä on kahden galaksin kolari. Kun molemmissa on todennäköisesti keskusaukko, niin niidenhän täytyy liikkua avaruudessa. Yhtäältä jos niiden aika on pysähtynyt, niiden ei pitäisi liikkua. Toisaalta koska galaksit liikkuu, pysyy aukot mukana vauhdissa ja näin liikkuvat. Eli teoria ja havainnot ei pidä yhtä...

Otsikko: Vs: Musta aukko ja sen muoto, tapahtumahorisontti
Kirjoitti: Eusa - 24.04.2016, 23:34:19
Lainaus käyttäjältä: mistral - 24.04.2016, 15:08:36
Kun kaksi mustaa aukkoa kieppuu toistensa ympäri, jossain vaiheessa niiden tapahtumahorisontit yhtyy, ensin pienellä sillalla mutta kuitenkin ovat pysähtyneen ajan mielessä "yhtä puuta". Eli samoin kuin horisontit on jähmettyneet, on myös kieppuminen jähmettynyt. Tässä tullaan kummalliseen tilanteeseen, jos kieppuminen pysähtyy niin miksi yksinäisen mustan aukon liike avaruudessa ei pysähdy? Kyse on kuitenkin molemmissa tapauksissa liikkeestä, siis miksi pyörivää liikettä ei sallita mutta suoraviivainen sallitaan. Esimerkki suoraviivaisesta liikkeestä on kahden galaksin kolari. Kun molemmissa on todennäköisesti keskusaukko, niin niidenhän täytyy liikkua avaruudessa. Yhtäältä jos niiden aika on pysähtynyt, niiden ei pitäisi liikkua. Toisaalta koska galaksit liikkuu, pysyy aukot mukana vauhdissa ja näin liikkuvat. Eli teoria ja havainnot ei pidä yhtä...
Ongelmasi on, että sekoitat havaintosuhteellisuuden ja avaruusajan keskenään. Se on valitettavan yleistä. Esimerkiksi monet ihmettelevät kuinka valo voi edetä, vaikka fotoni näyttäisi omassa kehyksessään olevan täysin ilman aikaa. Ratkaisu on, ettei fotoni hiukkasena etenekään, vain sen vaikutus siirtyy emissiosta absorptioon. Samoin voi ajatella mustasta aukosta. Se ei ole aineellista vaan vain eräänlainen avaruusajassa esiintyvä vaikutus.

Disclaimerina on heti omaan kvanttigravitaation tutkintaan perustuen todettava, ettei mielestäni tapahtumahorisontteja eli mustia aukkoja pääse syntymään tai jos syntyykin, ainetta tapahtumahorisontin sisälle ei jää. Singulariteetit eli avaruusajan epäjatkuvuuskohdat ovat kuin ovatkin lähinnä alkeishiukkasia. Ehkä eksoottisissa olosuhteissa voi syntyä erikosempia alkeishiukkasia ilman sisärakennetta... Mutta luonnollisesti näkemykseni, johon kuuluu kappaleiden pienetyminen ja etäisyyksien laajeneminen voimakkaassa gravitaatiokentässä, on vain yksi mielipide monien joukossa niin kauan kuin havainnot esim. neutronitähtien pinta-aloista puuttuvat.
Otsikko: Vs: Musta aukko ja sen muoto, tapahtumahorisontti
Kirjoitti: jussi_k_kojootti - 26.04.2016, 00:05:22
Lainaus käyttäjältä: Eusa - 24.04.2016, 23:34:19
Disclaimerina on heti omaan kvanttigravitaation tutkintaan perustuen todettava, ettei mielestäni tapahtumahorisontteja eli mustia aukkoja pääse syntymään tai jos syntyykin, ainetta tapahtumahorisontin sisälle ei jää.

No mutta jotain kai kuitenkin on jäänyt 10:n auringon massaiseen aukkoon tuplasti enemmän kuin 5:n auringon massaiseen aukkoon?
Otsikko: Vs: Musta aukko ja sen muoto, tapahtumahorisontti
Kirjoitti: Eusa - 26.04.2016, 00:37:26
Lainaus käyttäjältä: ketarax - 26.04.2016, 00:05:22
No mutta jotain kai kuitenkin on jäänyt 10:n auringon massaiseen aukkoon tuplasti enemmän kuin 5:n auringon massaiseen aukkoon?
Siis ei aukkoa, massaenergia on edelleen aineellisessa muodossa, esim. useina pikkuaukkoina eli alkeishiukkasina eli avaruusajan singulariteetteina. Mutta tämä on siis vain yksi työhypoteesi.

http://arxiv.org/pdf/hep-th/9202014v1
Otsikko: Vs: Musta aukko ja sen muoto, tapahtumahorisontti
Kirjoitti: Eusa - 26.04.2016, 15:42:44
Lainaus käyttäjältä: Joksa - 26.04.2016, 15:27:17
Tapahtumahorisontin poisto on melko rankka oletus, aukkoteoriahan lähti siitä liikkelle... Siis irtonaisten alkeishiukkasten pallo ilman tapahtumahorisonttia? Aikatekijä ilmeisesti ei myöskään mene nollaan saakka, vai? Vastaako sipulimallia 2 jossakin kehitysvaiheessa, siinä oli niitä käänteis-bb:n kerrostumia positiivisessa aikakehyksessä? Mitä tuo tai-jos vaihtoehto tarkoittaisi?
Luonnollisestikaan "tapahtumahorisontin poisto" ei ole ollut minulla oletuksena, vaan olen saanut sen tuloksena. Muutaman vuoden olen selvittänyt missä tein virheen ja varmastikin sen löydän, kunhan opiskelen syvemmin...

Aikatekijä ei mene millään hitaalla aineella nollaan koskaan. Tapahtumahorisonttihan on Schwarzschildin ratkaisun jatkokehitysversiossa juuri sitä varten, että aikaa kokevat ainekset voivat ajautua eri (havaittaviin) kaikkeuksiin. Alunperin sileä ratkaisu päättyi asymptoottiseen aukkoreunaan, mutta mittakaava säilyen päädyttiin kuitenkin "mielenkiintoisempaan" tulkintaan, jossa aineksien välinen horisontti on. Vastaavasti horisontti on otettu helppona käyttöön laajenevan kaikkeuden kosmologiassa, vaikka saattaisi olla itseensä kaareutumisen arvioinnin paikka sielläkin...
Otsikko: Vs: Musta aukko ja sen muoto, tapahtumahorisontti
Kirjoitti: mistral - 27.04.2016, 00:13:07
Lainaus käyttäjältä: Eusa - 24.04.2016, 23:34:19

Disclaimerina on heti omaan kvanttigravitaation tutkintaan perustuen todettava, ettei mielestäni tapahtumahorisontteja eli mustia aukkoja pääse syntymään tai jos syntyykin, ainetta tapahtumahorisontin sisälle ei jää.

Miksei voi ajatella niin että tapahtumahorisontti kasvaa ja alle jäävä materia jää horisontin sisäpuolelle? Eikö näin tapahdukin? Jos ei niin vapaasta avaruudesta katsottuna materia ei ainoastaan pysähdy vaan alkaa peruuttamaan meitä kohti kun horisontti kasvaa.

Jos teoriassa on "kieltosääntö" joka sanoo ettei materia voi saavuttaa horisonttia vapaan avaruuden ikuisuudessa, niin sitten on uskottava että se "peruuttaa" ulospäin.
Otsikko: Vs: Musta aukko ja sen muoto, tapahtumahorisontti
Kirjoitti: mistral - 26.05.2016, 00:30:03
T&A:n uutisessa "Hypoteesi: Muinaiset mustat aukot muodostavat valtaosan pimeästä aineesta", ja siitä lainaus:

"Tämä on yritys yhdistää laaja joukko ideoita ja havaintoja sekä testata, miten hyvin ne sopivat toisiinsa. Tulos on yllättävän hyvä", tiivistää Alexander Kashlinsky Nasan Goddardin avaruuslentokeskuksesta. "Mikäli tulos pitää paikkansa, kaikki galaksit, myös omamme, ovat mustien aukkojen muodostamien laajojen pallojen sisällä. Mustat aukot ovat massoiltaan karkeasti noin 30 kertaa Auringon kokoisia."


En ole kuullut että Linnunrata olisi "mustien aukkojen muodostamien laajojen pallojen sisällä". Onkohan tässä käännösvirhe?
Otsikko: Vs: Musta aukko ja sen muoto, tapahtumahorisontti
Kirjoitti: kvahlman - 26.05.2016, 08:57:09
Lainaus käyttäjältä: mistral - 26.05.2016, 00:30:03
T&A:n uutisessa "Hypoteesi: Muinaiset mustat aukot muodostavat valtaosan pimeästä aineesta", ja siitä lainaus:

"Tämä on yritys yhdistää laaja joukko ideoita ja havaintoja sekä testata, miten hyvin ne sopivat toisiinsa. Tulos on yllättävän hyvä", tiivistää Alexander Kashlinsky Nasan Goddardin avaruuslentokeskuksesta. "Mikäli tulos pitää paikkansa, kaikki galaksit, myös omamme, ovat mustien aukkojen muodostamien laajojen pallojen sisällä. Mustat aukot ovat massoiltaan karkeasti noin 30 kertaa Auringon kokoisia."


En ole kuullut että Linnunrata olisi "mustien aukkojen muodostamien laajojen pallojen sisällä". Onkohan tässä käännösvirhe?

Vaikuttaisi olevan ihan tarkka käännös kuten alta voi todeta. Ja miksei Linnunrata olisi jos muutkin galaksit ovat, ei tässä kai galaksikeskeistä maailmankuvaa ole syytä alkaa miettiä  :wink:

http://www.nasa.gov/feature/goddard/2016/nasa-scientist-suggests-possible-link-between-primordial-black-holes-and-dark-matter
Lainaa"If this is correct, then all galaxies, including our own, are embedded within a vast sphere of black holes each about 30 times the sun's mass."
Otsikko: Vs: Musta aukko ja sen muoto, tapahtumahorisontti
Kirjoitti: mistral - 26.05.2016, 12:26:30
Lainaus käyttäjältä: kvahlman - 26.05.2016, 08:57:09
Vaikuttaisi olevan ihan tarkka käännös kuten alta voi todeta. Ja miksei Linnunrata olisi jos muutkin galaksit ovat, ei tässä kai galaksikeskeistä maailmankuvaa ole syytä alkaa miettiä  :wink:

http://www.nasa.gov/feature/goddard/2016/nasa-scientist-suggests-possible-link-between-primordial-black-holes-and-dark-matter

Linnunradasta sanoin siksi koska se tunnetaan paremmin kuin muut galaksit.

Mutta ilmaisu "mustien aukkojen muodostamien laajojen pallojen sisällä" on vähän hämäävä koska ei ole itsestäänselvää että se viittaa verkostoon. Ensinnäkin pallopinta on pyöreä eikä ne mustat aukot tajua ryhmittyä pyöreään muotoon vaan likiarvoltaan pyöreään. Toiseksi on myös kosmologinen teoria jossa universumi olisi iso musta aukko, tähänkin voi sekoittaa "pallojen sisällä" ilmauksen.

Mutta itse teoria on mielenkiintoinen. Tosin Ligon havaitsema tapaus viittaa jättiläistähden romahdukseen, ei kahden valmiin mustan aukon törmäykseen. Jos olisi ollut valmiit aukot, ei gammapurkausta olisi pitänyt tulla. Gammateleskooppi Fermi mittasi alle sekunnin marginaalilla purkauksen, tuskin oli sattumaa.
Otsikko: Vs: Musta aukko ja sen muoto, tapahtumahorisontti
Kirjoitti: mistral - 26.05.2016, 18:21:18
Lainaus käyttäjältä: Joksa - 26.05.2016, 16:43:35
Onkohan laajat pallot muodostavien musta-aukkorykelmien mallinnettu erkanevan näkyvästä massasta galaksien törmätessä kuten pimeä aine ja mikähän selittää ilmiön?

Ovatkohan simulaatiolla päätyneet "pallomaiseen" muodostelmaan? Ei ainakaan tule mieleen miksi muodostelma syntyisi, siis mikä saisi alkurähdyksessä syntyneet mustat aukot ryhmittymään tuolla tavalla. Samojen taivaanmekaniikan lakien alla nekin ovat.
Otsikko: Vs: Musta aukko ja sen muoto, tapahtumahorisontti
Kirjoitti: kvahlman - 30.05.2016, 12:17:32
Varmaan myös kannattaa tarkistaa alkuperäisestä tutkimuksesta (mikäli mahdollista) että oliko pallomaisuus vain haastateltavan tapa ilmaista "mustien aukkojen ympäröimiä" vai oikeasti tutkimuksesta kummunnut oletus muodolle...
Otsikko: Vs: Musta aukko ja sen muoto, tapahtumahorisontti
Kirjoitti: mistral - 31.05.2016, 20:20:50
Kyllähän Ylekin uutisoi tästä joten asia on merkittävä ja uutta tietoa varmaan lähiaikoina tulee.

Toisaalta saattavat hiukkastutkijat löytääkin pimeän aineen hiukkasen, sekin on yksi vaihtoehto. Pimeää ainetta ei tarvitsisi olla paljoakaan maapallon tilavuudessa, muistaakseni n. 1kg jotta se selittäisi arvoituksen. Täytyy vielä yrittää tarkistaa tämä tieto jostain.

muok
Ei löytynyt vielä, jossain olen sen lukenut  :huh:
Otsikko: Vs: Musta aukko ja sen muoto, tapahtumahorisontti
Kirjoitti: kvahlman - 01.06.2016, 12:08:06
Yle uutisoi kyllä myös ahventohtorin sääennusteet, eli sillä perusteella ei varmaankaan kovin suuria johtopäätöksiä kannata tehdä  :evil:

Tiedemaailman näkökulmasta olisi varmaan lievä pettymys jos ei tarvitsisikaan löytää mystisiä pimeitä partikkeleita, mutta itseäni kyllä tälläinen selitys joka perustuu jo jotensakin ymmärrettyyn ilmiöön tyydyttäisi varsin hyvin. Toivottavasti tästä tehdään jonkunlaisia suorahkoja havaintojakin tulevaisuudessa.

P.S. taidettiin jo livetä ohi ketjun aiheen...
Otsikko: Vs: Musta aukko ja sen muoto, tapahtumahorisontti
Kirjoitti: Celest1al Sphere - 17.08.2016, 11:46:43
Lainaus käyttäjältä: kvahlman - 30.05.2016, 12:17:32
Varmaan myös kannattaa tarkistaa alkuperäisestä tutkimuksesta (mikäli mahdollista) että oliko pallomaisuus vain haastateltavan tapa ilmaista "mustien aukkojen ympäröimiä" vai oikeasti tutkimuksesta kummunnut oletus muodolle...

Itse olisin kääntänyt tämän kohdan:

"If this is correct, then all galaxies, including our own, are embedded within a vast sphere of black holes each about 30 times the sun's mass." 

seuraavasti:

"mikäli tämä pitää paikkansa, kaikkia galakseja, mukaanlukien meidän omamme, ympäröi valtava mustien aukkojen muodostama kehä (tai halo), jossa mustat aukot ovat massaltaan noin 30 auringon massaa".

Ehkä termi 'halo' olisi kaikkein osuvin. Ilmeisesti on niin, että nämä mustat aukot sijaitsevat Linnunradan haloa ympäröivässä pimeän aineen halossa, joka on valtavan paljon suurempi kuin Linnunrata (sen tavallisesta aineesta koostuva halo mukaanlukien). En lukenut juttua vielä, joten en osaa sanoa tarkemmin, tämä on lähinnä omaa pohdintaani.

/Juha
Otsikko: Vs: Musta aukko ja sen muoto, tapahtumahorisontti
Kirjoitti: mistral - 17.08.2016, 19:00:36
Lainaus käyttäjältä: Joksa - 17.08.2016, 09:43:59
Mielenkiintoinen uutinen jonka mukaan Hawkingin teoreettiset ajatelmat ovat saaneet vahvistusta simulaatioista
http://www.iltalehti.fi/ulkomaat/2016081622141010_ul.shtml

En vaan ymmärrä kuinka tapahtumahorisonttia voi simuloida, ehkäpä vain Hawkingin säteilyä haettiin ja löydettiin - mutta ilman gravitaation vaikutusta.
Otsikko: Vs: Musta aukko ja sen muoto, tapahtumahorisontti
Kirjoitti: mistral - 17.08.2016, 19:04:35
Lainaus käyttäjältä: Aldebaran - 17.08.2016, 11:46:43
seuraavasti:

"mikäli tämä pitää paikkansa, kaikkia galakseja, mukaanlukien meidän omamme, ympäröi valtava mustien aukkojen muodostama kehä (tai halo), jossa mustat aukot ovat massaltaan noin 30 auringon massaa".

Ehkä termi 'halo' olisi kaikkein osuvin.
/Juha

Varmaan tuo on se idea. Ja jos osoittautuu että siellä on tuo kehä tai halo, silloin ehkä ei pimeää ainetta ole olemassakaan.
Otsikko: Vs: Musta aukko ja sen muoto, tapahtumahorisontti
Kirjoitti: mistral - 19.08.2016, 11:29:32
Wiki selittää Hawkingin-säteilyä näin:

"Hawkingin säteily on hypoteettista säteilyä, jota oletetaan syntyvän mustan aukon tapahtumahorisontin läheisyydessä. Teorian kehitti Stephen Hawking vuonna 1974. Kyse on hiukkas-antihiukkaspareista, joita kutsutaan virtuaalihiukkasiksi, jotka ilmestyvät tyhjiöstä ja normaalisti annihiloituvat välittömästi – niin nopeasti etteivät ne edes ehdi rikkoa mitään luonnonlakeja olemassaolollaan. Kun hiukkaset syntyvät tapahtumahorisontin lähellä, hiukkasparin toinen hiukkanen voi joutua mustaan aukkoon ja luovuttaa parilleen hieman energiaa. Lisäenergiaa saanut hiukkanen puolestaan muuttuu reaaliseksi kumppaninsa kadottua ja pakenee mustan aukon ulottuvilta. Ulkopuolisesta tarkkailijasta näyttää siltä, että hiukkanen tulee mustasta aukosta ja musta aukko säteilisi.

Mustan aukon säteilyä ei ole havaittu. Tämä johtuu siitä, että tuntemamme mustat aukot ovat vain sellaisia mustia aukkoja, jotka ovat syntyneet tähden luhistuessa. Esimerkiksi muutaman Auringon massaisen mustan aukon säteily on vain asteen miljoonasosan päässä absoluuttisesta nollapisteestä. Koska kosminen taustasäteily on noin 2,7 kelviniä absoluuttisen nollapisteen yläpuolella, peittää se tehokkaasti suurten mustien aukkojen säteilyn."

Se mitä en ymmärrä, on hiukkasen ja antihiukkasen massaenergia. Eikö molemmilla ole sama massaenergia? Jos nyt hiukkanen pakee vapaaseen avaruuteen, kuinka antihiukkanen pienentäisi mustan massaa?

Lisäys.
Toki se on oikea ajatus että jos vakuumista poistuu energiaa, sinne jää vähemmän jäljelle. Mutta kuka todistaa että vakuumi syöksyy geodeesia pitkin horisontin sisäpuolelle?

Lisäys2:
Tässä Tiede.fi:stä keskustelua vuodelta 2010:



"Tässä artikkelin alku. Selittää vähän, mistä on kysymys.


Observation of quantum Hawking radiation andits entanglement in an analogue black hole

Jeff Steinhauer

We observe spontaneous Hawking radiation, stimulated by quantum vacuum fluctuations, emanating from an analogue black hole in an atomic Bose–Einstein condensate. Correlations are observed between the Hawking particles outside the black hole and the partner particles inside. These correlations indicate an approximately thermal distribution of Hawking radiation. We find that the high-energy pairs are entangled, while the low-energy pairs are not, within the reasonable assumption that excitations with dierent frequencies are not correlated. The entanglement verifies the quantum nature of the Hawking radiation. The results are consistent with a driven oscillation experiment and a numerical simulation.

Fifty years ago, Bekenstein discovered the field of black hole thermodynamics. This field has vast and deep implications, far beyond the physics of black holes themselves. The most

important prediction of the field is that of Hawking radiation. By making an approximation to the still-unknown laws of quantum gravity, Hawking predicted that the horizon of the black hole should emit a thermal distribution of particles. Furthermore, each Hawking particle should be entangled with a partner particle falling into the black hole. This presents a puzzle of information loss, and even the unitarity of quantum mechanics falls into question.

Despite the importance of black hole thermodynamics, there were no experimental results to provide guidance. The problem is that the Hawking radiation emanating from a real black hole should be exceedingly weak. To facilitate observation, Unruh suggested that an analogue black hole can be created in the laboratory, where sound plays the role of light, and the local flow velocity and speed of sound determine the metric of the analogue spacetime. Nevertheless, thermal Hawking radiation had never been observed before this work."


Otsikko: Vs: Musta aukko ja sen muoto, tapahtumahorisontti
Kirjoitti: mistral - 21.08.2016, 14:55:54
Jo pelkästään se että vakuumista ponnahtaa hiukkanen, on kummallista. Kukaan ei ole selittänyt onko se säilymislain mukaista, ja jos on, niin mikä on se vakuumin "astia" ja kuinka paljon energiaa se sisältää vaikkapa 1 kuutiometrissä?

Mutta itse Jeff Steinhauerin koe on kanssa hämärän peitossa, kuitenkin tuosta enkunkielisestä linkistä sain sen kuvan että kondensaatti kiihdytetään äänen nopeuteen ja ääniaalloilla sitten tehdään jotakin. Mahtaa olla kallis laitteisto jos kondensaatti on hyvin kylmää ja sen nopeus putkessa kiihdytetään lentokoneen(?) nopeuteen.
Otsikko: Vs: Musta aukko ja sen muoto, tapahtumahorisontti
Kirjoitti: mistral - 24.08.2016, 00:56:30
Lainaus käyttäjältä: Joksa - 21.08.2016, 18:08:15
Tyhjiöenergiasta http://www.kotiposti.net/ajnieminen/alku.pdf ja http://www.helsinki.fi/~enqvist/artikkeli.dir/dim_04.htm

Mikähän äänen nopeus tuollaisessa kondensaatissa lieneekään, ja onkohan sitten niin että itse kvanttifluktuaatioilmiö ei ollut mukana kokeessa..?

Ajniemisen jutusta lainaus:
Epätarkkuusperiaate sallii ns. "kvanttilainanoton".  Materiaa voi syntyä tyhjästä jos "energian lainaus" Ê maksetaan takaisin epämääräisyysajan ^T puitteissa. Suurikin energialainaus on siis mahdollista, jos "laina-aika" on hyvin pieni! Koko universumi on siis voinut syntyä tyhjästä massaenergiapisteestä, jonka painovoimakentällä on ollut yhtä suuri määrä vastakkaista (negatiivista) energiaa. Maailmankaikkeuden synnyssä ja kehityksessä pätee energian säilymislaki: kokonaisenergia Ekok on aina vakio. Universumin kokonaisenergia alussa oli nolla ja joka hetki senkin jälkeen nolla: Ekok=0. Näin siksi, että maailmankaikkeuden aineen määrä (10^50T) eli massanergia on positiivista, mutta gravitaatioenergia on negatiivista.


Tässä jutussa sanotaan  itseasiassa ETTEI TYHJIÖSSÄ OLE ENERGIAA. Eli alkuräjähdys olisi ollut vain kaavoilla kikkailua. Siis tyhjiön energiatase on nolla mutta epätarkkuusperiaatteella sieltä saadaan rakennettua meidän maailmankaikkeus.
Otsikko: Vs: Musta aukko ja sen muoto, tapahtumahorisontti
Kirjoitti: mistral - 25.08.2016, 00:21:16
Lainaus käyttäjältä: Joksa - 24.08.2016, 11:17:21
Eikö samaa logiikkaa käyttäen voisi päätellä myös niin ettei kosmoksessa ole energiaa... :rolleyes:

Jossain Hawkingin kirjassa oli tästä, eli että miinusmerkkisen ja plusmerkkisen energian summa olisi nolla. Käsittääkseni se vaatii kaikkien hiukkasten siirtymistä aika kauas naapurihiukkasesta jotta gravitaation miinusenergia saadaan nollattua. Tämä tarkoittaisi valtavan isoa taivaanpalloa mikä olisi alkuräjähdykseen verrattuna suorastaan vastakohta. Eli hyvin pieni taivaanpallo olisi alkutilana ja hyvin iso lopputilana. Mielestäni tätä ei voi kuitata olankohautuksella. Toki voidaan olettaa että tieteen ulottumattomissa on olemassa jokin "silta" joka yhdistäisi nämä vastakkaiset tilat, mutta kukaan ei tiedä mikä se on.
Otsikko: Vs: Musta aukko ja sen muoto, tapahtumahorisontti
Kirjoitti: jussi_k_kojootti - 25.08.2016, 01:50:10
Lainaus käyttäjältä: mistral - 19.08.2016, 11:29:32
Se mitä en ymmärrä, on hiukkasen ja antihiukkasen massaenergia. Eikö molemmilla ole sama massaenergia? Jos nyt hiukkanen pakee vapaaseen avaruuteen, kuinka antihiukkanen pienentäisi mustan massaa?

Tismalleen samalla tavalla, kuin jos kaadan maitotölkistä lasin täyteen, juon puolet ja kaadan puolet takaisin tölkkiin, on tölkistä vähentynyt puoli lasillista maitoa.

Lainaus käyttäjältä: Joksa - 21.08.2016, 11:28:20
Voisiko tuo energian luovutus selittää asiaa..?  Jos virtuaalipareista putoaakin se materiaalinen niin toimiiko luovutus ja miten päin..? Jos antihiukkanen annihiloituisi aukon sisällä niin syntyvä energia kai jäisi sinne, eli massaenergia ei muuttuisi.

Sillä kumpi parista putoaa aukkoon ei ole mitään merkitystä.  Energia on energiaa (mitä sitten lieneekään .. ai niin, kyky tehdä työtä :-)).

Annihilaatiossa vaikkapa
e+ + e- = gamma + gamma
eli
elektroni ja positroni muuttuu kahdeksi identtiseksi fotoniksi.  Identtinen tarkoittaa sitä, ettei ole mitään keinoa erottaa kumpi fotoneista tuli elektronista ja kumpi positronista.

Antimaterialla ei ole antimassaa, ainoastaan "antivaraus", siis vastakkaismerkkinen varaus.

Hawkingin säteilyssä ei ole kyse annihilaatiosta, vaan toinen yksinkertaisesti pääsee karkuun ja toinen ei.

Hawkingin säteilyssä ei myöskään oleteta, että prosessi (toinen karkuun, toinen ei) tapahtuu kaikille horisontin vierellä syntyville hiukkapareille.  Vaikka se yhtä aukkoa koskien tapahtuisi yhdelle parille koko universumin olemassaolon aikana, puhuttaisiin sittenkin mustien aukkojen säteilevän.

Sillä mitä tapahtumahorisontin sisällä tapahtuu ei ole mitattavissa olevaa merkitystä aukon ulkopuolella (eli, ei ole merkitystä Hawkingin säteilylle).

Lainaa
Muodostaisikohan tuo E-B kondensaatti vastaavan rajapinnan kuin horisontti, eli siihen osuva virtuaalihiukkanen ei voisikaan enää reagoida parinsa kanssa?

Juuri siitä kyseisessä lähestymistavassa on kyse.  Jos analogisella aukolla ei ole (analogista) tapahtumahorisonttia, niin mikäs aukko se sitten edes olisi olevinaan.

Siinä muutamia "koulukirjatotuuksia" aiheen tiimoilta, toivottavasti niistä on apua, tai ei ainakaan haittaa.  En lukenut muita viestejä.


Otsikko: Vs: Musta aukko ja sen muoto, tapahtumahorisontti
Kirjoitti: Eusa - 25.08.2016, 05:12:03
Siis eiköhän Hawkingin säteilyssä ajatella gravitaatiokentän menettävän energiaa tekemällä tuon hiukkasparin. Kun toinen hiukkanen karkaa, gravitoivaa massaa on vähentynyt juuri sen hiukkasen massan verran.

Tuntuisi, että vielä tehokkaammin massaa vähenisi, jos nuo hiukkaset annihiloituisivat säteilyksi saman tien...
Otsikko: Vs: Musta aukko ja sen muoto, tapahtumahorisontti
Kirjoitti: mistral - 25.08.2016, 17:19:10
Lainaus käyttäjältä: ketarax - 25.08.2016, 01:50:10
Tismalleen samalla tavalla, kuin jos kaadan maitotölkistä lasin täyteen, juon puolet ja kaadan puolet takaisin tölkkiin, on tölkistä vähentynyt puoli lasillista maitoa.


Toki näin jos kyse olisi mustan aukon massasta mutta kun kyse on vakuumista jonka roolia ma:n massanmuodostukseen ei tiedetä. Vai tiedetäänkö että vakuumikin on gravitaation vaikutuksesta ma:n vankina ja näin sieltä paennut fotoni keventäisi myös ma:n massaa? Jos vakuumi ei reagoi gravitaatioon, voi se seilata minne haluaa ja se kahden fotonin kevennys vaikuttaa siellä minne suuntaan sattuu seilaamaan.
Otsikko: Vs: Musta aukko ja sen muoto, tapahtumahorisontti
Kirjoitti: Eusa - 25.08.2016, 18:33:07
https://profmattstrassler.com/articles-and-posts/relativity-space-astronomy-and-cosmology/black-holes/black-hole-information-paradox-an-introduction/

Mustien aukkojen mysteerissä on vielä paljon spekuloitavaa. Itse olen päätynyt siihen, etteivät ne ole mustia vaan hämäriä. Energiatiheys lähestyy itseriittoista tilaa, eikä säteilyä riitä omista tarpeista enää juuri hämärän massiivin ulkopuolelle. Tapahtumahorisontin muodostumisen voi estää ns. miniatyrisoituminen eli tardis-ilmiö "suurempi sisältä kuin ulkoa" eli vaikka ainetta ja massaa on paljon koordinaatistoavaruudessa (ulkopuolisen havainto), tila on itseiskoordinaatistossa laajentunut. Vastaava idea on ns. gravastar, jossa massiivin rakenteessa on pimeällä energia merkittävä osuus, se ei kuitenkaan ota kantaa, kuten minä, että varsinaisesti säteilymekanismi aiheuttaisi tilan laajenemista eli olisi se pimeä energia.

Kuten edellä kirjoitin, hiukkaspari saa energiansa hämärän massiivin (mustan aukon) pienenemisestä, minkä jälkeen aukko voi siepata hiukkasen/hiukkaset riippuen niiden liikemääristä tai säteillä pois.
Voisikohan ajatella, että syntyvän hiukkasparin heti annihiloituessa ja sm-säteilyn suuntautuessa kiertoradan tangentin suunnassa, koko massiivin massafluktuaation sallima energiaemissio voisi säteillä massiivin ulottumattomiin? Marginaalispekulaatio.
Otsikko: Vs: Musta aukko ja sen muoto, tapahtumahorisontti
Kirjoitti: jussi_k_kojootti - 25.08.2016, 18:33:38
Lainaus käyttäjältä: mistral - 25.08.2016, 17:19:10
Toki näin jos kyse olisi mustan aukon massasta mutta kun kyse on vakuumista jonka roolia ma:n massanmuodostukseen ei tiedetä. Vai tiedetäänkö että vakuumikin on gravitaation vaikutuksesta ma:n vankina ja näin sieltä paennut fotoni keventäisi myös ma:n massaa? Jos vakuumi ei reagoi gravitaatioon, voi se seilata minne haluaa ja se kahden fotonin kevennys vaikuttaa siellä minne suuntaan sattuu seilaamaan.

Nyt täytyy todeta, että sikäli kun olen kuullut hyviä tarinoita, on Hawkingin säteilyn selitys virtuaalipareilla itse asiassa kansantajuistus.  Alkuperäisen idean matematiikka tarkastelee vakuumin olemusta eri havaitsijoiden vinkkelistä (havaitsijat voivat olla eri mieltä siitä mikä tila (state) itseasiassa vastaa vakuumia), ja näiden havaitsijoiden (suhteellinen) aika on esityksessä vahvasti mukana.  En ole itse perehtynyt Hawkingin säteilyyn tuota kansantajuista versiota syvällisemmin, joten en nyt osaa sanoa yksityiskohdista oikein juuta tai jaata.

MUTTA, minun mielestä vakuumi (eli aika-avaruus) on toki ma:n vankina aivan kuten ketjussa ylempänä koetin tolkuttaa; eli siinä mielessä että horisontin sisäpuolella kaikki tiet vie singulariteettiin, tai kuten silloin sanoin, "aika-avaruus-kin putoaa singulariteettiin".  Ja samoin vakuumi, eli aika-avaruus, reagoi gravitaatioon -- eli kaareutuu massaenergian vaikutuksesta.

Edit:
Se että virtuaaliparikuva on "kansantajuistus" ei tarkoita että se olisi hyödytön, tai edes automaattisesti väärä selitys.  Päin vastoin, se on hyödyllinen apuväline, mutta siihen liittyvät rajoitukset/reunaehdot tulee tuntea viimeistään sillä hetkellä, kun yritetään mennä pidemmälle.  Vähän niinkuin GR:n kumikalvoanalogia -- mitä erinomaisin tapa havainnollistaa ja käsitellä tiettyjä, moniakin GR:n piirteitä, mutta ei pidä olettaa että analogia on yleispätevä, tai että sen pohjalta esim. voi vaivatta selittää, saatika ennustaa, muitakin GR:n ilmiöitä.
Otsikko: Vs: Musta aukko ja sen muoto, tapahtumahorisontti
Kirjoitti: mistral - 25.08.2016, 22:26:15
Lainaus käyttäjältä: ketarax - 25.08.2016, 18:33:38

MUTTA, minun mielestä vakuumi (eli aika-avaruus) on toki ma:n vankina aivan kuten ketjussa ylempänä koetin tolkuttaa; eli siinä mielessä että horisontin sisäpuolella kaikki tiet vie singulariteettiin, tai kuten silloin sanoin, "aika-avaruus-kin putoaa singulariteettiin".  Ja samoin vakuumi, eli aika-avaruus, reagoi gravitaatioon -- eli kaareutuu massaenergian vaikutuksesta.


Itse olen ajatellut aika-avaruutta immuunina gravitaatiolle, siis siinä mielessä että se lähtisi "liukumaan" jonnekin. Sensijaan pidän sitä "liukumäkenä" jossa massaenergia liukuu (putoaa vapaasti) eli mäki pysyy paikallaan. Ja vakuumia olen pitänyt täydellisenä tyhjiönä paitsi sen poreilua joka antaa(?) sille jonkin massaenergia-arvon. Jos poreilu on luonteeltaan sellaista että siinä on yhtä paljon plus ja miinus energiaa, voiko silloin mennä takuuseen että sen massaenergia-arvo poikkeaa nollasta? Ja jos se ei poikkea nollasta, silloin gravitaatio ei pure siihen. Ainakaan mitään varmaa arvoa vakuumin energiasisällöstä ei olla onnistuttu mittaamaan.
Otsikko: Vs: Musta aukko ja sen muoto, tapahtumahorisontti
Kirjoitti: mistral - 30.08.2016, 23:15:13
Lainaus käyttäjältä: Joksa - 30.08.2016, 15:43:40
Tyhjiöenergia näyttäisi olevan rajaton energialähde.

On vaikea sulattaa sitä että kaavasta saadaan energiaa, sen takanahan pitäisi olla todellista olevaista. Eikö tyhjiöenergia ole vain kvanttiheilahteluun perustuva mekanismi, ikäänkuin "idea" eikä sen olemus ole selvillä. Jos sen olemus tunnettaisiin, saattaisi osoittautua ettei sieltä heru yhtään energiaa. No riippuu tietysti olemuksesta...
Otsikko: Vs: Musta aukko ja sen muoto, tapahtumahorisontti
Kirjoitti: mistral - 31.08.2016, 18:44:08
Lainaus Enqvistin jutusta:

"Poreilun energia täyttää koko avaruuden ja toimii siis kosmologisena vakiona. Efekti voidaan havaita ns. Casimir-ilmiönä. Siinä kahden metallilevyn väliin ilmestyy pienoinen mutta mitattava näennäinen vetovoima, joka johtuu siitä, että levyjen väliin syntyvien virtuaalisten hiukkasten aallonpituus on rajoitettu. Levyjen etäisyyttä suurempia aallonpituuksia ei yksinkertaisesti mahdu niiden väliin. Sen sijaan niiden ulkopuolella kaikenlaiset virtuaalihiukkaset ovat mahdollisia. Seurauksena on levyjä ulkoapäin yhteen työntävä paine. Tyhjiön poreilun seurauksena syntyvän energiatiheyden suuruus on valtaisa. Esimerkiksi protonin rakenneosasia kvarkkeja kuvaavassa kvanttikromodynamiikassa saisimme kosmologisen vakion, jonka suuruus olisi 10^{76} kertaa sallittua suurempi. Tämän energiatiheyden katoaminen on eräs mysteereistä, jonka gravitaation ja kvanttifysiikan yhdistävän teorian odotetaan joskus selvittävän."


En löydä selitystä työntövoimalle poreilusta. Casimir-ilmiö on vain todiste poreilusta mutta ei luulisi sen puskevan partikkeleita niin voimakkaasti että se kumoaisi gravitaation, vai onko kyse siitä että suurilla etäisyyksillä g. on niin heikko ettei se enää voita poreilua?

Lisäys:
Jos Casimir-voima puskee avaruutta, tullaan uuteen ongelmaan mikä ollaan huomattu, eli tyhjästä syntyy energiaa mikä on vaikea ongelma. Mielellään näkisi jonkin mekanismin jonka kautta Casimir-voima saa kyvyn tehdä työtä. Jos se löytyisi, olisi koko teoria vahvemmalla pohjalla.
Otsikko: Vs: Musta aukko ja sen muoto, tapahtumahorisontti
Kirjoitti: Eusa - 31.08.2016, 23:24:53
Lainaus käyttäjältä: mistral - 31.08.2016, 18:44:08
Lainaus Enqvistin jutusta:

"Poreilun energia täyttää koko avaruuden ja toimii siis kosmologisena vakiona. Efekti voidaan havaita ns. Casimir-ilmiönä. Siinä kahden metallilevyn väliin ilmestyy pienoinen mutta mitattava näennäinen vetovoima, joka johtuu siitä, että levyjen väliin syntyvien virtuaalisten hiukkasten aallonpituus on rajoitettu. Levyjen etäisyyttä suurempia aallonpituuksia ei yksinkertaisesti mahdu niiden väliin. Sen sijaan niiden ulkopuolella kaikenlaiset virtuaalihiukkaset ovat mahdollisia. Seurauksena on levyjä ulkoapäin yhteen työntävä paine. Tyhjiön poreilun seurauksena syntyvän energiatiheyden suuruus on valtaisa. Esimerkiksi protonin rakenneosasia kvarkkeja kuvaavassa kvanttikromodynamiikassa saisimme kosmologisen vakion, jonka suuruus olisi 10^{76} kertaa sallittua suurempi. Tämän energiatiheyden katoaminen on eräs mysteereistä, jonka gravitaation ja kvanttifysiikan yhdistävän teorian odotetaan joskus selvittävän."


En löydä selitystä työntövoimalle poreilusta. Casimir-ilmiö on vain todiste poreilusta mutta ei luulisi sen puskevan partikkeleita niin voimakkaasti että se kumoaisi gravitaation, vai onko kyse siitä että suurilla etäisyyksillä g. on niin heikko ettei se enää voita poreilua?

Lisäys:
Jos Casimir-voima puskee avaruutta, tullaan uuteen ongelmaan mikä ollaan huomattu, eli tyhjästä syntyy energiaa mikä on vaikea ongelma. Mielellään näkisi jonkin mekanismin jonka kautta Casimir-voima saa kyvyn tehdä työtä. Jos se löytyisi, olisi koko teoria vahvemmalla pohjalla.
Minun ehdotukseni on, että säteily ei pelkästään punasiirry avaruuden laajenemisesta, vaan avaruusajan aaltoilun energiaa siirtyy tyhjöenergiaksi, joka näkyy laajenemisena ja punasiirtymänä, yhteisvaikutus.
Otsikko: Vs: Musta aukko ja sen muoto, tapahtumahorisontti
Kirjoitti: mistral - 01.09.2016, 23:43:06
Lainaus käyttäjältä: Eusa - 31.08.2016, 23:24:53
Minun ehdotukseni on, että säteily ei pelkästään punasiirry avaruuden laajenemisesta, vaan avaruusajan aaltoilun energiaa siirtyy tyhjöenergiaksi, joka näkyy laajenemisena ja punasiirtymänä, yhteisvaikutus.

Tarkoitatko että gravitaatioaaltojen energia muuttuisi Casimir-voimaksi?


Tuli tässä iltasella mieleen Casimir-voimasta sellainen asia että sehän vaikuttaa myös kappaleen toisella puolella ja näin kumoutuu jolloin nettovoima olisi tasan nolla. Sama periaate vaikuttaa sukellusveneeseen, vaikka veden paine olisi 100 baaria, pysyy se paikallaan siksi koska paine vaikuttaa kaikista suunnista.

Eli en nyt ymmärrä, kuinka Casimir-voima pystyisi kiihdyttämään taivaankappaleita...
Otsikko: Vs: Musta aukko ja sen muoto, tapahtumahorisontti
Kirjoitti: mistral - 02.09.2016, 15:11:33
Kosmologiassa puhutaan avaruuden laajenemisesta, ei välttämättä siitä että kappaleet leviää laajemmalle alueelle vaan että tyhjiökin laajenisi. Tässä ajatuksessa on vaan katkos tapahtumaketjussa.

Jos alkutilanteessa pidetään tyhjiöenergiaa moottorina laajenemiselle, ensimmäinen katkos tulee siitä, kuinka tyhjiöenergia esim lisää avaruutta Linnunradan ja M101:n välille. Kukaan ei osaa selittää sitä.

Toinen katkos tulee siitä, kuinka tyhjä avaruus voisi hinata kappaleita mukanaan. Oletetaan vaan että on olemassa näkymättömät siimat jotka "kiinnittyvät" avaruuden koordinaatistoon. Koko pimeän energian hypoteesi perustuu tähän, siis siimojen tekemään työhön (tai puskemiseen, mikä sitten onkaan se mekanismi, olennaista on pimeän energian määrä).

Eli hypoteesi perustuu siihen että tunnetaan lähtökohta ja lopputulos (tyhjiöenergia ja kiihtyvä laajeneminen), ja näiden väliin jäävät asiat on tuntemattomia.

Ja tähän hypoteesiin ilmeisesti perustuu ajatus siitä että avaruuden laajeneminen voi ylittää valon nopeuden, mitä ei kuitenkaan voida havaita koska se menee meidän havaintohorisontin ulkopuolle.

Lisäys: Tarkistin M101:n speksejä ja löytyi:
Redshift: 0.000804
Helio radial velocity: 241 ± 2 kilometres per second

....joten olettaisin että se menee meistä poispäin. Näin uutta avaruutta tulisi meidän ja M101:n välille. Tämä esimerkki ei kerro koko totuutta mutta on suuntaa antava. Mutta toinen juttu on siis se, syntyykö uutta avaruutta ylipäätään.
Otsikko: Vs: Musta aukko ja sen muoto, tapahtumahorisontti
Kirjoitti: mistral - 03.09.2016, 00:54:43
Lainaus käyttäjältä: Joksa - 02.09.2016, 17:54:44
Voisiko systeemi olla analoginen jonkilaiseen waterworld-kompleksiin, eli a-a olisi eri tasoilla olevaa altaistoa, vakuumi vettä ja aine vedessä kelluvia partikkeleita..? Jos (joku) allas laajenee niin siinä olevat partikkelit eivät ui vastavirtaan pitääkseen ent sijaintinsa vaan kelluvat pinnan laajemisliikkeen mukana ja siten erkaantuvat toisistaan.

Epäilen. Pimeän energian määrä lasketaan käsittääkseni maailmankaikkeuden massasta jolloin jokaisen kappaleen kiihdyttämiseen tarvitaan sitä energiaa. Nyt jos vesitankissa kappaleet kiihtyvät ikäänkuin siivellä, silloin niiden osuus olisi aika pieni kokonaisenergiasta. Tämä mekanismi siis on tuntematon, tiedossa on vain lähtötilanne ja arvio nykytilanteesta, sekin on vain arvio. Mitä siinä välissä on tapahtunut, on mysteeri, ehkä salaisuus jonain päivänä selviää mutta tänään avaruuden laajeneminen on vain käsite - ei konkreettinen asia joka osataan selittää. Tai ainakin ne Ursan kirjat ym mitä olen lukenut, ei selitä sitä. Tilanne ratkeaisi jos Saul Perlmutterin ja toisaalta Brian Schmidtin havainnot osoittautuisivat vääriksi tulkinnoiksi, silloin pimeää energiaa ei tarvittaisikaan.

Lisäys:
Analogia vedessä uivista kappaleista pitää sisällään samanpainoisuuden ja paine-erot virran myötä- ja vastapuolella, eli vaikkapa "uppotukki" virrassa kiihtyessään virran nopeuteen, aiheuttaa ylipaineen toiselle puolelle, alipaineen vastakkaiselle puolelle. Tämä ei oikein toimi vakuumiin, jos sen nopeus/lepotila on sama havaitsijan liiketilasta riippumatta (valon nopeus on sama havaitsijan liiketilasta riippumatta). Ja jos vakuumilla olisikin nopeutta, silloinkin tulee ongelmia. Esim joku asteroidi jonka massa on miljoona tonnia niin sillä on joku pinta-ala. Toinen asteroidi painaisi miljardi tonnia mikä on 1000x. Pinta-alaero olisi vain 100x. Näin pienempi kiihtyisi 10x nopeammin.

Toki paikallinen gravitaatio peittäisi näin pienet Casimir-voimat eikä kiihtyvyyserot näkyisi käytännössä. Ehkä laboratoriossa ollaankin mitattu, onko C-voima sama joka suunnassa, jos on, silloin ilmeisesti vakuumi ei liiku havaitsijaan nähden...
Otsikko: Vs: Musta aukko ja sen muoto, tapahtumahorisontti
Kirjoitti: mistral - 04.09.2016, 15:42:00
Avaruuden laajenemisen geometrinen kuvaus ei ole itselleni mitenkään selvä. Oikotie kuvaukseen on vissiin se että muutetaan mittakaavaa. Jos kaavaa muutetaan suuntaan tai toiseen, kappaleiden suhteelliset etäisyydet toisiinsa nähden pysyvät samoina, myös kulmat pysyvät samoina. Tämä vissiin on tasalaatuisuuden vaatimus, mikä on hyvin ymmärrettävää kosmoksen alun kannalta. Jos jo alussa olisi poikettu tasalaatuisuudesta, olisi tietyt alueet romahtaneet kasaan "alkumetreillä" ja tietyt jääneet lähes tyhjiksi. Eli, tasalaatuisuuden vaatimus on tärkeä.

Joku on sanonut ettei alkuräjähdys ollut verrattavissa dynamiittipanokseen, en sitten tiedä pitääkö paikkansa. Jos tonni dynamiittia vietäisiin Lagrangen pisteeseen ja räjäytettäisiin, olisi mielenkiintoista mitata, onko avaruus vaikkapa 1000km säteellä tasalaatuinen eli onko jokainen kaasumolekyyli samalla suhteellisella etäisyydellä ja samassa kulmassa toisiinsa nähden kuin räjähdyksen alkuvaiheessa (tässä on oiottu mutkia suoraksi).

Jokatapauksessa, oli geometria millainen tahansa, fyysikot on laskeneet että materiaa joudutaan kiihdyttämään ja ehkä meihinkin kohdistuu tietty kiihdytys koko ajan vaikka sitä olisi mahdoton mitata. Tai sitten olemme universumin keskipisteessä eikä lähiseudulla ole mitään kiihtyvyyttä. Tai kolmantena, mitään laajenemiskiihtyvyyttä ei sitten alkuräjähdyksen ole ollutkaan, siis jos Perlmutter/Schmidt-tulkinta on väärä.

Olisi hienoa jos maailmalta tai Suomesta tulisi kosmologiakirja joka havainnollistaisi avaruuden laajenemisen problematiikkaa ja eri teorioita. Vai onko ne "ammattisalaisuuksia"?
Otsikko: Vs: Musta aukko ja sen muoto, tapahtumahorisontti
Kirjoitti: mistral - 05.09.2016, 22:16:05
Tosiaan tuokin teoria on ollut esillä, en vaan muistanut sitä. Vähän kummalta vaan tuntuu se että Hawkingin säteilyssä on selvästi positiivista energiaa ja nyt mennäänkin toiseen ääripäähän ja väitetään että vakuumissa olisi negatiivista energiaa. Näköjään kosmologiassa mennään tuntemattomien alueiden yli tehdään niistä ad hoc tulkintoja eli "tarkoitusta varten" tulkintoja. Ja tässäkin tulkinnassa on se ongelma että jos vakuumi puskee materiaa, puskee se sitä molemmilta puolilta jolloin nettovaikutus on nolla. Ainoastaan universumin reunalla nettovaikutus voi poiketa nollasta, siis jos esim 10 metrin päässä ei olekaan vakuumia vaan "täydellinen tyhjiö".
Otsikko: Vs: Musta aukko ja sen muoto, tapahtumahorisontti
Kirjoitti: jussi_k_kojootti - 06.09.2016, 10:53:48
Lainaus käyttäjältä: mistral - 05.09.2016, 22:16:05
Näköjään kosmologiassa mennään tuntemattomien alueiden yli tehdään niistä ad hoc tulkintoja eli "tarkoitusta varten" tulkintoja.

Koetan tässä kohtaa pysäyttää sut katseellani, edes hetkeksi:  :rolleyes:
Otsikko: Vs: Musta aukko ja sen muoto, tapahtumahorisontti
Kirjoitti: mistral - 06.09.2016, 12:30:58
Lainaus käyttäjältä: ketarax - 06.09.2016, 10:53:48
Koetan tässä kohtaa pysäyttää sut katseellani, edes hetkeksi:  :rolleyes:

Negatiivinen energia kyllä ilmenee gravitaatiokaivossa, ei se käsitteenä ole tuntematon. Mutta jos vakuumin energia on negatiivista, kuinka siitä saataisiin fotoneja? Hawkingin säteily on fotoneita joten eikö hän silloin ymmärrä vakuumia eri tavalla kuin ne kosmologit jotka sanoo että vakuumi on miinusmerkkinen? Siis eikö Hawking pidä vakuumia neutraalina josta kvanttiporeilun takia voitaisiin siepata plusmerkkistä energiaa?
Otsikko: Vs: Musta aukko ja sen muoto, tapahtumahorisontti
Kirjoitti: Kaizu - 06.09.2016, 12:47:19
Lainaus käyttäjältä: mistral - 06.09.2016, 12:30:58
Hawkingin säteily on fotoneita ...
Hawkingin säteily on mitä tahansa hiukkasia joiden virtuaalipari on siirtynyt tapahtumahorisontin sisäpuolelle ja toinen jäänyt ulkopuolelle.

Kaizu
Otsikko: Vs: Musta aukko ja sen muoto, tapahtumahorisontti
Kirjoitti: mistral - 07.09.2016, 00:56:39
Lainaus käyttäjältä: Kaizu - 06.09.2016, 12:47:19
Hawkingin säteily on mitä tahansa hiukkasia joiden virtuaalipari on siirtynyt tapahtumahorisontin sisäpuolelle ja toinen jäänyt ulkopuolelle.

Kaizu

En ole tuota ajatellutkaan, voi hyvin olla. Mutta kaikki ne hiukkaset on normaalia energiaa, ei eksoottista "negatiivista" energiaa. Täytyy vielä lukea Joksan linkki kunhan ehdin.
Otsikko: Vs: Musta aukko ja sen muoto, tapahtumahorisontti
Kirjoitti: mistral - 07.09.2016, 23:54:10
Lainaus käyttäjältä: Joksa - 06.09.2016, 12:36:46


Kyseessä on negattiivinen paine ja sen aiheuttama (negatiivinen) vaikutus gravitaatioon.

Jos tuota rinnastetaan ilmanpaineeseen, niin vaikka hehkulampussa oleva negatiivinen paine on suhteellista painetta. Avaruudessa hehkulampun alipaine lakkaa, siellä ollaan absoluuttisessa tyhjiössä. Kuinka mennä siitä eteenpäin, on outoa, etumerkin muuttaminen plussasta miinukseksi vaatii kvalitatiivisen muutoksen.
Otsikko: Vs: Musta aukko ja sen muoto, tapahtumahorisontti
Kirjoitti: mistral - 09.09.2016, 13:16:56
Lainaus käyttäjältä: Joksa - 08.09.2016, 16:10:56
Negatiivisen paineen olemuksesta ei tunnu löytyvän aineistoa.

Yksinkertainen ajatuskoe: jos 0-paineessa olevaa suljettua tilaa laajentaa hieman niin syntyykö tilaan väistämättä negatiivinen paine... ja jos ei niin miten?

Jos puhutaan vaikka ilmanpaineesta, ei synny. 
Esim ilmajousitetun auton maavara pysyy kaasun atomien törmäysnopeuden voimalla tietyllä korkeudella. Pakkasella nopeus laskee ja maavarakin laskee, tosin automatiikkaa pumppaa lisäilmaa ja korjaa tason oikeaksi. Eli laskenut nopeus kompensoidaan lisäämällä atomien määrää jolloin kumipalkeeseen saadaan sama voima kuin kesälläkin.
Jos vaikka pneumaattinen sylinteri painetaan kasaan, tulpataan liitokset ja sitten yritetään vetää auki, on vedon voima sama joka kohdassa iskupituutta, eli tässä ei tilavuuden lisääminen suurenna alipainetta.

Kiinteällä aineella en tiedä miten on. Esim 5mm teräsvaijerin (löysin vanhasta Maritimin luettelosta) murtolujuus on 2310kg. Jos vaijeria kuormitetaan 2300 kilolla, on siinä ainakin näennäinen negatiivinen paine. Raudan atomien residuaalinen sm-voima on rajalla antaa periksi, ehkä tämä sitten on negatiivista painetta?

Mitä sitten olisi vakuumin negatiivinen paine, ei aavistustakaan.



Otsikko: Vs: Musta aukko ja sen muoto, tapahtumahorisontti
Kirjoitti: Lauri Kangas - 09.09.2016, 13:35:14
Lainaus käyttäjältä: mistral - 09.09.2016, 13:16:56
Esim 5mm teräsvaijerin (löysin vanhasta Maritimin luettelosta) murtolujuus on 2310kg. Jos vaijeria kuormitetaan 2300 kilolla, on siinä ainakin näennäinen negatiivinen paine.

Miten tämä näennäinen negatiivinen paine ilmenee?

Onko lujuusopin suure nimeltä jännitys tuttu?
Otsikko: Vs: Musta aukko ja sen muoto, tapahtumahorisontti
Kirjoitti: pnuu - 09.09.2016, 15:03:56
Lainaus käyttäjältä: mistral - 09.09.2016, 13:16:56
Jos vaikka pneumaattinen sylinteri painetaan kasaan, tulpataan liitokset ja sitten yritetään vetää auki, on vedon voima sama joka kohdassa iskupituutta, eli tässä ei tilavuuden lisääminen suurenna alipainetta.

Tämä on helppo todeta vääräksi vaikkapa lääkeruiskun tai fillarin pumpun avulla. Purista ruutta melkein tyhjäksi, pistä sormi tukkeeksi, ja ala vetää mäntää ulos. Mitä pidemmälle sitä vedät, sitä suuremman voiman tarvitset. Ja miksikö? No koska paine sylinterissä alenee. Seuraa vähän lukio-fysiikkaa:

F = pA

Jotta voima (F) pysyisi vakiona, pitää paineen (p) ja poikkipinta-alojen (A) tulon pysyä vakiona. Muuttuuko sylinterin poikkipinta-ala? Ei, joten painekaan ei voisi muuttua. Mutta jospa kuitenkin käytettävän voiman määrä muuttuu, mitäs sitten? Silloin, kuten juuri todettiin, pitää paineen (p) muuttua.

pV = NRT = vakio

- N (ainemäärä) ei muutu
- R on vakio

-> pV/T = vakio

Jos yksi muuttuu, pitää muiden kompensoida. Tarkastellaanpa vaikka tilavuuden kaksinkertaistumista, miten kompensoivat suureet voivat muuttua, jos vain toinen hoitaa kompensoimisen:

1) paine puolittuu: 1000 hPa -> 500 hPa
2) lämpötila kaksinkertaistuu: 293.15 K (20 °C) -> 586.3 K (293.15 °C).

Tässä ääripäät. Miten kävi, suliko käyttämäsi lääkeruisku/pumppu?
Otsikko: Vs: Musta aukko ja sen muoto, tapahtumahorisontti
Kirjoitti: mistral - 09.09.2016, 15:17:29
Lainaus käyttäjältä: Lauri Kangas - 09.09.2016, 13:35:14
Miten tämä näennäinen negatiivinen paine ilmenee?

Onko lujuusopin suure nimeltä jännitys tuttu?

Ajattelin vaan että jännitys=alipaine. Mutta lauseessani oli kysymysmerkki, jolla tahdoin sanoa etten tiedä varmuudella.

Havainnollistaen jos vaikka 10cm teräskuulaan hitsattaisiin 100 tankoa kaikkiin suuntiin ja kaikilla vedettäisiin lähes murtorajalle, jäisi kuulan ytimeen negatiivinen paine, vastaava tilanne kuin vaijeriin. Verrattuna kaasunpaineeseen, kyse on kanssa sm-voimasta, kuulan tapauksessa vetävästä sm-voimasta. Eli jos työntävä on pluspainetta, vetävä olisi miinuspainetta.

Gravitaatiossa en vaan ymmärrä kuinka nollapaine ohitetaan, sm-voimassa se on peruskauraa. Gravitaatiossa vedon muuttaminen työnnöksi taitaa olla vain olla hypoteesi.
Otsikko: Vs: Musta aukko ja sen muoto, tapahtumahorisontti
Kirjoitti: mistral - 09.09.2016, 15:27:53
Lainaus käyttäjältä: pnuu - 09.09.2016, 15:03:56
Tämä on helppo todeta vääräksi vaikkapa lääkeruiskun tai fillarin pumpun avulla. Purista ruutta melkein tyhjäksi, pistä sormi tukkeeksi, ja ala vetää mäntää ulos. Mitä pidemmälle sitä vedät, sitä suuremman voiman tarvitset. Ja miksikö? No koska paine sylinterissä alenee.

Esimerkkini oli ideaalinen, ts pneumaattinen sylinteri oli täysin tyhjä. Kun mäntää sitten aletaan vetää ulospäin, pommittaa männän ulkopintaa normaali ilmanpaine. Sisäpintaa pommittaa nolla atomia joten tilanne pysyy samana koko iskupituudelta; sisällä ei-mitään, ulkona normaali ilmakehä. Noin havainnollistuksena, jos männän pinta-ala on 100cm^2 ja bodari vetää mäntää suoraan ylöspäin, silloin bodari nostaa männän päällä olevaa 100kg:n ilmapatsasta ylöspäin.
Otsikko: Vs: Musta aukko ja sen muoto, tapahtumahorisontti
Kirjoitti: Eusa - 09.09.2016, 17:39:31
Hm. Jos analogisoi, eikö gravitaatio ole mieluumminkin alipainetta ja pimeä energia ylipainetta?
Otsikko: Vs: Musta aukko ja sen muoto, tapahtumahorisontti
Kirjoitti: mistral - 09.09.2016, 21:42:01
Lainaus käyttäjältä: Eusa - 09.09.2016, 17:39:31
Hm. Jos analogisoi, eikö gravitaatio ole mieluumminkin alipainetta ja pimeä energia ylipainetta?

Se negatiivinen paine tulee ymmärtääkseni tästä: massaenergia tekee gravitaation------> antimassaenergia tekee antigravitaation. Jotta antigravitaatio eli puskeva voima saadaan aikaan, tarvitaan jokin ihmeellinen kenttä tai vastaava jonka energiasisältö on miinusmerkkinen ja vakuumilla olisi sitten tämä ominaisuus. En nyt ole varma asiasta mutta siis muuttamalla gravitaation etumerkki saataisiin pusku aikaan.
Otsikko: Vs: Musta aukko ja sen muoto, tapahtumahorisontti
Kirjoitti: Eusa - 09.09.2016, 23:34:19
Lainaus käyttäjältä: mistral - 09.09.2016, 21:42:01
Se negatiivinen paine tulee ymmärtääkseni tästä: massaenergia tekee gravitaation------> antimassaenergia tekee antigravitaation. Jotta antigravitaatio eli puskeva voima saadaan aikaan, tarvitaan jokin ihmeellinen kenttä tai vastaava jonka energiasisältö on miinusmerkkinen ja vakuumilla olisi sitten tämä ominaisuus. En nyt ole varma asiasta mutta siis muuttamalla gravitaation etumerkki saataisiin pusku aikaan.
Luulenpa, että asia on huomattavasti havainnollisempaa käsittää, kun gravitaatio tulkitaan tilan vähenemiseksi ja pimeä energia tilan lisääntymiseksi kuin, että niitä ajateltaisiin voimina.

Massa vähentää tilaa. Mikä olisi antimassaa, joka lisää tilaa? Sopisiko valo ja neutriinot siihen rooliin?
Otsikko: Vs: Musta aukko ja sen muoto, tapahtumahorisontti
Kirjoitti: mistral - 10.09.2016, 14:53:41
Lainaus käyttäjältä: Eusa - 09.09.2016, 23:34:19
Luulenpa, että asia on huomattavasti havainnollisempaa käsittää, kun gravitaatio tulkitaan tilan vähenemiseksi ja pimeä energia tilan lisääntymiseksi kuin, että niitä ajateltaisiin voimina.

Massa vähentää tilaa. Mikä olisi antimassaa, joka lisää tilaa? Sopisiko valo ja neutriinot siihen rooliin?

Valo ja neutriinot on tavallaan myös massaa. Antimassa on vain selitysyritys, aika näyttää onko sitä olemassakaan.
Otsikko: Vs: Musta aukko ja sen muoto, tapahtumahorisontti
Kirjoitti: Eusa - 11.09.2016, 00:34:50
Lainaus käyttäjältä: mistral - 10.09.2016, 14:53:41
Valo ja neutriinot on tavallaan myös massaa. Antimassa on vain selitysyritys, aika näyttää onko sitä olemassakaan.
Lähinnä ajattelin, että yleinen punasiirtymä olisi sitä antimassaa, säteilyenergia vähenee avaruusajan geometrialle.
Otsikko: Vs: Musta aukko ja sen muoto, tapahtumahorisontti
Kirjoitti: pnuu - 11.09.2016, 00:45:13
Lainaus käyttäjältä: Eusa - 11.09.2016, 00:34:50
punasiirtymä olisi sitä antimassaa

Et sattumoisin ole koskaan kuullut Dopplerista?

Lainaa
säteilyenergia vähenee avaruusajan geometrialle.

Sama matemaattisesti, kiitos. Tai suomeksi :-)
Otsikko: Vs: Musta aukko ja sen muoto, tapahtumahorisontti
Kirjoitti: Eusa - 11.09.2016, 09:44:01
Matemaattisesti. Laajentunut avaruus tarkoittaa lievää potentiaalienergialisää, punasiirtynyt säteily lievää energiakatoa. Jospa 1-1=0.
Otsikko: Vs: Musta aukko ja sen muoto, tapahtumahorisontti
Kirjoitti: mistral - 13.09.2016, 22:42:10
Lainasin pätkän tähän aiheeseen liittyvää:


3 PIMEÄ ENERGIA
11
3  Pimeä energia
SCDM-mallissa maailmankaikkeuden energiasisältö koostuu säteilystä ja epärelativis-
tisesta aineesta. Aineen ja säteilyn paineet ja energiatiheydet ovat positiivisia, joten
kiihtyvyysyhtälön (7) mukaan skaalatekijän
a
toinen aikaderivaatta on negatiivinen, eli
maailmankaikkeuden laajeneminen SCDM-mallissa on hidastuvaa. Tämä ei kuitenkaan
vastaa kosmologisia havaintoja, joiden mukaan laajeneminen kiihtyy.
Ongelma voidaan ratkaista ottamalla mukaan uusi energiatyyppi, pimeä energia

Pai-
neen on oltava negatiivinen ja niin iso, että se riittää kumoamaan positiivisen energiati-
heyden kiihtyvyysyhtälössä.
________________________________________

Mielestäni tässä ei kerrota paineesta sen enempää, mistä tulee sellainen kuva ettei siitä tiedetä sen enempää. Negatiivinen paine olisi vaan "työnimi" tekijälle jota tarvitaan kaavassa, vai miten tuo pitäisi ymmärtää?
Otsikko: Vs: Musta aukko ja sen muoto, tapahtumahorisontti
Kirjoitti: mistral - 18.09.2016, 18:48:42
Uusimmassa T&A:ssa no 6 sivulla 36 on pieni juttu Jeff Steinhauerin kokeesta, lainaus:

"Elokuussa israelilainen tutkija J.S. kertoi rakentaneensa maanpäällisen version mustasta aukosta omassa laboratoriossaan ja mitanneensa siitä Hawkingin säteilyä.
Hurjalta kuulostava koe koostuu laserilla kiihdytetyistä superkylmistä atomeista, jotka lopulta kulkevat putkessa ääntä nopeammin. Atomeissa tapahtuu kvanttiväreilyä, joka synnyttää niin kutsuttuja fononeja eli äänikvantteja. Ne syntyvät aina pareittain. Kun fononipari syntyy rajalla, jonka toisella puolella atomien liike muuttuu ääntä nopeammaksi, vain toinen fononeista pystyy karkaamaan toisen huuhtoutuessa atomien virran mukaan.
Rajapinta käyttäytyy täsmälleen samoin kuin oikean mustan aukon tapahtumahorisontti. Kun siinä syntyy valohiukkasten eli fotonien pari, toinen niistä voi päästä säteilemään pois. Tätä kutsutaan Hawkingin säteilyksi."

Wikipediasta kuvaus fononista:
Fononi

Fononi on kiteen värähtelyenergian kvantti.[1] Fononeihin liittyvät ilmiöt ovat tärkeä osa tiiviin aineen fysiikkaa, koska ne vaikuttavat materiaalien fysikaalisiin ominaisuuksiin kuten lämmönjohtavuuteen ja sähkönjohtavuuteen. Lämmön johtuminen eristeessä johtuu fononien etenemisesta ja matalataajuiset fononit ovat äänen kvantittuneita värähtelyjä kiinteässä aineessa.

Klassisen mekaniikan mukaan kiteen värähtely voidaan hajottaa eri värähtelykomponenttien superpositioksi Fourier'n muunnoksen avulla. Kun näitä eri värähtelytiloja tarkastellaan kvanttimekaniikan mallein niissä ilmenee hiukkasominaisuuksia. Värähtelyn energiatilojen miehitys voidaan siksi kuvata kiteessä liikkuvilla hiukkasilla, joita kutsutaan fononeiksi. Fononilla ei kuitenkan ole varsinaista liikemäärää, sillä se ilmaisee atomien suhteellista liikettä eikä atomien massakeskipisteiden liikettä.[1] Hiukkasina fononit käyttäytyvät kuten bosonit.


Hmm, jos fononit vaatii kiteen, niin kuinka kaasussa silloin voi olla fononeita? 

Muokkaus:
Ilmeisesti Wikipedian kuvaus fononista on vajavainen, eli varmaan kaasussa on osittain samat mekanismit kuin kiteessäkin, perimmältään kaasussa kyse on työntövoimasta joka kohdistuu massallisiin hiukkasiin - ei "massattomiin" fotoneihin.

muokkaus2:
Löytyi Jeff Steinhauerin video aiheesta:
https://video.search.yahoo.com/search/video?fr=yfp-t-s&p=jeff+steinhauer#id=3&vid=fe8bb0afd657512888a16b96b549d57b&action=view

Menee kyllä yli ymmärryksen.

Äänen nopeus JS:n kokeessa olikin vain 1mm/sekunnissa! Mikä johtuu kylmyydestä ym. "Pieni" virhe kun alussa oletin että se on lentokoneen nopeus, siis virhe oli "vain" miljoonakertainen  :rolleyes:
Otsikko: Vs: Musta aukko ja sen muoto, tapahtumahorisontti
Kirjoitti: mistral - 20.09.2016, 20:39:24
Olisikohan väärinkäsitys kun sanotaan että JS:n kokeessa olisi tullut Hawkingin säteilyä. Jospa JS tarkoittaa että fononit vain vastaa Hawkingin säteilyä? Se on kyllä varmaan totta että ne kylmät atomit tai mitälienee, kaasua kuitenkin, niin niissä ilmenee pientä kvanttiheilahtelua ja näin moottorina toimii kvanttimaailma. Mutta fononi ei ole fotoni joten voidaanko silloin puhua säteilystä? Varsinkin tässä tapauksessa kun fononi etenee 1mm sekunnissa, fotoni taas 300 000 000 000mm sekunnissa.
Otsikko: Vs: Musta aukko ja sen muoto, tapahtumahorisontti
Kirjoitti: mistral - 21.09.2016, 00:43:00
Lainaus täältä:

http://www.nature.com/articles/nphys3863.epdf?referrer_access_token=RVR2FBZKFwLjC6ZUjO_fntRgN0jAjWel9jnR3ZoTv0NCXzqnWChN_xeBarCrCpSclZLb2F1Ccbtl91ELxPv9svmF6uLLgQaRxwznk8nD4IszWAOhzxiMvBNK01Vj9xmTk-gs5fW-pAaRWxSrUxtIndu4FTes-FINN9MaFA5x8wkAIzJunpWu6YqOl6iiZm-7&tracking_referrer=www.bbc.com

Observation of Hawking radiation

Figure 4a shows the measured correlation function between pairs of points (x, x') along the analog black hole. This correlation function is computed from an ensemble of 4600 repetitions of the experiment, requiring six days continuous measurement....

Kyllä sinänsä töitä on tehty mutta oikeaan mustaan aukkoon verrattuna ollaan hirmu kaukana siitä.

Esim oletus siitä että vakuumi saavuttaa valon nopeuden horisontissa, niin onko se fakta?
Otsikko: Vs: Musta aukko ja sen muoto, tapahtumahorisontti
Kirjoitti: mistral - 24.09.2016, 01:39:23
Lainaus käyttäjältä: Joksa - 22.09.2016, 00:00:08
Jos fononin käytöllä on haettu fotoni-analogiaa niin fotonillahan ei ole mitään antifotonia eli se ei ole niitä kvanttiflutuaatiossa syntyviä aine-antiainepareja joista Hawking-säteily syntyy. Fotoni syntyy yksin ilman mitään antiparia mutta fononit ilmeisti syntyvät pareittain, eli lämpökvantti siirtyy yhteen suuntaa ja vastaava lämmön pudotus samalla vastakkaiseen, mikä tehnee siitä simulaatioon soveliaan energiakvantin.

Käsittääkseni fotonilla on toinen fotoni parina vaikka niitä nimitetään virtuaalifotoneiksi. Kaksi tarvitaan vissiin siksi ettei rikota liikemäärän säilymislakia, eli kun vastakkaisiin suuntiin menee samanlaiset hiukkaset, on niiden yhteenlaskettu liikemäärä nolla.

Myös vaatimus valon nopeudesta karsii muut hiukkaset pois, jäljelle jää vain fotonit joiden pakonopeus riittää pakoon horisontin pinnasta.

Mutta en sitten tiedä onko nämä vakuumin virtuaalifotonit samoja kuin esim kestomagneetissa olevat välittäjähiukkaset, eikös kestomagneetin voima mene tiiviinkin materiaalin läpi. Kokeilin että meni 5mm paksun lehden läpi eli sähkömagneettisen vuorovaikutuksen välittäjähiukkanen läpäisi sen, tavallinen valo ei mene sen läpi. Eli jos kestomagneetin kenttä ulottuu jopa äärettömyyteen, matkustaa välittäjähiukkanen valon nopeudella pitkän matkan mutta vakuumin virtuaalifotoni on "eri laatuinen" koska sen matka katkeaa melkein heti alkuunsa.
Otsikko: Vs: Musta aukko ja sen muoto, tapahtumahorisontti
Kirjoitti: jussi_k_kojootti - 24.09.2016, 18:14:37
Lainaus käyttäjältä: Joksa - 24.09.2016, 10:21:19
Jos horisontissa syntyvän hiukkasparin energia tulkitaan olevan mustan aukon energiaa ja osa siitä muuttuu foton(e)iksi ja pääsee pakenemaan niin silloin ajatus toimisi, mutta eikös se ole ihan tulkintajuttu onko rajan tyhjiöenergia nimeomaan m-a:n energiaa?

Tyhjiö on tyhjiötä.  Sen kihinästä syntyy virtuaalinen (hetkellinen) hiukkaspari, joka mustan aukon gravitaatiokentän energialla välttääkin "normaalin" katoamisensa.  Toisin sanoen siitä tulee -- aukon energialla -- oikea hiukkaspari.  Näistä toinen aukkoon ja toinen kaukaisuuteen, jolloin aukon massa on pienentynyt hiukkasen.


Lainaa
Ja miten ulkopuolelle syntynyt virtuaalihiukkanen muuttuisi fotoniksi, annihiloitumallako jonkun muun hiukkasen kuin oman parinsa kanssa?

Pitääkö sen muuttua fotoniksi?
Otsikko: Vs: Musta aukko ja sen muoto, tapahtumahorisontti
Kirjoitti: mistral - 24.09.2016, 23:32:49
Tuolla magneettijutulla yritin sanoa kuinka arkisissa asioissa virtuaalifotoneja voi epäsuorasti havaita. Rupesin vielä tutkimaan mitä netti sanoo niistä ja ilmeisesti joudun perumaan puheitani, magneettikentän kaukovaikutus ehkä ei sittenkään perustu virtuaalifotoneihin. Ei se voi perustua tavallisiinkaan fotoneihin koska kestomagneetin pitäisi silloin "tyhjentyä" koska tavalliset fotonit kuluttaa energiaa. En sitten tiedä miten vuorovaikutus tapahtuu jos siinä ei ole virtuaali- tai tavallisia fotoneja, kaareutuuko siinä sähkömagneettinen avaruus vastaavalla tavalla kuin gravitaatiossakin?
Otsikko: Vs: Musta aukko ja sen muoto, tapahtumahorisontti
Kirjoitti: mistral - 25.09.2016, 00:50:57
Lainaus käyttäjältä: ketarax - 24.09.2016, 18:14:37
Tyhjiö on tyhjiötä.  Sen kihinästä syntyy virtuaalinen (hetkellinen) hiukkaspari, joka mustan aukon gravitaatiokentän energialla välttääkin "normaalin" katoamisensa.  Toisin sanoen siitä tulee -- aukon energialla -- oikea hiukkaspari.  Näistä toinen aukkoon ja toinen kaukaisuuteen, jolloin aukon massa on pienentynyt hiukkasen.

Ymmärrän kyllä mitä sanot mutta jos Hawkingin säteilyn edellytys on vakuumin putoaminen valon nopeudella horisontin läpi, niin silloin hiukkaset tulee ulkoa sisään eikä ole ma:n omaa tuotantoa. Eikö nyt ainoa keino pienentää ma:n massaa ole se että vangiksi jäänyt hiukkanen olisi massaenergialtaan negatiivinen? Jotain tällaista Jeff Steinhauer selittikin, olikohan se niin että kun fotonin aallonpituus horisontissa menisi nollaan, niin sen sisäpuolella taas alkaisi kasvaa, mutta miinusmerkkisenä, en tajunnut sitä alkuunkaan. Tässä aiempi linkki:
https://video.search.yahoo.com/search/video?fr=yfp-t-s&p=jeff+steinhauer#id=3&vid=fe8bb0afd657512888a16b96b549d57b&action=view

Hawkingin säteilyssä on toinenkin kummallisuus (ensimmäinen oli negatiivinen massaenergia), koordinaatisto on yhtä aikaa vakuumin sisäinen nollanopeus eli vakuumissa etenevät poreilut on lepokoordinaatistossa ja lähiympäristö on aika levollinen, vähän niin kuin koskenlaskija koskessa toteaa että samaa vauhtia mennään kuin vesikin tosin kuplia riittää  :azn:  Mutta toisaalta on koordinaatisto jonka suhteen vakuumi syöksyy valon nopeudella ja vedotaan tähän koordinaatistoon jotta H-säteily on mahdollista.
Eli lepokoordinaatisto ei kelpaa yksinään. Jos se riittäisi, palautuisivat virtuaalihiukkaset ihan samalla lailla vakuumiin kuin ovat sieltä ponnahtaneet esiin - riippumatta siitä, ollaanko vapaassa avaruudessa tai horisontin pinnassa.



[/quote]
Otsikko: Vs: Musta aukko ja sen muoto, tapahtumahorisontti
Kirjoitti: Eusa - 25.09.2016, 04:56:05
Tyhjön syöksymisen nopeus on ristiriitainen käsite, koska ei ole olemassa mitään vertailukoordinaatistoa.

Voidaan ajatella, että massan aiheuttama tyhjövaje täyttyy kausaalisesti eli jos tyhjön syöksyminen olisi havaittavissa suoraan, sen nopeus olisi valonnopeus c aina, määritelmällisesti. Tyhjöenergian gradientti, eli paljonko tyhjöenergiaa siirtyy poikkipinta-alaa kohti, olisi määriteltävissä suoraan tapahtumajatkumon eli avaruusajan kaarevuuden mukaan.

Tyhjöenergiagravitaation ja -inertian mallissa massakappaleet tarvitsevat massan määräämällä teholla tyhjöenergiaa. Säteittäisesti syöksyvän tyhjöenergian gravitaatiokentässä tuo tarkoittaa, että tyhjöenergiaa on saatavissa tiheimmin gravitaatiosuunnan puolella ja kappale tyhjöenergiaa ottaessaan siirtyy kiihtyen siihen suuntaan.
Otsikko: Vs: Musta aukko ja sen muoto, tapahtumahorisontti
Kirjoitti: mistral - 25.09.2016, 20:15:23
Lainaus käyttäjältä: Eusa - 25.09.2016, 04:56:05
Tyhjön syöksymisen nopeus on ristiriitainen käsite, koska ei ole olemassa mitään vertailukoordinaatistoa.


Vertailukoordinaatistona olisi vain avaruuden kaarevuus. Steinhauerin analogia valon nopeuden ja äänen nopeuden välillä tarkoittaa että vakuumilla on horisontissa valon nopeus. Miksi näin, en tiedä.

Ja eikös vakuumin nopeuden olettaminen joksikin, merkitse myös eetterin olemassaolon olettamista?
Otsikko: Vs: Musta aukko ja sen muoto, tapahtumahorisontti
Kirjoitti: Eusa - 26.09.2016, 00:16:12
Lainaus käyttäjältä: mistral - 25.09.2016, 20:15:23
Vertailukoordinaatistona olisi vain avaruuden kaarevuus. Steinhauerin analogia valon nopeuden ja äänen nopeuden välillä tarkoittaa että vakuumilla on horisontissa valon nopeus. Miksi näin, en tiedä.

Ja eikös vakuumin nopeuden olettaminen joksikin, merkitse myös eetterin olemassaolon olettamista?
Kyllä - kaikissa muissa tapauksissa paitsi, jos tyhjö oletetaan kaikkialla valonnopeuteen c. Eri nopeuksilla toisiinsa suhtautuvat kappaleet olisivat kuitenkin suhteessa tyhjöön kuitenkin kaikki samassa nopeudessa. Tuollainen tyhjö ei itse kokisi aikaa, Diracin ajattomuus, josta ne magneettiset monopolinsakin voisivat löytyä...
Otsikko: Vs: Musta aukko ja sen muoto, tapahtumahorisontti
Kirjoitti: Eusa - 26.09.2016, 11:10:30
Hawkingin ratkaisussa mustan aukon gravitaatiokenttä fluktuoi niin, että hiukkaspari saa varastettua energiaa muodostuakseen. Aukko siis pienee tuossa vaiheessa. Kun hiukkaspari annihiloituu, toinen säteilysuunta on aukkoon ja toinen aukosta ulos. Tuossa vaiheessa aukko taas suurenee säteilykvantin verran, mutta koska toinen säteilykvantti karkasi, Hawkingin säteilyn vaikutuksesta aukko jäi hieman pienemmäksi kuin ennen sitä.

Harmittaa, kun fysiikan viralliset ulostulot ovat niin kryptisiä, ettei teille harrastajille useinkaan muodostu järkevää käsitystä tapahtumien kulusta.  :angry:
Otsikko: Vs: Musta aukko ja sen muoto, tapahtumahorisontti
Kirjoitti: Eusa - 26.09.2016, 13:08:01
Lainaus käyttäjältä: Joksa - 26.09.2016, 12:36:09
"gravitaatiokenttä fluktuoi"? Eikös sen tee vakuumi, ei siihen tavallisesti mainita tarvittavaan gravitaatiokentältä energiaa.
Halusin korostaa, että energia nimeomaan otetaan keskusmassasta. Konsensusilmaisu lienee "tapahtumahorisontin epätarkkuus sallii tyhjöfluktuaation tapahtua mustan aukon energialla kuten tunnelointi yleisesti kvanttimekaniikassa".

Tuo käsitys on tosin vanhentunut. Tällä hetkellä moni tutkija ajattelee, että jyrkkä kaarevuus sinänsä riittää Hawkingin säteilyn ilmiöön ja onpa tutkimuksia, joissa todetaan, että loogisella tilastotarkastelulla mekanismi on tähden romahtaessa niin laajaa ja tehokasta, ettei tapahtumahorisonttia koskaan muodostu.
Otsikko: Vs: Musta aukko ja sen muoto, tapahtumahorisontti
Kirjoitti: Eusa - 26.09.2016, 17:54:20
Lainaus käyttäjältä: Joksa - 26.09.2016, 13:34:21
Ok, tämä selvensi asiaa himan, mutta ei selitä vielä miksi tuo epätarkkuus olisi tilastollisesti siten vinoutunut että vastaavassa määrin aukon ulkopuolisen energian fluktuoimista hiukkaspareista ei voisi toinen jäädä aukon loukkuun ja toinen paeta. Eikös epätarkkuuden puitteissa 'sama' horisontilla oleva piste voi välillä ottaa ma:n energiaa välillä ulkopuolisen vakuumin, tasapuolisesti..?
Selvästi aukon ulkopuoliset tapahtumat eivät liity Hawkingin säteilyyn mitenkään. Sieltähän syöksyy kaikenlaista. Lähdetään siitä, että ulkopuolista energiaa ei vaikuta aiheeseen - silloin aukko haihtuu pikku hiljaa kiihtyen.

Vihillä olet siinä, että ihan massakeskittymän omasta energiasta fluktuoituu tilastollisesti niin paljon Hawkingin säteilyä, että tähden romahtaminen mustaksi aukoksi riittävän nopeasti on kyseenalaista. Nyttemmin ajatellaan, että alkuperäisessä vain tapahtumahorisontin tuntumaan keskittyvässä mallissa on juurikin "tilastollista vinoutumaa" ihan muodostuvan aukon itsensä energiaympäristössä.
Otsikko: Vs: Musta aukko ja sen muoto, tapahtumahorisontti
Kirjoitti: mistral - 26.09.2016, 20:43:23
Lainaus käyttäjältä: Eusa - 26.09.2016, 13:08:01
Halusin korostaa, että energia nimeomaan otetaan keskusmassasta. Konsensusilmaisu lienee "tapahtumahorisontin epätarkkuus sallii tyhjöfluktuaation tapahtua mustan aukon energialla kuten tunnelointi yleisesti kvanttimekaniikassa".


Jos tunnelointi tuottaa poreilua horisontin ulkopuolelle niin sitten se selittäisi yhden asian. On kuitenkin toinen asia mikä ei selviä, itse Hawkingin mekanismi. Käsittääkseni vakuumissa poreileva hiukkaspari elää vain Planckin ajan. Wikin lainaus:
"Planckin aika on ajan luonnollinen yksikkö, jota merkitään tP. Se on nimetty Max Planckin mukaan. Planckin aika on se ajanjakso, joka valon nopeudella kulkevalta fotonilta kuluu Planckin pituuden mittaiseen matkaan,[1] ja sen suuruus on...."
Ja Planckin pituus on:
"10−35 metriä ja on yksi Planckin yksiköistä. Planckin pituus voidaan laskea käyttämällä kolmea luonnonvakiota: valonnopeutta, Planckin vakiota ja gravitaatiovakiota"

Tosi pienestä ajasta ja pituudesta puhutaan. Kysymys kuuluu, mikä ero on hiukkasparin poreella horisontin ulkopuolella verrattuna siihen että sama tapahtuu horisontin pinnassa? Eli kuinka tilanne eroaa siinä mielessä etteivät virtuaalihiukkaset pääsekään palaamaan yhteen?
Otsikko: Vs: Musta aukko ja sen muoto, tapahtumahorisontti
Kirjoitti: Eusa - 26.09.2016, 21:15:53
Lainaus käyttäjältä: mistral - 26.09.2016, 20:43:23
Jos tunnelointi tuottaa poreilua horisontin ulkopuolelle niin sitten se selittäisi yhden asian. On kuitenkin toinen asia mikä ei selviä, itse Hawkingin mekanismi. Käsittääkseni vakuumissa poreileva hiukkaspari elää vain Planckin ajan. Wikin lainaus:
"Planckin aika on ajan luonnollinen yksikkö, jota merkitään tP. Se on nimetty Max Planckin mukaan. Planckin aika on se ajanjakso, joka valon nopeudella kulkevalta fotonilta kuluu Planckin pituuden mittaiseen matkaan,[1] ja sen suuruus on...."
Ja Planckin pituus on:
"10−35 metriä ja on yksi Planckin yksiköistä. Planckin pituus voidaan laskea käyttämällä kolmea luonnonvakiota: valonnopeutta, Planckin vakiota ja gravitaatiovakiota"

Tosi pienestä ajasta ja pituudesta puhutaan. Kysymys kuuluu, mikä ero on hiukkasparin poreella horisontin ulkopuolella verrattuna siihen että sama tapahtuu horisontin pinnassa? Eli kuinka tilanne eroaa siinä mielessä etteivät virtuaalihiukkaset pääsekään palaamaan yhteen?
Eroavat hiukkasethan voivat annihiloitua (ja varmaankin lähes kaikki annihiloituvatkin) ilman, että Hawkingin mekanismissa muuttuu mikään. Annihilaatiossa kvantit lähtevät aina tasan vastakkaisiin suuntiin ja silloin toisen reitti kulkee vääjäämättä kohti massiivikeskusta ja toisen siitä ohi.

Kuten sanoin, horisonttia ei pidetä nykyisin Hawkingin säteilylle mitenkään tarpeellisena, vaan sitä katsotaan tapahtuvan vain sitä enemmän, kuta jyrkemmäksi gravitaatiokenttä käy.
Otsikko: Vs: Musta aukko ja sen muoto, tapahtumahorisontti
Kirjoitti: Eusa - 26.09.2016, 22:38:23
Lainaus käyttäjältä: Joksa - 26.09.2016, 21:26:22
Fluktuaatio jossa toinen hiukkasosapuoli putoaa aukkoon ja toinen pakenee ei kai ole selvästi ma:n ulkoinen tapahtuma olipa fluktuaation energia peräisin kummalta puolelta tahansa. Molempien huomioinnista seuraisi että Hawkingin säteilyn määrä tuplaantuisi mutta vaikutus ma:n massaan neutraloituisi. Jälkimmäinen on melko pienimerkityksinen täsmennnys, koska höyrystyminen olisi kuitenkin universuminkin elinkaareen verrattuna pitkäaikainen juttu mutta säteilytehon tuplaantumisen vaikutus ma:n sisäiseeen rakenteeseen ja dynamiikkaan voisi olla oleellinen.

Jos Hawking on jättänyt asian kokonaan huomiotta niin se saattaisi olla hänen pahin munauksensa... :rolleyes:
Ahaa, nyt huomasin mistä kenkä puristaa.

Massahan ei ole vain massapiste siellä jossain, vaan koko gravitaatiokenttä on ainerakennetta, massan manifestaatiota, massakenttä. Jos kentässä tapahtuu jotain energeettistä, gravitaatiomuutos signaloi potentiaalin muutoksen kaikkialle ja yhteinen massa muuttuu. Gravitaatiokentän fluktuointi tarkoittaa siis uuden massan muodostuessa negatiivista massaa ympäristöönsä. Analogiana voisi mainita keskihakuisuuden tai kiihtyvyyden yleensä. (keskihaku-)kiihtyvyyden lisääntyessä suljetun järjestelmän muut kiihtyvyydet vähenevät. Esimerkiksi, jos kaksi vauhtipyörää kulmakiihdyttävät toisiaan, niiden sidokset (sidosvoimt/-kiihtyvyydet) venyvät ja voivat pettääkin...

Ei ole merkitystä missä massallinen aine sijaitsee. Jos se voi vastaanottaa tietoa potentiaaliasemansa muutoksesta, sen inertia voi vähentyä tai lisääntyä riippuen kumpaa tietoa saapuu.
Otsikko: Vs: Musta aukko ja sen muoto, tapahtumahorisontti
Kirjoitti: mistral - 27.09.2016, 21:32:44
Lainaus käyttäjältä: Eusa - 26.09.2016, 21:15:53
Eroavat hiukkasethan voivat annihiloitua (ja varmaankin lähes kaikki annihiloituvatkin) ilman, että Hawkingin mekanismissa muuttuu mikään. Annihilaatiossa kvantit lähtevät aina tasan vastakkaisiin suuntiin ja silloin toisen reitti kulkee vääjäämättä kohti massiivikeskusta ja toisen siitä ohi.

Kuten sanoin, horisonttia ei pidetä nykyisin Hawkingin säteilylle mitenkään tarpeellisena, vaan sitä katsotaan tapahtuvan vain sitä enemmän, kuta jyrkemmäksi gravitaatiokenttä käy.

Mistä hiukkaspari tietää suuntautua niin että toinen menee prikulleen kohti ma:n ydintä tai prikulleen ulospäin? Veikkaan että poreen tai miksi sitä sanotaan, kun pore alkaa muistuttaa hiukkasta, gravitaatio "saa otteen" siitä ja taivuttaa muodostuvaa hiukkasta kuin tuuli aallonharjaa.


Mutta tämä on yllättävää jos "horisonttia ei pidetä nykyisin Hawkingin säteilylle tarpeellisena". Näin itsekin ajattelen, tai oikeastaan ajattelen ettei koko säteilyä ole olemassakaan.
Mutta jos tällaista ajattelua nykyään on, on JS:n koe vanhaa ajattelua, siinä nimenomaan mainitaan horisontti.
Otsikko: Vs: Musta aukko ja sen muoto, tapahtumahorisontti
Kirjoitti: Eusa - 27.09.2016, 22:40:31
Lainaus käyttäjältä: mistral - 27.09.2016, 21:32:44
Mistä hiukkaspari tietää suuntautua niin että toinen menee prikulleen kohti ma:n ydintä tai prikulleen ulospäin? Veikkaan että poreen tai miksi sitä sanotaan, kun pore alkaa muistuttaa hiukkasta, gravitaatio "saa otteen" siitä ja taivuttaa muodostuvaa hiukkasta kuin tuuli aallonharjaa.


Mutta tämä on yllättävää jos "horisonttia ei pidetä nykyisin Hawkingin säteilylle tarpeellisena". Näin itsekin ajattelen, tai oikeastaan ajattelen ettei koko säteilyä ole olemassakaan.
Mutta jos tällaista ajattelua nykyään on, on JS:n koe vanhaa ajattelua, siinä nimenomaan mainitaan horisontti.
Jos on musta aukko, on horisontti. Vastakkaisiin suuntiin kohdistuvat fotonit voivat tietysti suuntautua molemmat puolipallosuuntiin. Alkuperäisessä ajatuksessa horisontti on niin lähellä, että toisen puolipallon suunnat kaikki johtavat lopulta horisontin taakse. Mitä kauempana horisontista annihilaatio tapahtuu sitä useammalla fotonilla on mahdollisuus selvitä joutumatta aukkoon, selvä se - siksikin romahtamassa olevan aukon haihtuminen on niin tehokasta ja saattaa olla, ettei mustia aukkoja voikaan muodostua. Mitä muodostuu ja miten voisimme havaita noita annihilaatioita?, mitä se säteily on? Voiko osa mustan kappaleen mukaisesta taustasäteilystä muodostuakin näissä prosesseissa? Kysymyksiä on enemmän kuin tarpeeksi. Mikäli lähiaikoina toteutuvaksi aiottu mustan aukon valokuvaus onnistuu, voimme saada joitain vastauksia...
Otsikko: Vs: Musta aukko ja sen muoto, tapahtumahorisontti
Kirjoitti: jussi_k_kojootti - 28.09.2016, 09:05:02
Lainaus käyttäjältä: mistral - 27.09.2016, 21:32:44
Mistä hiukkaspari tietää suuntautua niin että toinen menee prikulleen kohti ma:n ydintä tai prikulleen ulospäin?

Geometriasta.  Seiso seinän vieressä ja heitä umpimähkään palloa.  Noin puolet osuu ensin seinään jonka vieressä seisot.

Lainaa
Mutta tämä on yllättävää jos "horisonttia ei pidetä nykyisin Hawkingin säteilylle tarpeellisena". Näin itsekin ajattelen, tai oikeastaan ajattelen ettei koko säteilyä ole olemassakaan.
Mutta jos tällaista ajattelua nykyään on, on JS:n koe vanhaa ajattelua, siinä nimenomaan mainitaan horisontti.

Tässä ketjussa ajattelu kuin ajattelu alkaa tuntua vanhalta ennemmin kuin arvaatkaan :-)
Otsikko: Vs: Musta aukko ja sen muoto, tapahtumahorisontti
Kirjoitti: mistral - 28.09.2016, 12:21:41
Lainaus käyttäjältä: ketarax - 28.09.2016, 09:05:02
Geometriasta.  Seiso seinän vieressä ja heitä umpimähkään palloa.  Noin puolet osuu ensin seinään jonka vieressä seisot.
Lainaa

Jos viittaat geodeeseihin, niin oikeasti yli 99,99% niistä kaartaa ma:n ytimeen. Jos viittaat gravitaation suuntaan niin sillä on vain yksi suunta.


Lainaa
Tässä ketjussa ajattelu kuin ajattelu alkaa tuntua vanhalta ennemmin kuin arvaatkaan :-)

Itse en ole oikeastaan koskaan uskonut että Hawkingin säteily on mahdollista, Steinhauerin koekin mittasi atomien kvanttiheilahtelua, ei vakuumia. Kuitenkin villakoiran ydin on Hawkingin mekanismissa, hiukkasparin "yhteinen" koordinaatisto eli se mistä hiukkaset ponnahtavat esiin, ei tiedä mitään tapahtumahorisontista. Näin hiukkasparin kannalta on yhdentekevää tapahtuuko ponnahdus maan ja kuun välissä vai horisontissa.
Otsikko: Vs: Musta aukko ja sen muoto, tapahtumahorisontti
Kirjoitti: mistral - 28.09.2016, 21:42:54
Lainaus käyttäjältä: Joksa - 28.09.2016, 13:59:25
Monet viestit ja näkökulmat ketjun alussa ovat kyllä edelleenkin ihan mielenkiintoisia, tämä näyttäisi olevan kosmologia-teeman pisin ketju.

Katos, tämähän on ohittanut "Maailmankaikkeuden reuna" ketjunkin, emme päästä Hawkingia helpolla  :smiley:
Otsikko: Vs: Musta aukko ja sen muoto, tapahtumahorisontti
Kirjoitti: Eusa - 29.09.2016, 14:35:16
Lainaus käyttäjältä: Joksa - 29.09.2016, 03:17:41
Pieni pläjäys yksinkertaista mutta toivon mukaan selkeää logiikkaa tähän liittyen, lähtökohtana olisi se yleisesti hyväksytty ajatus että massan energia on positiivinen ja g-kentän energia määrältään sitä vastaava mutta negatiivinen.

Gravitaatiokentän fluktuointi kuten esitätte tarkoittaisi kai sitä että virtuaaliparista materialisoituvan hiukkas-antihiukkasparin positiivinen massaenergia ulosmitattaisiin negatiivisesta g-kentästä. Se tarkoittaa arkijärjen mukaan sitä että g-kentän negatiivinen energia lisääntyisi joka taas edellyttäisi sitä että ma:n positiivisen massan tulisi lisääntyä vastaavassa määrin, eli kahden hiukkasen verran.

Jos näistä kahdesta syntyneestä hiukkasesta toinen vielä putoaa aukkoon ja toinen karkaa niin aukkoonhan kertyisi kaikkiaan kolmen hiukkasen positiivinen energialisäys yhtä Hawking-säteillyttä hiukkasta kohti. Mielestäni tuossa g-kentän fluktuointiajatuksessa jokin siis mättää pahasti, vai onko päättelyssäni jotakin pahasti pielessä? Hawkingin säteilyn energiansiirron kvanttimekanismi ei voi toimia tällä tavalla g-kentän välityksellä.

Mikäli fluktuaation energiansiirto tarkoittaisi vastaavaa g-kiihtyvyyden syntymistä normaalisti a-a:n käyryyden välittämänä niin senhän ei ole todettu vaikuttavan kappaleiden g-kenttään tai massoihin muutoin kuin g-aaltoilutilanteessa, jolta tämä ei vaikuta. Kokonaisuuden massa- ja g-kenttävastaavuuskin säilyy.

Osaako joku kertoa mikä se ajateltu Hawkingin ergiavälitysmekanismi olisi - ellei tuollaisen aiemmin hahmottelemani kaltainen kelpaa?
Kun hiukkaspari muodostuu massafluktuaationa, se on saman tien pois mustan aukonkin massasta, eikä muodosta negatiivista potentiaalia minkään suhteen. Toinen vaihtoehtohan olisi sellainen, missä olemassa olevalla hiukkasella on aukosta erillinen massansa ja myös potentiaalinsa. Nuo kulkevat luonnollisesti käsi kädessä.

Havainnollistava mekanismi olisi sellainen, että tyhjön syöksyminen kertoo/määrittää kullakin pallokuorella massapisteen, eli tässä dominoivan mustan aukon, ympärillä tuon rakenteen hankkimasta energia- ja aikasisällöstä. Jos tyhjöä kulutetaankin fluktuoituviksi hiukkasiksi, on se pois aukon energiasta, aukko viilenee massakadon verran. Tämä voi tuntua hankalalta ja epäloogiselta, mutta eipä siihen oikeastaan jää vaihtoehtoja, jos Hawkingin säteilymekanismi muuten nähdään todelliseksi...

Muutenhan asia on hyvin ymmärrettävä: jos jostain säteilee enemmän pois, kuin se absorboi, energiasisältö luonnollisesti vähenee.
Otsikko: Vs: Musta aukko ja sen muoto, tapahtumahorisontti
Kirjoitti: Eusa - 29.09.2016, 16:38:56
Lainaus käyttäjältä: Joksa - 29.09.2016, 15:36:04
Alleviivatut lauseet koen hieman ristiriitaisina. Tuon "saman tien pois" tapahtuman tarkka välitysmekanismi tässä on ollut haarukoitavana.

Koska materialisoinnin vaatiman energian 'ulosmittaus' tietää kohdistua ma:han niin sen g-kentällä on asian kanssa tekemistä, ja jos se on massaenergian välittäjäreittinä niin siinähän käy kuten kuvasin. Jos taas ei niin mitä ihmeen reittiä ma:n energia kulkisi materialisoituviin hiukkasiin? Jonkun lisäulottuvuuden kautta kulkeva siirtymäreitti tuntuisi satuilulta ja "sinne se vaan popsahtaa" ei tunnu mitenkään riittävältä selitykselta - vaikka energiasummapeli jonkun siirtymämekanismin olemassaolon vaatiikin.

Säteilyjuttu yleisessä mielessä ok. Säteily kontra absorbtio tuossa g-kenttävälitteisessä energiareitissä ei olleetkaan kohdillaan...
Syöksyvä tyhjö tai gravitaatiomuutoksen signaalit ovat informaatioreitti, ja aina aukkoon päin. Aukosta ei tarvita mitään tietoa ulkopuolelle. Musta aukko tai mikä tahansa ainekeskittymä tarvitsee massaenergiansa vastineeksi jatkuvaa tyhjön syöksyä. Jos fluktuaatioilla voidaan osa tuosta syöksystä ryöstää, tarkoittaa se välittömästi aukon energian vähenemistä, viilenemistä / massavajetta.

Eräs näkökohta on se, tarvitseeko Hawkingin säteily romahtaneen aineen matalimman tason identiteettin, sileän kondensaatin, muodostumista, josta fluktuaatioiden tunneloituminen aukon ulkopuoliseksi aineeksi olisi mahdollista, vai voiko tuo tapahtua ihan tavallisen radioaktiivisen hajoamisen periaatteillakin: kun syöksyvän tyhjön energiaa ei ole riittävästi tarjolla, aine "haihtuu"; sidokset heikentyvät, jne. ?

Jos fluktuaatioilla voi vaikuttaa massakenttään, olisi ehkä mahdollinen sellainen generaattori, joka keinotekoisesti tuottaa fluktuaatioita gravitaatiokentästä, esim. kuihduttamalla auringon/tähden massaa. Myös tilan vähentäminen tyhjöä nyhtämällä voisi silloin onnistua - mikäli emDrive osoittautuu todelliseksi ilmiöksi, siinäpä yksi selityskandidaatti.
Otsikko: Vs: Musta aukko ja sen muoto, tapahtumahorisontti
Kirjoitti: mistral - 30.09.2016, 00:58:41
Palaan vielä ns. Hawkingin mekanismiin, kuinka hiukkaspari voisi joutua erotetuiksi toisistaan. Ja siis siihen koordinaatistoon josta hiukkaset ponnahtaa esiin. Jos otetaan ääripäät, silloin
A) paikallaan suhteessa ma:han oleva koordinaatisto on horisontin pinnassa ja toinen fotoni menee "myötätuuleen" kohti singulariteettia, toinen "vastatuuleen" kohti vapaata avaruutta. Molemmat menee valon nopeudella koska valon nopeus riippumaton havaitsijan liiketilasta.

B) valon nopeudella läpi horisontin syöksyvä koordinaatisto lähettää hiukkasparin samoin kuin paikallaan oleva koordinaatisto mutta erona on se että vapaaseen avaruuteen pyrkivä fotoni punasiirtyy äärettömän pitkäaaltoiseksi ja jos sitä tarkastellaan vapaasta avaruudesta, sitä ei siis ole olemassakaan sieltä katsottuna. Mutta omasta koordinaatistosta katsottuna se ilmeisesti on olemassa, samoin sisäänpäin menevä fotoni.

Näin molemmissa A ja B tapauksissa omassa koordinaatistossa ei näyttäisi olevan suurta eroa, eli horisontti ei näyttäisi voivan erottaa hiukkasia toisistaan. En ainakaan keksi mikä olisi muuttunut niin paljon että hiukkaset olisi Planckin ajan jälkeen tuomittu eroamaan toisistaan.

Tässä A ja B on ääripäät joten niiden väliin jäävät vaihtoehdot on samanlaisia eli hiukkaset yhtyy Planckin ajan jälkeen eikä säteilyä synny. Tai mikä se aika sitten onkaan. Horisontti ei paikalliselle koordinaatistolle ole mikään erityinen raja, meille vapaan avaruuden havaitsijoille se on.

Avaruudessa muuten on niin suuria mustia aukkoja että niiden horisontissa gravitaatio on samaa luokkaa kuin maan pinnalla. Näin on yhdentekevää gravitaation kannalta, käveleekö järven jäällä tai horisontin "jäällä". Tämä ehkä auttaa ymmärtämään ettei tällaisessa paikalliskoordinaatistossa ole sinänsä mitään erikoisia olosuhteita. Tietysti mitä pienempi ma on, sitä jyrkempi gravitaatiokaivo on. Jos pölyhiukkanen prässätään niin pieneksi että se romahtaa oman gravitaationsa kaivoon, on kaivo hyvin jyrkkä - millin päässä ei ole mitään tietoakaan kaivon olemassaolosta, mutta kun mennään tarpeeksi pieniin mittoihin, syöveri imaisee mukaansa. Tässä tapauksessa olosuhteet on kyllä erikoiset, mutta en tiedä erottaako sekään Planckin pituuksilla hiukkasparia toisistaan.
Otsikko: Vs: Musta aukko ja sen muoto, tapahtumahorisontti
Kirjoitti: Kaizu - 30.09.2016, 10:01:01
Lainaus käyttäjältä: mistral - 30.09.2016, 00:58:41
Palaan vielä ns. Hawkingin mekanismiin, kuinka hiukkaspari voisi joutua erotetuiksi toisistaan. Ja siis siihen koordinaatistoon josta hiukkaset ponnahtaa esiin. Jos otetaan ääripäät, silloin
A) paikallaan suhteessa ma:han oleva koordinaatisto on horisontin pinnassa ja toinen fotoni menee "myötätuuleen" kohti singulariteettia, toinen "vastatuuleen" kohti vapaata avaruutta. Molemmat menee valon nopeudella koska valon nopeus riippumaton havaitsijan liiketilasta.

B) valon nopeudella läpi horisontin syöksyvä koordinaatisto lähettää hiukkasparin samoin kuin paikallaan oleva koordinaatisto mutta erona on se että vapaaseen avaruuteen pyrkivä fotoni punasiirtyy äärettömän pitkäaaltoiseksi ja jos sitä tarkastellaan vapaasta avaruudesta, sitä ei siis ole olemassakaan sieltä katsottuna. Mutta omasta koordinaatistosta katsottuna se ilmeisesti on olemassa, samoin sisäänpäin menevä fotoni.

Tästä saa sen kuvan että Hawkingin säteily olisi fotoneita.

Kaizu
Otsikko: Vs: Musta aukko ja sen muoto, tapahtumahorisontti
Kirjoitti: mistral - 30.09.2016, 13:47:40
Lainaus käyttäjältä: Joksa - 30.09.2016, 09:50:26
Ulkoisen havainnoijan kaukoputkeen tulee aukon horisontin tienoilta fotoneita kun osaa vain havaita välimatkan pituisia aallonpituuksia.


Tässä B tapauksessa jossa vakuumi syöksyy c nopeudella horisontin läpi, on kahta eri punasiirtymää. Ensiksi on Doppler-siirtymä joka venyttää fotonin aallonpituuden äärettömäksi ja sitten vielä lisäksi gravitaatiopunasiirtymä joka tekee saman asian joten käytännössä B tapauksessa ei mitään säteilyä pääse vapaaseen avaruuteen.

Kuitenkin B tapaus on mielestäni ainoa jossa Hawkingin säteily olisi jotenkin rinnastettavissa JS:n tekemään fononi kokeeseen, eli tässä tilanteessa rinnastus olisi fononin nopeus keinotekoisessa horisontissa olisi toisella puolella alle äänen nopeus, toisella yli äänen nopeus. Ja vastaavat jutut olisi ma:n kohdalla: vakuumin putoamisnopeus horisontin yläpuolella alle c ja alapuolella yli c.
Mutta tästä huolimatta en tajua kuinka hiukkaspari saadaan eroamaan toisistaan.

Kansanomaisessa selityksessä hiukkaspari erotetaan sillä että sisäpuolella oleva hiukkanen ei voi ylivalonnopeudella palata takaisin ulkopuolella olevan luo.  Kuitenkaan tässä ei puhuta mitään siitä, mikä estää ulkopuolella olevaa hiukkasta palaamasta sisäpuolella olevan luo.
Otsikko: Vs: Musta aukko ja sen muoto, tapahtumahorisontti
Kirjoitti: mistral - 30.09.2016, 13:53:20
Lainaus käyttäjältä: Kaizu - 30.09.2016, 10:01:01
Tästä saa sen kuvan että Hawkingin säteily olisi fotoneita.

Kaizu

Sehän on se idea millä ma:n massa haihtuisi "äärettömien" aikojen kuluessa.
Otsikko: Vs: Musta aukko ja sen muoto, tapahtumahorisontti
Kirjoitti: mistral - 30.09.2016, 16:03:36
Lainaus käyttäjältä: Joksa - 30.09.2016, 14:48:25
Jos horisontin koordinaatiston luistelija tuijaa taskulampullaan ylöspäin vaikka johonkin puunlatvaan tai mastoon tms niin ei siis koe mitään erikoista valon 'lurpahtamista' vaan kokee valon näyttävän normaalisti. Niin luistelijan perskpektiivistä, ulkoisen havainnoijan näkökulmasta ei koska hänen mielestään luistelija olisi jähmettynyt hänen ajan olessa pysähtynyt. Koska gravitaation vaikutusmatka olisi huimasti pidempi kuin täällä niin valon punertuisi ja teho heikkenisi edetessään pidemmälle kuin täältä poistuessaan. Valon etenemismatka ennen kuin fotonit kääntyy takaisin tai niiden aallonpituus on ääretön, olisi pitkä...

Eli :A) tapauksessasi valo etenee horisontista mutta aikatekijät ovat erilaiset horisontin ja ulkoisen havainnoijan kannalta. B)tä on vaikeampi ruotia aikakysymyksen vuoksi.

Ongelma ei taida olla siinä etteikö valo pääsisi mustan aukon horisontista tai jopa sen sisältä vaan aikatekijä, eli tapahtuminen on äärettömän hidasta. Tämä vaikuttaa myös Hawkingin säteilyn osalta.

Joo, supermassivisen ma:n vaikutus ulottuu voimakkaana miljardien kilometrien päähän, siksi maan suuruisen gravitaation voima riittää "kumoamaan" c pakonopeuden.
A:ssa vaikuttaa vain gravitaatiopunasiirtymä, B:ssä sen lisäksi Doppler-punasiirtymä. En tiedä oliko JS ottanut molemmat huomioon kun sanoi että Hawkingin säteily olisi jotain 3 nanokelviniä vai oliko se 0,3 nanoa. Taustasäteily on n. 2,7 kelviniä joten lienee mahdotonta kaivaa taustasäteilyn seasta noin pikäaaltoista komponenttia ulos.
Otsikko: Vs: Musta aukko ja sen muoto, tapahtumahorisontti
Kirjoitti: mistral - 01.10.2016, 22:30:49
Lainaus käyttäjältä: Joksa - 01.10.2016, 17:07:05
Taskulampulla tuijaileva luistelijaesimerkkihän on suorastaan valaiseva. Vielä ma:n g-kentässä olevat havainnoijat havaitsisivat luistelijan liikkuvan, etäämmältä hitaammin ja tuijun valo enemmän punertuneena. Toisin sanoen lähempänä olevien havainnoijille tapahtumahorisontti olisi syvemmällä. Tapahtumahorisontin syvyys kuten havainnoijien aikakin on siis suhteellinen riippuen havainnoijan etäisyydestä.

Se oli keväällä kun vastasin ketaraxille tästä asiasta viestissä 93:

ketarax:
    "(En ole aivan varma ymmärrämmekö "asymptoottisen tulkinnan" samalla tavalla -- minä ymmärrän nämä tulkinta-asiat niin, että sekä kaukaisen havaitsijan että putoajan havainnot ovat tosia -- eli putoaja ylittää horisontin, ja havaitsija todella mittaa punasiirtyneitä fotoneita (eikä koskaan mittaa sitä näennäiseltä "0-aikavyöyhkkeeltä" lähtevää fotonia).  Toisin sanoen yhden sijaan totuuksia on kaksi -- ja tämä on edelleen mielestäni "paradoksaalista""

mistral:
"Tai oikeastaan totuuksia on yksi mutta se yksi totuus on kaksiosainen (tai useampiosainen). Ensimmäinen osa kattaa vapaan avaruuden aikakauden, toinen osa poissulkee ensimmäisen ja kattaa horisontin sisäpuolta. Mutta se että se kattaa sisäpuolta, ei tarkoita että kattaisi koko sisäpuolen, sillä kun astronautti syöksyy horisontin läpi, on hänen näkökulmasta seuraava horisontti syvemmällä syövereissä. Se on yksinkertaisesti niin syvällä, mistä ei edes valo pääse pakenemaan ykkös horisontille. Ja edelleen kun hän on kakkoshorisontilla, odottaa kolmoshorisontti seuraavana. Eli tavallaan voidaan niinkin ajatella että horisontti olisi liukuva taso.
Kummallista tässä on jos ykköshorisontin aika on pysähtynyt, niin ykköseltä katsottuna taas kakkosen aika on pysähtynyt jne. Eli kuinka pysähtyneen ajan suhteen voi jonkun toisen paikan aika pysähtyä? Tämä tietysti vaatii sen että suhteellisuusteoria on voimassa gravitaatiokaivon pohjalle saakka."



Jostain netistä sain tämän idean ja sinänsä periaate on yksinkertainen, fotonin pakonopeus. Olen kuitenkin ihmetellyt miten seuraava tilanne menee:

Oletetaan että maan lähellä vaikka 10 valovuoden päässä on pienehkö musta aukko. Maasta lähetetään 5 satelliittia jonossa ma:han ja tarkoitus on saada dataa hyvin syvältä. Satelliittien välimatka olkoon vaikka 1000km. Kun ne alkavat syöksyä aukkoon, pääsee etummainen vastaanottamaan ehkä dataa jopa horisontin takaa, siis maan näkökulmasta. Kun etummainen lähettää saman datan 2:lle ja 2 lähettää 3:lle jne, niin pystyykö näin saamaan maahan tietoa horisontin sisäpuolelta? Suhteellisuusteorian mukaan horisontin takaa ei voi tulla tietoa koska aika on siellä pysähtynyt. Onko juju nyt siinä että horisontin takaa ei voi saada hiukkasia mutta informaatiota voisi saada? Tässä on vaan se mahdottomuus että informaatio olisi jostain tulevaisuudesta mikä on täysin mahdotonta  :huh:

Otsikko: Vs: Musta aukko ja sen muoto, tapahtumahorisontti
Kirjoitti: Kaizu - 04.10.2016, 09:24:52
Lainaus käyttäjältä: mistral - 30.09.2016, 13:53:20
Sehän on se idea millä ma:n massa haihtuisi "äärettömien" aikojen kuluessa.
Eipäs. Hawkingin säteilyä on virtuaalihiukkasparin se osapuoli joka ei joudu tapahtumahorisontin sisälle.
Elikkä sähkömagneettisen kentän tapauksessa elektroni tai positroni. Mitä syntynyt hiukkanen touhuaa syntymästään eteenpäin on sitten toinen juttu, saattahan se tavata vastaavalla tavalla syntyneen antihiukkasensa ja anhiloitua yhdessä ja tuottaa fotoniparin.

Kaizu
Otsikko: Vs: Musta aukko ja sen muoto, tapahtumahorisontti
Kirjoitti: mistral - 04.10.2016, 19:22:20
Lainaus käyttäjältä: Kaizu - 04.10.2016, 09:24:52
Eipäs. Hawkingin säteilyä on virtuaalihiukkasparin se osapuoli joka ei joudu tapahtumahorisontin sisälle.
Elikkä sähkömagneettisen kentän tapauksessa elektroni tai positroni. Mitä syntynyt hiukkanen touhuaa syntymästään eteenpäin on sitten toinen juttu, saattahan se tavata vastaavalla tavalla syntyneen antihiukkasensa ja anhiloitua yhdessä ja tuottaa fotoniparin.

Kaizu

Miksei mutta kuinka elektroni pääsee pois jos sen nopeus ei riitä?
Otsikko: Vs: Musta aukko ja sen muoto, tapahtumahorisontti
Kirjoitti: mistral - 04.10.2016, 19:54:24
Lainaus käyttäjältä: Joksa - 04.10.2016, 10:09:59
Olisikohan kyse etäisimmän havaitsijan ja välihavaitsijoiden ajankulun eroista? Tuo ma:n tieto tai fotoni tulisi etäisimmällle havaitsijalle niin pitkän ajan päästä ettei se enää koskisikaan tulevaisuutta, sen etäisimmän havaitsijan mielestä välihavainnoijien toimet kun sujuisivat kovin verkaan. Saattaisi olla gps:ien ajankulun kanssa analoginen tilanne tai kaksosparadoksin ikämatematiikan ja heidän keskinäisen viestinvaihdon ajahetkien tapainen härdelli. Yksityiskohtaisempi selitysyritys vaatisi varmaan jokseenkin intensiivistä aivojumppaa...  :huh:

Saattaa olla ettei luotaimilla saavuteta mitään muuta kuin signaalin suuntaus suoraan seuraavalle luotaimelle. Periaatteessa luotain ei voi nopeuttaa frekvenssiä koska se ei voi vastaanottaa sen nopeammin kuin alempaa tulee. Jos se nopeuttaisi frekvenssiä lähetysantenniin, joutuisi se ensin odottamaan että viimeinen bitti on tallennettu ja silloin oltaisiin jo pudottu hirmu matka alaspäin. Viimeinen bitti näköjään määrää kattonopeuden. Joku laskutaitoinen tietty voisi laskea pystyykö luotaimilla nostamaan frekvenssiä. Maassa sillä olisi merkitystä jos bittien väli olisi 24 tunnin sijasta vaikka 24 sekuntia.
Otsikko: Vs: Musta aukko ja sen muoto, tapahtumahorisontti
Kirjoitti: Kaizu - 05.10.2016, 09:07:36
Lainaus käyttäjältä: mistral - 04.10.2016, 19:22:20
Miksei mutta kuinka elektroni pääsee pois jos sen nopeus ei riitä?
Jos se ei pääse pois niin se ei ole Hawkingin säteilyä. Hiukkasparilla on syntyessään sekä massa- että liike-energiaa, mitä enemmän sen lyhyempi on hiukkasparin sallittu elinikä. Jotta se elektronina pääsisi pois niin sille pitää jäädä riittävä liikemäärä kun tapahtumahorisontti korjaa sen kaverin pois tästä maailmasta. Vaihtoehtoisesti se voi etsiä kaverikseen vastaavalla lailla syntyneen positronin ja anhiloitua fotonipariksi. Tämä koskee yhtälailla positroneja ( ja muitakin hiukkasia ja antihiukkasia). Feynmanin kaavioita käyttämällä asia on helpompi mieltää. Elektroni ja positroni vuorovaikuttavat lähettelemällä toisilleen fotoneja. Tapahtumahorisontin sisälle joutuneen osapuolen fotonit eivät koskaan saavu perille jolloin ulkopuolelle jäänyt osapuoli ei enää tunne vetoa ja lähtee jäljelle jääneellä liikemäärällään viipottamaan vapauteen.

Kaizu

Otsikko: Vs: Musta aukko ja sen muoto, tapahtumahorisontti
Kirjoitti: Eusa - 05.10.2016, 10:18:18
Lainaus käyttäjältä: Kaizu - 05.10.2016, 09:07:36
Tapahtumahorisontin sisälle joutuneen osapuolen fotonit eivät koskaan saavu perille jolloin ulkopuolelle jäänyt osapuoli ei enää tunne vetoa ja lähtee jäljelle jääneellä liikemäärällään viipottamaan vapauteen.
Tapahtumahorisonttiin liittyy ns. palomuuriongelma, kun tarkastellaan sähkömagneettisia vuorovaikutuksia. Tapahtumahorisonttiin liitetyt fysikaaliset vuorovaikutusrajaukset ovat vaikeuksissa.

Oletettavin tapahtumaketju on sellainen, että ihan gravitaatiokentän kaarevuuden voimakkuudesta johtuen hiukkaspari, esim. elektroni-positroni, saa niin kauan elinaikaa, että eivät jää virtuaalisiksi eli säteilytasapainoon sisltyväksi laskelmaksi, vaan ehtii muodostua todellisia hiukkasia. Nuo samaiset hiukkaset voivat seuraavaksi keskenään annihiloitua ja muodostuneista fotoneista toinen pääsee karkuun.
Otsikko: Vs: Musta aukko ja sen muoto, tapahtumahorisontti
Kirjoitti: Kaizu - 05.10.2016, 11:59:57
Ulkoa katsoen aika näyttää hidastuvan kun jokin tapahtuma joutuu voimakkaaseen gravitaatiokenttään ja lopulta pysähtyvän tapahtumahorisontin kohdalla. Kyseisen tapahtuman kokema paikallinen aika ei sen sijaan muutu miksikään. Tapahtumahorisontista ulospäin katsottaessa muu maailmankaikkeus taas nopeutuu äärettömiin.

Kaizu
Otsikko: Vs: Musta aukko ja sen muoto, tapahtumahorisontti
Kirjoitti: Eusa - 05.10.2016, 12:15:06
Lainaus käyttäjältä: Kaizu - 05.10.2016, 11:59:57
Ulkoa katsoen aika näyttää hidastuvan kun jokin tapahtuma joutuu voimakkaaseen gravitaatiokenttään ja lopulta pysähtyvän tapahtumahorisontin kohdalla. Kyseisen tapahtuman kokema paikallinen aika ei sen sijaan muutu miksikään. Tapahtumahorisontista ulospäin katsottaessa muu maailmankaikkeus taas nopeutuu äärettömiin.
Noinhan se matematiikka on sen jälkeen ollut, kun singulariteetti ja tapahtumahorisontti eriytettiin toisistaan. Mustien aukkojen tutkijoille yleiseksi ohjeeksi sopii sama kuin William Faulknerin ohje kirjoittajille "kill your darlings!"

Eli kannattaa tutkailla miten homma toimii mikäli tapahtumahorisontti asetetaankin teoriassa saavuttamattoman asymptootin asemaan.
Otsikko: Vs: Musta aukko ja sen muoto, tapahtumahorisontti
Kirjoitti: Jarpes - 13.04.2017, 21:54:01
Moit!
..Kahalsin silmäillen kymmenen ensimmäistä sivua ja loikkasi tänne...
...Ihan hienoa sepustusta ja esimerkkejä kertymäkiekosta ja avaruusäijän siirtymisestä tapahtumahorisontin tuolle puolen ja sitten iäsyyden kuluessa ytimeen..

..Mutta ihmettelen jotteiko kertymäkiekossa aineen vauhdin noustessa julmiin nopeuksiin myöskin lämpötila kasva ?

..Kuinka kiehuvan aineen kertymäkiekkoa on niin vaikea havaita? em. Lämpötilan nousu kiihtyvyydessä...

....Miten käy esimerkkien avaruusäijän olomuodolle hyörystävässä kertymäkiekkossa?

Jos moista hyörystymistä ei ole, kuinka noin mittakaavan supistumisessa selviäisi kuntippuu tapahtumahorisontin tuolle puolen, satakiloinen, esim vain kitistyy (kasvamaan alkavassa painovoimassa) ja ainakaan itse en haluaisi, jos tuntisin edelleen tätä ikuista näläntunnetta iäti (olettaen ettei kasvava  painovoima repisi kappaleiksi)...
Sinänsä ajatuksellekesti hyviä mutta ontuvia esimerkkejä..

....Ja yritin ontuvalla englannilla no löytää Sag A* :sta lämpötilaa, jos täältä nähdään kaukaisen galalksimme keskipiste, jokuhan tietää sielläolevan lämpötilan?

T:...Näin MA:Jen havainnoimisen vaikeutta ihmettelvä
Otsikko: Vs: Musta aukko ja sen muoto, tapahtumahorisontti
Kirjoitti: jussi_k_kojootti - 14.04.2017, 00:03:21
Lainaus käyttäjältä: Jarpes - 13.04.2017, 21:54:01
..Mutta ihmettelen jotteiko kertymäkiekossa aineen vauhdin noustessa julmiin nopeuksiin myöskin lämpötila kasva ?

Kyllä vaan kasvaa.  Kertymäkiekot liittyy todennäköisesti useimpiin luonnon "äärimmäisistä" prosesseista.

Lainaa
..Kuinka kiehuvan aineen kertymäkiekkoa on niin vaikea havaita? em. Lämpötilan nousu kiihtyvyydessä...

Havainto sinällään ei aina ole niin vaikea, riittävä varmuus siitä että havaittu kohde todella on kertymäkiekko (tai, jotain muuta) on sitten vähän eri asia, näin kaukaa kuin me havaitaan.

Lainaa
....Miten käy esimerkkien avaruusäijän olomuodolle hyörystävässä kertymäkiekkossa?

Surkeasti.  Ajatuskokeet jms. ovat idealisoituja, kaikki avaruusäijän tappava jätetään huomiotta / siitä ei välitetä.

Lainaa
Jos moista hyörystymistä ei ole, kuinka noin mittakaavan supistumisessa selviäisi kuntippuu tapahtumahorisontin tuolle puolen, satakiloinen, esim vain kitistyy (kasvamaan alkavassa

Selviämisen yksityiskohdat riippuu lähestyttävästä aukosta.  Tähdenmassaisista ei selviä (kuten ei neutronitähdistäkään -- tai pannahinen, Auringosta :-)).  (Elämän kannalta) idealisoidussa supermassiivisessa (tarpeeksi monta auringon massaa) voi, teoriassa, selvitä jonkin aikaa.

Lainaa
....Ja yritin ontuvalla englannilla no löytää Sag A* :sta lämpötilaa, jos täältä nähdään kaukaisen galalksimme keskipiste, jokuhan tietää sielläolevan lämpötilan?

Jos katsot tästä (https://www.youtube.com/watch?v=u_gggKHvfGw), ei galaksimme keskipisteessä *näytä* olevan yhtikäs mitään.  Oletus on se, että siellä on musta aukko, jollainen mustan kappaleen säteiljiäksi tulkittuna on kylmä.

Lainaa
T:...Näin MA:Jen havainnoimisen vaikeutta ihmettelvä

Ota takasta hiili, ja mene pimeään komeroon katselemaan :-)
Otsikko: Vs: Musta aukko ja sen muoto, tapahtumahorisontti
Kirjoitti: Timo Kuhmonen - 09.04.2019, 20:17:21
radiointerferometriaan liittyvä ketju, jaettu erilliseksi...
https://www.avaruus.fi/foorumi/index.php?topic=17859
Otsikko: Vs: Musta aukko ja sen muoto, tapahtumahorisontti
Kirjoitti: mistral - 04.12.2019, 20:23:30
Tänään 4.12 uutisessa kova väite:
"Valkoiset aukot nähdään usein eksoottisina kohteina, mutta suhteellisuusteorian näkökulmasta ne eivät ole kovin kummallisia. Ne ovat vain mustia aukkoja, jotka on käännetty ajan suhteen", toteaa Aurélien Barrau CNRS-tutkimuslaitoksesta Grenoblesta.

Tämä vissiin tarkoittaa että valkoisesta aukosta lähtevä entropia pienenee? Eikö termodynamiikan mukaan entropian pitäisi suureta?
Otsikko: Vs: Musta aukko ja sen muoto, tapahtumahorisontti
Kirjoitti: Mare Nectaris - 05.12.2019, 11:34:42
Uutisessa tutkijat itsekin pohtivat tuon pallon pompottelu -vertauksensa osuvuutta toteamalla, että sitä ei oikeastaan voi soveltaa mustaan aukkoon, koska tuntemamme fysiikan lait eivät siellä päde. Pointti oli siis siinä, että kvanttitason tunneloituminen voisi teoreettisesti kääntää ajan suunnan, mutta ilmiön havaitseminen ulkopuolelta olisi mahdotonta aikaskaalan vuoksi.

Ehkä voitaisiin ajatella, että kvanttitason ilmiöt ovat kuin Creme Brulé -vanukkaan sisus: siellä "kihisee ja kohisee", ja kvanttitason ilmiönä tarkasteltuna mekin - ja kaikki muu materia - ovat vain todennäköisyysaaltoja. - Tosin juuri katselin "Through The Wormhole with Morgan Freeman" - ohjelman jakson, jossa eräiden tutkijoiden hypoteesin mukaan kvanttitasolla liikkuu informaatiota ajassa eteenpäin ja taaksepäin, ja informaation kohdatessa "syntyy" nykyhteki (analogiana oli curling-rata, jossa liikkuu kiviä molempiin suuntiin; toiset sinisiä ja toiset punaisia). Tutkijoiden mukaan tässä ei ole kyse hiukkaskiihdyttimen tasoisista törmäyksistä. Teorian mukaan todennäköisyysaalto siis "romahtaa" todellisuudeksi (ts. havaitsemaksemme vaihtoehdoksi tapahtumista), kun menneisyys ja tulevaisuus kohtaavat nykyhetkessä.

Mutta mikä sitten tekee entropiasta havaitsemamme ajan nuolen täällä "materian maailmassa"? - Voisiko kysymys olla Higgsin kentästä? Jos Higgsin kenttä "antaa hiukkasille massan", samalla se tekee atomien ja materian muodostumisesta mahdollista. Higgsin kenttä mahdollistaa siis myös entropian suunnan, koska se määritellään aineen / energian käsitteillä. Higgsin kenttä on siis kuin Creme Brulén sokerista sulatettu, rapea pinta, jossa myös kvanttitason todennäköisyysaallot ovat "romahtaneet" tai tavallaan "jäätyneet" (onko mekanismi sitten juuri Higgsin kentässä noiden tutkijoiden hypoteesin mukainen).

Higgsin kenttä romahtanee mustassa aukossa, jolloin informaatio on jälleen kvanttitason ilmiöiden "armoilla", eli todennäköisyysaaltoja ei "kiinnitä" mikään.

On hauskaa ajatella, että sähkömagnetismin maailmassa (neuronien väliset sähköpurkaukset aivojen synapseissa) ajalla ei myöskään ole "koettua suuntaa", eli unissamme aika voi kulkea miten tahansa. - Ruusu kuitenkin kuihtuu yön aikana maljakkoon, vaikka yöllä unia näkisimmekin.

Näistä kosmologisista teorioista saa niin hauskoja juttuja ja uutisia. Ongelmana vain on empiirinen todentaminen. Esimerkiksi kysymys siitä, onko entropian havaittavien ilmiöiden ja kokemuksemme "taustalla" ikuisuus (aika), ja olemmeko oman maailmankaikkeutemme sisällä "kuplassa", kuin yksi monista ilmapalloista huoneessa (em. ohjelmassa esitetty hauska vertaus monimaailmankaikkeuksien teoriasta). Emme koskaan voi "nähdä" koko huonetta, koska olemme oman ilmapallomme sisällä. Myöskään informaatio ei voi siirtyä ns. todennäköisyystimantin ulkopuolelle (ts. valokartio, joka syntyy yhdistämällä informaatio menneestä ja tulevasta). Em. ohjelmassa esiintyneen tutkijan mukaan pelkästään omassa maailmankaikkeudessamme kokonaisinformaatio nykyisellä aikaskaalalla olisi 10 potenssiin 10 potenssin 123. Ja informaatio, jonka voisimme koskaan saavuttaa, olisi 10 potenssiin 10 potenssiin 93. - Tutkijan mukaan kuitenkin pelkästään atomien laskemiseen noin desistä vettä vaadittaisiin noin Yhdysvaltain kokoinen laite, joten kokonaisinformaation tuottamiseen vaadittaisiin suurempi laite, kuin mitä tunnettuun maailmankaikkeuteen mahtuisi. Ja hänen ironisen kommenttinsa mukaan sekin romahtaisi massansa vuoksi mustaksi aukoksi.

Uutisoinnin ongelma on perimmiltään siinä, että sekoitetaan kvanttifysiikka, materiaalinen fysiikka (Higgsin kenttä atomeineen) ja ajan kokeminen. Hauskoja anekdootteja tästä sopasta saa kirjoihin, mutta ennen kuin kukaan näistä popularisoivista ja kvanttitason ruusuun sekoittavista tyypeistä esittelee meille kuolleen isänsä, ovat jutut vain juttuja. Ja ruusu kuihtuu. 

Toistaiseksi Einsteinin teoria ajan ja tilan yhteenkietoutumisesta on vahvimmin empiirisesti todennettu malli ajan ja avaruuden yhteydestä. Teoreettisesti tuossakaan mallissa ajalla ei ole "suuntaa", mutta Higgsin kentässä massan saavat hiukkaset huolehtivat siitä, että nyt on nyt, eilen oli eilen, ja huomenna ruusu on kuihtunut.
Otsikko: Vs: Musta aukko ja sen muoto, tapahtumahorisontti
Kirjoitti: mistral - 05.12.2019, 17:19:19
Ajattelen niin että aika on sopimus: sovitaan että mennyt on mennyttä ja tuleva tulevaa. Sopimusta ei voi rikkoa, tai tietysti voi jos haluaa määritellä asian toisella tavalla. Mutta jos pitää kiinni sopimuksesta, silloin se on riippumaton muista fysikaalisista voimista, etäisyyksistä ym. Siis aika on mielestäni aina etenevä kohti tulevaisuutta. Se että eri koordinaatistoissa on eri ajan nopeus toisiinsa verrattuna, ei poista "plus merkkiä" eli eteenpäin merkkiä. Mielestäni suhteellisuusteoria toimii ikäänkuin hammasrattaina eri koordinaatistojen välillä. Kaikissa koordinaatistoissa tunti kestää tunnin mutta näkymättömät hammasrattaat kytkee ne toisiinsa. Eli mielestäni rattaita ei koskaan saa pyörimään päinvastaiseen suuntaan, no tämä on vain mielipide.
Otsikko: Vs: Musta aukko ja sen muoto, tapahtumahorisontti
Kirjoitti: Kaizu - 11.12.2019, 01:59:22
Kvanttimekaniikan sovellusalueella aika voi kulkea mihin suuntaan vain. Makroskooppisten kohteiden ajan suunta määräytyy termodynamiikan mukaan, lämmin kappale jäähtyy ajan myötä. Makroskooppinen havaitsija ei voi kääntää ajan suuntaa mutta voi vaikuttaa henkilökohtaiseen aikaansa siirtymällä voimakkaampaan gravitaatiokenttään tai liikkumalla relativistisella nopeudella.

Kaizu
Otsikko: Vs: Musta aukko ja sen muoto, tapahtumahorisontti
Kirjoitti: Eusa - 11.12.2019, 08:32:35
Lainaus käyttäjältä: Kaizu - 11.12.2019, 01:59:22
Kvanttimekaniikan sovellusalueella aika voi kulkea mihin suuntaan vain. Makroskooppisten kohteiden ajan suunta määräytyy termodynamiikan mukaan, lämmin kappale jäähtyy ajan myötä. Makroskooppinen havaitsija ei voi kääntää ajan suuntaa mutta voi vaikuttaa henkilökohtaiseen aikaansa siirtymällä voimakkaampaan gravitaatiokenttään tai liikkumalla relativistisella nopeudella.
Ajan suunta on vain väärinymmärrys tai keskittymis-/jäsentelyhäiriö. Se on kytköksissä siihen, että valolla on vääjäämätön suuntansa toisin kuin valoa hitaammilla massallisilla hiukkasilla.

Valo määrittää tilaa ja aika massaa.
Otsikko: Vs: Musta aukko ja sen muoto, tapahtumahorisontti
Kirjoitti: mistral - 11.12.2019, 17:30:50
Lainaus käyttäjältä: Kaizu - 11.12.2019, 01:59:22
Kvanttimekaniikan sovellusalueella aika voi kulkea mihin suuntaan vain. Makroskooppisten kohteiden ajan suunta määräytyy termodynamiikan mukaan, lämmin kappale jäähtyy ajan myötä. Makroskooppinen havaitsija ei voi kääntää ajan suuntaa mutta voi vaikuttaa henkilökohtaiseen aikaansa siirtymällä voimakkaampaan gravitaatiokenttään tai liikkumalla relativistisella nopeudella.

Kaizu

Olen miettinyt tätä juttua: sanotaan että fysiikassa monet tapahtumat voi kääntää vastakkaiseen suuntaan, pallon pomppaus voi mennä vastakkaiseen suuntaan tai vaikkapa kolari jossa auto palautuu ehjäksi. Eihän koskaan niin käy mutta jos kaikki voimat, lämpö ja säteily menisi tarkkaan takaperin, silloin se olisi mahdollista. Kuitenkin entropian väitetään olevan erilainen, se ei voisi mennä toiseen suuntaan. Väite ettei entropia voisi mennä taaksepäin on mielestäni kaksipiippuinen juttu. Kuvaan sitä suurennoksella, suurennetaan kaasua. Otetaan vaikka huoneilmasta yksi pieni kuutiotilavuus johon mahtuisi 2 happiatomia ja 8 typpiatomia. Suurennetaan ne kuutiometrin tilaan ja annetaan niille värit että näkyvät lasikaapissa. Ne törmäilisi seiniin ja toisiinsa ikuisesti. Jotkut atomit olisi välillä nopeampia kuin toiset mutta osat vaihtuisivat sattumanvaraisesti. Viereen laitetaan samanlainen lasikaappi jossa on pingispalloja jotka pomppivat ikuisesti törmäten seiniin ja toisiinsa. Tässä huomaa sen ettei lasikaapeilla ole mitään eroa fysikaalisesti. Molemmissa kappaleet törmäilee samojen lakien mukaan ikuisesti. Kuitenkin väitetään että pingispallojen kohdalla aika voidaan kääntää taaksepäin mutta atomien kohdalla ei voida (koska se rikkoisi entropian sääntöä vastaan).
Joten entropiaa pidetään syynä ettei aika kulje taaksepäin. Varmaan sinänsä totta mutta entropiakin on seuraus eikä perimmäinen syy. Kun kelaan syitä, pidän entropian syynä tai olemuksena todennäköisyyttä. Epätodennäköisyys/todennäköisyys taas johtuu ikäänkuin monista vaihtoehdoista (arpakuutio), luonto on täynnä vaihtoehtoja eikä etukäteen tiedä miten asiat menee. Kuitenkin todennäköisyys pätee suuriin lukumääriin ja käänkuin se perimmältään kertoisi ajan suunnan. Suuret lukumäärät voittaa kisan pieniä lukumääriä vastaan ja siksi ajan on toteltava voittajaa?? Ei, en usko että aika määräytyy tuolta pohjalta vaan sen suunta on välttämättömyys. Eli tajuan ajan suunnan olevan ehdoton "luonnonlaki" jolle ei voi mitään. Aika on mielestäni looginen rakenne jota ei voi purkaa.
Otsikko: Vs: Musta aukko ja sen muoto, tapahtumahorisontti
Kirjoitti: Eusa - 09.02.2020, 15:14:20
Aika muodostuu aineen hitaudesta kullekin hiukkaselle omiaan. Kyseessä on rakenteellinen jatkumo, jossa uusi versio korvaa lähes samassa paikassa edellisen (epävarmuusperiaate). Nimenomaan "alkeishitaus" on oikeastaan entrooppisen kehityksen tae ja sitä kuvaa Standardimallissa Higgsin mekanismi. Higgs voidaan korvata tulkinnalla, jossa avaruusaika määrittyy kullekin paikallistapahtumalle ja on dynaaminen (kaareva) kokonaisuus, jossa alkeishitaus ilmenee tilan muuttumisena ajaksi (inertia, gravitaatio). Inertia-gravitaatio-kenttä on yleisen suhteellisuusteorian kaareutunut avaruusaika - aineen massa edellyttää siten kontribuutiota osana gravitaatiota. Massa tässä yhteydessä tarkoittaa kaikkea energiaa vuorovaikutuksina. Vaikka yleisesti korostetaan, että Higgsin mekanismi vastaa vain alkeishiukkasten massasta/itseisajasta, ilmeiseltä vaikuttaa, että ilman alkeishitautta ei olisi muunkaan rakentuneen aineen massaa/itseisaikaa; alkeishitaus kertautuu. Ja luultavasti alkeishiukkasen massallakin on alimekanisminsa ja avaruudellisuussuhteensa liittyen perusentropiaan, jota ei vielä tarkasti tunneta...
Otsikko: Vs: Musta aukko ja sen muoto, tapahtumahorisontti
Kirjoitti: mistral - 09.02.2020, 17:05:45
Voi kun ymmärtäisi käsitteitä, myös Higgsin kenttä on meikäläisellä jäänyt hämäräksi.
Jokatapauksessa eräänä päivänä sain sellaisen käsityksen ajan nuolesta että on yhdentekevää osoittaako se "itään vai länteen". Olennaista on se että aina tapahtumat etenee todennäköisyyden mukaisesti. Tietysti pienessä hetkessä saattaa tapahtua epätodennäköisiä asioita mutta kun aikaa tulee lisää, paljastuu nuolen suunta. Tämä perustuu siihen että mahdollisuuksien määrä on valtava tulevaisuudessa eli suuremmassa entropiassa kuin pienemmässä entropiassa.
Lyhyesti: entropia ei määrää ajan nuolta, vaan ajan nuoli entropian.

Lasikaappi kuvaus on erikoistapaus, siitä on vaikea päätellä ajan suuntaa jollei kokeen alussa pallot ole siistissä jonossa tms.
Otsikko: Vs: Musta aukko ja sen muoto, tapahtumahorisontti
Kirjoitti: tomipa - 16.05.2020, 06:07:47
Laskin tapahtumahorisontin säteeksi pyörimättömälle, varauksettomalle mustalle aukolle r = GM/c^2. Tämä on auringon massaiselle mustalle aukolle 1477 m. Laskelma myös tuottaa tuloksen, että mustaan aukkoon putoavan hiukkasen nopeus saavuttaa säteen suuntaisen maksimin 2073 m etäisyydelle, ja että tämä säteen suuntainen nopeus tippuu nollaan täsmälleen tapahtumahorisontissa. Siis spekuloin, että putoava aine ei voi koskaan ylittää tapahtumahorisonttia. Tällöin ei tarvita myöskään fysikaalista singulariteettia.
Otsikko: Vs: Musta aukko ja sen muoto, tapahtumahorisontti
Kirjoitti: mistral - 16.05.2020, 11:37:30
Lainaus käyttäjältä: tomipa - 16.05.2020, 06:07:47
Siis spekuloin, että putoava aine ei voi koskaan ylittää tapahtumahorisonttia. Tällöin ei tarvita myöskään fysikaalista singulariteettia.

No ehkä ei singulariteettia mutta tässäkin teoriassa ainetta jää horisontin sisäpuolelle. Se jää sinne kun horisontti kasvaa ajan myötä. Alussa horisontin läpimitta on saattanut olla vain 5km (arvaus) mutta kun aikaa kuluu, se on voinut kasvaa 5 miljoonaan kilometriin. Toinen asia on teoria jossa ei ole horisonttia lainkaan, olisi yksinkertaisempi ymmärtää.
Otsikko: Vs: Musta aukko ja sen muoto, tapahtumahorisontti
Kirjoitti: tomipa - 18.05.2020, 08:15:09
LainaaNo ehkä ei singulariteettia mutta tässäkin teoriassa ainetta jää horisontin sisäpuolelle. Se jää sinne kun horisontti kasvaa ajan myötä.

Intuition mukaan aine jää oskilloimaan 0,5 ja noin 10 Schwarzschild:n säteen väliseen tilavuuteen, mutta en siis ole laskenut, voiko oskillaatio syntyä liike-energiamaksimin ympärille. Aukon kasvaessa oskillaation alaraja siirtyy kauemmas. Aine ei pääse mallissa 0,5 Schwarzschild:n säteen sisäpuolelle.  Se, kuinka tiheää tai kuumaa aine on sallituilla energitiloilla ja mihin suuntaan se säteilee energiaa, on hyvä kysymys.

Noita kahta lukuarvoa lukuun ottamatta muu on spekulaatiota, jota ei kannata jatkaa tässä sanallisin kuvauksin ilman pohjalaskutyötä.
Otsikko: Vs: Musta aukko ja sen muoto, tapahtumahorisontti
Kirjoitti: mistral - 18.05.2020, 19:20:44
Tarkoitatko oskillaatiolla sitä että aika menee taaksepäin horisontin sisäpuolella ja siksi pudotus muuttuu paoksi? Ja materia olisi ikäänkuin tramboliinilla pomppien ja lopulta asettuen "lepotilaan" horisontin pinnalle.
Otsikko: Vs: Musta aukko ja sen muoto, tapahtumahorisontti
Kirjoitti: tomipa - 18.05.2020, 21:49:01
Aika kuluu eteenpäin. Valonnopeuden rajallisuudesta johtuen avaruus kaareutuu aukon läheisyydessä, mikä voidaan mallintaa myös potentiaaliseinäksi Schwarzschildin säteen tuntumassa. Yksinkertaistaen voidaan tehdä analogia klassiseen heiluriin. Mustaan aukkoon putoavan hiukkasen liike-energia on maksimissaan, kun nopeus on maksimissa. Nopeuden lähetessä nollaa tapahtumahorisontissa energian säilymisen nojalla liike-energia muuttuu potentiaalienergiaksi. Tässä potentiaali-liike-energiakuopassa hiukkanen jatkaa oskillaatiota, jos se ei säteile energiaa pois. Dynaamiset mallit voidaan laatia muodostamalla oskillaation differentiaaliyhtälöt huomioimalla myös atsimuutti ja elevaationopeuskomponentit radiaalisen komponentin lisäksi.

En mielelläni linkitä kiistanalaista tai pseudotiedettä, mutta laiton poikkeuksellisesti luonnoksen perusmallista tänne http://www.elisanet.fi/tomi.parviainen/Article_Parviainen.pdf (http://www.elisanet.fi/tomi.parviainen/Article_Parviainen.pdf), koska se on kolme sivua pitkä ja jotta lukijalla olisi samat hypoteesit käytössään kuin itselläni. Mallirakennelma on julkinen, vaikka se osoittautuisi heti tai myöhemmin vääräksi. Mallin mukaan mustan aukon kaikki aine on kertymäkiekossa, mutta olomuotoa en tiedä. En myöskään tiedä, onko malli ristiriidassa suhteellisuusteorian kanssa, joskin Schwarzschildin ratkaisussa esiintyy luku 2. Mielelläni keräisin mallin kaikki puutteet ja korjaisin ne kerralla tai hylkäisin mallin.
Otsikko: Vs: Musta aukko ja sen muoto, tapahtumahorisontti
Kirjoitti: Celest1al Sphere - 19.05.2020, 12:18:06
Lainaus käyttäjältä: tomipa - 18.05.2020, 21:49:01
Aika kuluu eteenpäin. Valonnopeuden rajallisuudesta johtuen avaruus kaareutuu aukon läheisyydessä, mikä voidaan mallintaa myös potentiaaliseinäksi Schwarzschildin säteen tuntumassa. Yksinkertaistaen voidaan tehdä analogia klassiseen heiluriin. Mustaan aukkoon putoavan hiukkasen liike-energia on maksimissaan, kun nopeus on maksimissa. Nopeuden lähetessä nollaa tapahtumahorisontissa energian säilymisen nojalla liike-energia muuttuu potentiaalienergiaksi. Tässä potentiaali-liike-energiakuopassa hiukkanen jatkaa oskillaatiota, jos se ei säteile energiaa pois. Dynaamiset mallit voidaan laatia muodostamalla oskillaation differentiaaliyhtälöt huomioimalla myös atsimuutti ja elevaationopeuskomponentit radiaalisen komponentin lisäksi.

En mielelläni linkitä kiistanalaista tai pseudotiedettä, mutta laiton poikkeuksellisesti luonnoksen perusmallista tänne http://www.elisanet.fi/tomi.parviainen/Article_Parviainen.pdf (http://www.elisanet.fi/tomi.parviainen/Article_Parviainen.pdf), koska se on kolme sivua pitkä ja jotta lukijalla olisi samat hypoteesit käytössään kuin itselläni. Mallirakennelma on julkinen, vaikka se osoittautuisi heti tai myöhemmin vääräksi. Mallin mukaan mustan aukon kaikki aine on kertymäkiekossa, mutta olomuotoa en tiedä. En myöskään tiedä, onko malli ristiriidassa suhteellisuusteorian kanssa, joskin Schwarzschildin ratkaisussa esiintyy luku 2. Mielelläni keräisin mallin kaikki puutteet ja korjaisin ne kerralla tai hylkäisin mallin.

Haasteesi ei ole aivan mitätön, itse Einsteinkään ei onnistunut yhdistämään gravitaatiota ja kvanttimekaniikkaa. Jos mustiin aukkoihin liittyvät ilmiöt olisivat selitettävissä klassisella mekaniikalla, se olisi todennäköisesti jo tehty.
Otsikko: Vs: Musta aukko ja sen muoto, tapahtumahorisontti
Kirjoitti: mistral - 19.05.2020, 21:26:13
Lainaus käyttäjältä: tomipa - 18.05.2020, 21:49:01
Aika kuluu eteenpäin. Valonnopeuden rajallisuudesta johtuen avaruus kaareutuu aukon läheisyydessä, mikä voidaan mallintaa myös potentiaaliseinäksi Schwarzschildin säteen tuntumassa. Yksinkertaistaen voidaan tehdä analogia klassiseen heiluriin. Mustaan aukkoon putoavan hiukkasen liike-energia on maksimissaan, kun nopeus on maksimissa. Nopeuden lähetessä nollaa tapahtumahorisontissa energian säilymisen nojalla liike-energia muuttuu potentiaalienergiaksi. Tässä potentiaali-liike-energiakuopassa hiukkanen jatkaa oskillaatiota, jos se ei säteile energiaa pois. Dynaamiset mallit voidaan laatia muodostamalla oskillaation differentiaaliyhtälöt huomioimalla myös atsimuutti ja elevaationopeuskomponentit radiaalisen komponentin lisäksi.

Ok, siis aika kuluu eteenpäin. En kyllä ymmärrä kuinka omassa koordinaatistossa kiihtyvässä tai putoavassa liikkeessä oleva kappale saadaan meidän koordinaatistossa näyttämään heiluriliikkeeltä. Siis jos potentiaalienergia vähenee ja liike-energia samoin vähenee niin niiden summa pienenee jolloin heiluri jäisi "jumiin" alakuolokohtaan tai oikeastaan klo 6 ja 12 välille.
Otsikko: Vs: Musta aukko ja sen muoto, tapahtumahorisontti
Kirjoitti: tomipa - 20.05.2020, 07:34:42
Kuvaus on tehty koko ajan t_0- tai Euklidisessa koordinaatistossa tai meidän koordinaatistossa. Liike-energialla on yhtälössä maksimi eli liike-energia kasvaa ensin putoamisessa maksimiin ja pienenee siitä nollaan kohti tapahtumahorisonttia. Yhtälö koostuu kahdesta termistä, joilla on eri etumerkki ja eri suuruus.

Ajattelen asiaa energian säilymislain kautta. Kaukana liike-energia on nolla ja energia on potentiaalienergiaa (yhtä suuri kuin työ, joka tehdään massan siirtämiseksi pois potentiaalikaivosta). Massan ollessa vapaassa pudotuksessa liike-energiaa vastaava osuus potentiaalienergiasta pienenee nollaan, mutta tämän jälkeen potentiaalienergia jälleen kasvaa ja nopeus pienenee nollaan. En näe muuta mahdollisuutta nopeuden pienenemiselle kuin, että näennäiskiihtyvyyden suunta vaihtuu, mitä ei tule sekoittaa Newtonin kiihtyvyyteen.

Analogia voisi olla mäkiauto, joka rullaa kahden kukkulan välissä loputtomiin. Analogia on siitä huono, että mäkien käyristymä on sama origon suhteen. Asia on vaikea, ja joudun turvautumaan johdettuihin yhtälöihin. Selittämällä asiaa sanallisesti tulee tehtyä hyvin helposti virhejohtopäätöksiä. Mielelläni kommentoisin vain, jos yhtälöiden johtamisessa, oletuksissa tai hypoteeseissa on virheitä.

Otsikko: Vs: Musta aukko ja sen muoto, tapahtumahorisontti
Kirjoitti: mistral - 20.05.2020, 11:34:48
Nykyinen tieteen paradigma on tapahtumahorisontin kannalla. Itse "toivoisin" että asia muuttuisi sillä lailla että tapahtumahorisontti säilyisi massalle mutta ei fotoneille. Jos fotoneilla ei olisi tapahtumahorisonttia ja ajan hidastuminen riippuisi fotoneista, silloin aika ei pysähtyisikään horisonttiin vaan jatkuisi singulariteettiin saakka. No ei tästä sen enempää.

Hawking hämmästytti maailmaa kun sanoi että maailmankaikkeuden massa voitaisiin nollata jos sen potentiaalienergia otetaan pois. Perustelu oli se että  potentiaalienergia on negatiivista ja näin negatiivisen energian määrä kumoaisi positiivisen ja jäljelle ei jäisi mitään.  Niin tai näin, potentiaalienergia ei voi kasvaa kun laskeudutaan alemmas. Eli jos nollataso on vapaassa avaruudessa, silloin maassa on miinusmerkkinen taso.
Kun hiukkanen putoaa mustaan aukkoon se aina menettää potentiaalia.

Yksi asia on kanssa, kuinka meidän koordinaatistosta käsin voidaan laskea toisen koordinaatiston tapahtumia, tarkoitan sitä että nopeus näennäisesti pysähtyy vaikka horisontin koordinaatistossa kiihtyy. Joka tapauksessa säilymislaki rikkoutuu jos sekä potentiaali että nopeus laskee.

Lisäys: Niin en osaa laskea noita juttuja, joten sen puolesta olen jäävi arvioimaan.

Vielä Hawkingin väitteestä, nyt muistin miten se meni: jos kaikki hiukkaset nostetaan niiden gravitaatiokuopistaan, tarvitaan niin paljon energiaa/massaenergiaa että kaikkien hiukkasten massaenergia hupenisi siihen eikä mitään jäisi jäljelle. Tällainen tilanne tosin olisi kummallinen, silloin jokaisen hiukkasen pitäisi olla hyvin kaukana toisistaan ja avaruuden läpimitta kasvaisi huomattavasti. Lopputulos siis olisi että avaruus olisi täysin tyhjä hiukkasista.
Otsikko: Vs: Musta aukko ja sen muoto, tapahtumahorisontti
Kirjoitti: tomipa - 20.05.2020, 23:23:34
Johtopäätelmät olivat ennenaikaisia. Mallin mukaan mustaan aukkoon tippuva elektroni saa liike-energiaerotusmaksimin (eri kvanttiluvuilla) tapahtumahorisontin tuntumassa ja näillä erotusenergioilla on nollakohta puolikkaan Schwarzschildin säteen kohdalla. Mutta mitä en aiemmin huomannut, mustalla aukolla on erittäin syvä minimierotusenergia atomin mittakaavan kokoluokassa, noin 1e-14 m, mutta ei kuitenkaan origossa. Energia on kvanttitilasta riippuen noin 1e20 eV. Lisäsin malliin uuden kuvan mittakaavoista. En tällä kertaa tee johtopäätöksiä.

Vetäydyn nyt löytöretkeilemään lisää, ja kommentoin aihetta vain, jos mallista löytyy virhe. Voin tehdä yksityisesti yhteistyötä, jos joku kiinnostuu vaihtamaan ajatuksia aiheesta tai haluaa kirjoittaa siitä julkaisun.


Otsikko: Vs: Musta aukko ja sen muoto, tapahtumahorisontti
Kirjoitti: jussi_k_kojootti - 01.09.2020, 15:08:10
Lainaus käyttäjältä: mistral - 09.02.2020, 17:05:45
Olennaista on se että aina tapahtumat etenee todennäköisyyden mukaisesti.

Kyllä; tämä todella vaikuttaa olennaiselta, siis minunkin mielestäni.

Lainaa
Tämä perustuu siihen että mahdollisuuksien määrä on valtava tulevaisuudessa

Tulevaisuudessa, tai ehkäpä vain olevaisuudessa.
Otsikko: Vs: Musta aukko ja sen muoto, tapahtumahorisontti
Kirjoitti: jussi_k_kojootti - 01.09.2020, 15:12:10
Lainaus käyttäjältä: tomipa - 16.05.2020, 06:07:47
Tämä on auringon massaiselle mustalle aukolle 1477 m.

Kakkosella ohi :-) eli tuo on auringon Rs:n puolikas.

Lainaa
Siis spekuloin, että putoava aine ei voi koskaan ylittää tapahtumahorisonttia.

Meidän, ulkopuolisten vinkkelistä ei, mutta putoavan aineen vinkkelistä voi, ja ylittää (https://jila.colorado.edu/~ajsh/bh/schw.html).
Otsikko: Vs: Musta aukko ja sen muoto, tapahtumahorisontti
Kirjoitti: mistral - 02.09.2020, 22:25:18
Lainaus käyttäjältä: ketarax - 01.09.2020, 15:08:10

Tulevaisuudessa, tai ehkäpä vain olevaisuudessa.

Tarkoitat varmaan fysiikkaa jossa aika saadaan sievennettyä pois?
Otsikko: Vs: Musta aukko ja sen muoto, tapahtumahorisontti
Kirjoitti: jussi_k_kojootti - 03.09.2020, 13:53:44
Tarkoitan sitä, että aika vaikuttaisi olevan suhteellista, riippuvan havainnoijasta, tai yleisemmin ajateltuna suhteellisuusteoriasta tutuista viitekehyksistä, frames of reference.  Ehkä todellisuus vain "on", ajattomasti/ajattomana, viitekehyksistä vapaana tai "niiden ulkopuolella", ja aika näyttäytyy vain meidänkaltaisillemme, viitekehyksiin sidotuille materiaalisille entiteeteille. Eli vaikka kiville.

Huh.  Lähtipä heti laukalle.
Otsikko: Vs: Musta aukko ja sen muoto, tapahtumahorisontti
Kirjoitti: mistral - 03.09.2020, 19:09:05
Muistaakseni Brian Greenen kirjasta luin erikoisen jutun, vesiämpäri oli tyhjässä avaruudessa jossa ei ollut muuta materiaa olemassa. Kun vesiämpäriä pyöritetään, alkaa vesi nousta reunojen yli. Kuitenkin on toinenkin mahdollisuus, jospa vesi pysyisikin tasaisena koska pyörimistä ei voinut todistaa ympäristöstä, ympäristö puuttui.
Aika kysymys on vähän samanlainen, jos "ympäristö" puuttuu, käykö "kello" samalla lailla kuin vanhassa viitekehyksessä.
Otsikko: Vs: Musta aukko ja sen muoto, tapahtumahorisontti
Kirjoitti: Pegatsu - 27.01.2022, 19:03:01
En tiedä onko kuvissa ja päätelmissä loogista ajattelua vai onko se 3d mallinnus taidetta enemmän vai onko se itsessään teesi... Toivoisin, että viisaammat ja asiaan perehtyneet valaisisivat asiaa ja kertoisivat, mikäli kuvissa on edes jotain toivon ripettä :) Olin ajattelemassa paperin ja kynän kanssa mustaa aukkoa ja tähän päädyin. Ajatus lähti auringonlaskusta horisontissa, jolle tein omat kiertoradat ja yhtäkkiä olinkin jo mustan aukon ytimessä paperilla. Todellisuudessahan me tiedämme mustista aukoista sen, että emme tiedä niistä mitään.

Tekstikuva menee näin:
Virtausdynamiikka = eteen- ja taaksepäin
Painovoima = ylös- ja alaspäin (kuvassa "kiertorata auringon" kehät)
Vaaka-akseli = Painovoiman vastavoima (Nimesin sen existentiaaliseksi voimaksi/existentiaalisuudeksi) Tapahtumahorisontti?

Ja päätelmä: Painovoimaa ei pystyakselilla voida määrittää sen jatkuvan muutoksen vuoksi. Mikä voima muodostuu horisontaaliselle akselille?`- Nimesin sen tosiaan existentiaaliseksi voimaksi. (mm. paino, leveys, pituus, tilavuus jne...)
Otsikko: Vs: Musta aukko ja sen muoto, tapahtumahorisontti
Kirjoitti: Kaizu - 28.01.2022, 16:00:19
Suhteellisuusteorian läpikäyntiin tämä foorumi sopii huonosti. Suosittelen käyntiä kirjastossa.
Gronström & Lipas, Johdatus sähkdynamiikkaan ja suhteellisuusteoriaan. Sitä olen lukenut itselleni iltasaduksi. Kirja on aika matematiikkapainotteinen ja sitä myötä unettava.
Vaihtoehtoisesti Gamow&Russelin  Herra Tompkinsin uusi maailma. Se käsittelee suhteellisuusteoriaa hieman vähemmällä matematiikalla. Herra Tompkinsia olen lukenut lapsenlapsille iltasaduksi.

Kaizu
Otsikko: Vs: Musta aukko ja sen muoto, tapahtumahorisontti
Kirjoitti: mistral - 12.05.2022, 17:49:00
Kuva oman Linnunratamme keskusaukosta on julkaistu. Läpi kaasupilvien otettu kuva oli varmaan vaikeampi kuin M87 kuva, todella hieno suoritus.
Sitä ihmettelen, kuinka kertymäkiekko näkyy päältä kuvattuna vaikka SagittariusA:n kertymä on tullut Linnunradan kiekosta ja me olemme sen litteässä tasossa. Periaatteessa sama kuin oman aurinkomme historia, se on kerännyt pyörimisliikemääränsä aurinkokunnasta. Näin voisi kuvitella että Sagittariuksen kuva on kallistettu jotta sen mieltää paremmin.
Otsikko: Vs: Musta aukko ja sen muoto, tapahtumahorisontti
Kirjoitti: Pappis - 12.05.2022, 20:53:02
Selitys on tässä videossa: https://youtu.be/zUyH3XhpLTo
Otsikko: Vs: Musta aukko ja sen muoto, tapahtumahorisontti
Kirjoitti: mistral - 12.05.2022, 21:37:39
Ok, tuo käy järkeen. Toisaalta voidaan kysyä, antaako kuvajainen saman valotehon kuin kiekon katsominen antaisi kohtisuorassa linjassa? Niin vielä "varjosta", eikö se ole kooltaan 1,5x rS?

Edit. Katsoin videon uudestaan ja siinähän se sanottiin, varjo oli 2,6x rS (S tarkoittaa Schwarzschildin sädettä), no nyt sekin selvisi :)

Eli varjosta on tapahtumahorisonttia vain n. 15%?
Otsikko: Vs: Musta aukko ja sen muoto, tapahtumahorisontti
Kirjoitti: Mare Nectaris - 13.05.2022, 12:07:39
Tutkijoiden mukaan (vastoin odotuksia) mustan aukon pyörimisakseli osoittaa Maahan päin: "The EHT team conducted supercomputer simulations to compare with their data, and concluded that Sagittarius A* is probably rotating along an axis that roughly points along the line of sight to Earth. The direction of that rotation is anticlockwise, Gómez said.

"What blows my mind is that we're seeing it face-on," says Regina Caputo, an astrophysicist at NASA–Goddard Space Flight Center in Greenbelt, Maryland. NASA's Fermi Gamma-Ray Space Telescope, which Caputo works with, had previously detected giant glowing features above and below the centre of the galaxy, which could have been produced by Sagittarius A* during periods of intense activity in the past. But those features, known as Fermi bubbles, would seem to require matter to swirl around the black hole edge-on as seen from Earth, rather than face-on."

Lainauksen lähde: https://www.nature.com/articles/d41586-022-01320-y
Otsikko: Vs: Musta aukko ja sen muoto, tapahtumahorisontti
Kirjoitti: mistral - 13.05.2022, 23:27:52
Naturen artikkeli yllättää koska on kahdenlaista todistetta, kuplat Linnunradan molemmilla puolilla ja toisaalta simulaatiot puhuvat sen puolesta että Sgt.A pyörisi akseli Maahan päin. Nämä on ristiriidassa keskenään, erikoinen tilanne. Tuli mieleen, voisiko molemmat olla totta, eli kuplat on syntyneet kauan sitten kun Linnunrata oli eri asennossa ja jokin galaksi olisi vääntänyt Linnunradan 90 astetta eri asentoon? Tuskin mahdollista...
Otsikko: Vs: Musta aukko ja sen muoto, tapahtumahorisontti
Kirjoitti: Pappis - 14.05.2022, 09:49:58
Ei Linnunradan asento ole välttämättä muuttunut mihinkään. Sen mustan aukon asentoa on helpompi muuttaa, ja ei sen pyörimisakselin tarvitse olla kuin villissä hyrräliikkeessä ulkoisten voimien tai aiempien yhdistymisten takia, niin tilanne voi nyt näyttää siltä mitä se on...
Otsikko: Vs: Musta aukko ja sen muoto, tapahtumahorisontti
Kirjoitti: Mare Nectaris - 14.05.2022, 14:30:30
Tässä vielä linkki aihetta koskeviin artikkeleihin The Astrophysical Journal Letters -julkaisussa (linkit tekstin lopussa):

https://iopscience.iop.org/journal/2041-8205/page/Focus_on_First_Sgr_A_Results
Otsikko: Vs: Musta aukko ja sen muoto, tapahtumahorisontti
Kirjoitti: Lithos - 14.05.2022, 15:07:12
Loogisesti todennäköisin vaihtoehto omasta mielestäni on että kuplat ovat syntyneet kauan sitten kun linnunrata oli nuori ja musta aukko tosi aktiivinen. Törmäys esim toisen galaksin kanssa olisi myöhemmin voinut heilauttaa mustan aukon eri asentoon, ja nykyisellä aktiivisuudellaan se ei uusia kuplia synnytä kuten ennen.

Minkäänlaista tieteellistä pohjaa tällä pohdiskelullani ei ole, ainoastaan oma tulkintani julkaissusta tiedosta.
Otsikko: Vs: Musta aukko ja sen muoto, tapahtumahorisontti
Kirjoitti: Eusa - 15.05.2022, 06:11:14
Sellainenkin valitettava mahdollisuus on otettava huomioon, että kuvan koostamismenetelmä tekee renkaan, eikä sitä kohteessa todellisuudessa olisikaan.
Otsikko: Vs: Musta aukko ja sen muoto, tapahtumahorisontti
Kirjoitti: mistral - 15.05.2022, 17:28:23
Lainaus käyttäjältä: Eusa - 15.05.2022, 06:11:14
Sellainenkin valitettava mahdollisuus on otettava huomioon, että kuvan koostamismenetelmä tekee renkaan, eikä sitä kohteessa todellisuudessa olisikaan.

Niin jos kuva on koostettu monista yksittäisistä kuvista mutta eikö yksittäisissäkin ollut renkaan hahmo? Jokatapauksessa radioaalloista kuva on syntynyt, oli niin iso tiedosto ettei pystynyt siirtämään internetissä vaan levyinä rahdattiin.
Otsikko: Vs: Musta aukko ja sen muoto, tapahtumahorisontti
Kirjoitti: Kaizu - 15.05.2022, 23:59:27
Kuva ei ole valokuva vaan interferometriksi kytkettyjen antennien datasta laskettu kuva. Eli ei ole tehty samalla lailla pinoamalla kuin tähtivalokuvat. Käytetty aallonpituus ei ole näkyvää valoa vaan millimetrialueen radioaaltoja.
n.3,5mm =>86GHz

Kaizu
Otsikko: Vs: Musta aukko ja sen muoto, tapahtumahorisontti
Kirjoitti: ikekokkiik - 16.05.2022, 20:11:07
Lainaus käyttäjältä: Lorelei - 22.09.2015, 20:24:31
Tämä ei nyt liity ihan aiheeseen, mutta jotenkin on ristiriitaista, että esim fyysikot (luonnontieteilijät jne) sanovat selittävänsä "kaiken" oman tieteellisen systeeminsä puitteissa. Mitään mystistä ei ole, ja sitä rataa. Silti singulariteetti "selitetään" sillä, ettei se nyt (hupsis) mahdu kontekstiin, se on siis se Juttu aukottomassa tieteenteossa joka ei mahdu kuvioon... :azn: Se on siis järjen ulottumattomissa, mitä ei tosin kai saisi sanoa ääneen.
Kyllä, onhan se totta, että luonnontieteilijöillä hieman itsekeskeinen tavoite on kaikki selittää omien tieteellisten systeemiensä puitteissa. Se ei kuitenkaan tarkoita sitä, että kyseiset asiat olisivat helposti selitettävissä. On kuitenkin niin paljon mitä mahtipontinen ihmismieli ei voi ymmärtää tai käsittää. Juuri ja juuri, kun alkuräjähdyksenkin voimme selittää.

Olet hyvinkin oikeassa, tieteessä ei ääneen puhuta järjen ulottumattomista asioista (tässä vaiheessa filosofia tulee kuvioihin), mutta olemme sentään ymmärtäneet esimerkiksi juuri singulariteettien olemassaolon. Pystymme sanomaan että ne ovat, muttemme miksi. Tässä lieneekin ihmisyyden ja tieteen vaikeus. Saa nähdä mitä tuleman pitää...
Otsikko: Vs: Musta aukko ja sen muoto, tapahtumahorisontti
Kirjoitti: mistral - 18.05.2022, 01:01:55
Lainaus käyttäjältä: Kaizu - 15.05.2022, 23:59:27
Kuva ei ole valokuva vaan interferometriksi kytkettyjen antennien datasta laskettu kuva. Eli ei ole tehty samalla lailla pinoamalla kuin tähtivalokuvat. Käytetty aallonpituus ei ole näkyvää valoa vaan millimetrialueen radioaaltoja.
n.3,5mm =>86GHz

Kaizu

Eikös se ole digitaalinen kuva jossa informaatio on pakattu tiedostoiksi, jokaisen ajanhetken tiedosto on erilainen koska muutoksia tapahtuu kuin elokuvassa. Siitä sitten on tehty kompromissikuva. Mutta kuinka kuva rakennetaan, on arvoitus. Itse tarjosin M87:n julkaisun aikaan trigonometristä laskentaa siihen, eli trig kolmiot menisi eri antennien kesken ja niillä saataisiin se resoluutio. Tiedä sitten onko noin mutta jotenkin ne saavuttaa huikean resoluution, sehän vastaa samaa kuin munkkirinkilä kuussa.
Otsikko: Vs: Musta aukko ja sen muoto, tapahtumahorisontti
Kirjoitti: Eusa - 20.05.2022, 20:50:06
Lainaus käyttäjältä: mistral - 15.05.2022, 17:28:23
Niin jos kuva on koostettu monista yksittäisistä kuvista mutta eikö yksittäisissäkin ollut renkaan hahmo? Jokatapauksessa radioaalloista kuva on syntynyt, oli niin iso tiedosto ettei pystynyt siirtämään internetissä vaan levyinä rahdattiin.
Simuloitua kuvaa haetaan olettamalla rengasmainen kertymäkiekko. Havainnoista haetaan ennusteeseen soveltuvia piirteitä, jotka kelpaavat ennakoituihin virtuaalimalleihin.

https://iopscience.iop.org/article/10.3847/2041-8213/ab0e85

Onkohan tuossa turhan paljon viritystä siihen mitä odotetaan näkyvän.
Otsikko: Vs: Musta aukko ja sen muoto, tapahtumahorisontti
Kirjoitti: mistral - 20.05.2022, 23:19:45
Lainaus käyttäjältä: Eusa - 20.05.2022, 20:50:06
Simuloitua kuvaa haetaan olettamalla rengasmainen kertymäkiekko. Havainnoista haetaan ennusteeseen soveltuvia piirteitä, jotka kelpaavat ennakoituihin virtuaalimalleihin.

https://iopscience.iop.org/article/10.3847/2041-8213/ab0e85

Onkohan tuossa turhan paljon viritystä siihen mitä odotetaan näkyvän.

Linkissä oli avainsana: very long baseline interferometry (VLBI). Eikös tämä ole "hyvin pitkä kanta interferometria". Itse ymmärrän interferometrin toimivan niin että tunnettu 'räjähdys', 'sykäys' tai mikä tahansa kiintopiste kertymäkiekosta on tunnistettavissa kaikilla radioantenneilla Maassa. Nyt kun kaikki tunnistaa saman tapahtuman atomikellon tarkkuudella, saadaan radioaalto informaatio samanaikaistettua, tämä on ensimmäinen tekijä. Seuraava tekijä on kannan eli linjan mittaus, mitataan kaikkien lautasten linjat toisiinsa nähden ristiin rastiin. Seuraavaksi piirretään kolmio avaruuteen jossa tunnetaan kanta ja yksi kateetti. Kateetti selviää atomikellojen antaman viiveen kautta, esm jos viive on 0,01 sekuntia niin se n. 3000km valon nopeudella. Kun kanta ja kateetti nyt tiedetään, eikö silloin toinenkin kateetti selviä? Näin kolmion yksi sivu osoittaa tarkkaan kohti kertymäkiekkoa. Ja ristiin mittaamalla eri lautasten välillä löytyy tarkka paikka.
Toki lautasten eri korkeus maan pinnasta antaa atomikelloille eri ajankulun mutta eiköhän sekin saada kompensoitua. Ja Maan pyörimisliike muuttaa kolmioiden muotoja jatkuvasti mutta sekin on huomioitava.
Näin olen ymmärtänyt interferometrin periaatteen ja jos kaikki tekijät on oikein, pitäisi tuloksenkin olla oikein. Mutta valtavasti muuttujia on etten ihmettele jos jokin ei täsmää.